סיקור מקיף

SpaceX שיגרה מכונית לחלל העמוק במשגר החזק בעולם

טיסת הבכורה של הפאלקון הכבד, המשגר החדש של SpaceX, הושלמה בהצלחה, והפכה אותו למשגר הפעיל החזק ביותר בעולם. הוא שיגר לחלל העמוק את מכונית הטסלה רודסטר של אילון מאסק, עם בובת אסטרונאוט במושב הנהג. הנחיתה של שלושת השלבים הראשונים, מהם מורכב המשגר, בוצעה באופן חלקי: שני השלבים הצדיים ביצעו נחיתה כפולה ומרהיבה בקייפ קנוורל, אך השלב האמצעי התרסק באוקיינוס האטלנטי.

הטסלה רודסטר, ובובת האסטרונאוט, על רקע כדור הארץ. לא זו לא בדיחה. מקור: SpaceX.
הטסלה רודסטר, ובובת האסטרונאוט “סטארמן”, על רקע כדור הארץ. לא זו לא בדיחה. מקור: SpaceX.

עדכון, 9/2/18: הידיעה עודכנה על מנת לציין כי מכונית הטסלה לא תגיע לחגורת האסטרואידים, כפי שאילון מאסק טען בחשבון הטוויטר שלו. מסלולה יגיעה קצת מעבר למסלול מאדים, כפי שתכננה החברה במקור.

לעיני הצופים הרבים שבאו במיוחד כדי לצפות בשיגור מקייפ קנוורל, ומיליוני צופים נוספים מרחבי העולם, הפאלקון הכבד המריא אתמול (שלישי), 15:45 שעון מקומי, לטיסת בכורה מוצלחת ומדהימה, מכן השיגור LC-39A במרכז החלל קנדי בפלורידה.

הפאלקון הכבד הוא המשגר החדש של חברת ספייס אקס (SpaceX). הוא מבוסס על המשגר הרגיל של חברת שיגורי החלל, פאלקון 9, משגר ענק בפני עצמו שמתנשא לגובה של 70 מטר. למשגר פאלקון 9 יש שני שלבים – שלב ראשון מרכזי ורב-שימושי, ושלב עליון קטן יותר וחד-פעמי. לפאלקון הכבד יש שני שלבים ראשונים נוספים, המחוברים לשלב הראשון המרכזי ומשמשים כמאיצי צד. באופן זה, יכולת השיגור לחלל של המשגר גדלה באופן משמעותי, ולאחר טיסת המבחן המוצלחת הוא הפך באופן רשמי למשגר הפעיל החזק ביותר בעולם.

למרות חשיבותו הרבה של המשגר, שלשם הדגמת ביצועיו בוצעה הטיסה אתמול, גם המטען שלו סיפק ריגוש ופליאה רבים. המטען המופלא היה מכונית הטסלה רודסטר האישית של מייסד ומנכ”ל החברה אילון מאסק, עם בובת אסטרונאוט במושב הנהג, שנכנסו למסלול אליפטי סביב השמש, בו יגיעו בנקודה הרחוקה ביותר קצת מעבר למרחקו של מאדים מהשמש.

“אקרובטיקת חלל”

המראת הפאלקון הכבד מכן השיגור במרכז החלל קנדי. מקור: SpaceX.
המראת הפאלקון הכבד מכן השיגור במרכז החלל קנדי. מקור: SpaceX.

מי שצפה בשידור החי מהשיגור, לא יכל היה שלא להתרשם מהמספר הרב של שלבים, פרטים ואירועים, מרשימים כל אחד בפני עצמו, שאירעו בזמן כל כך קצר. התיאור של אילון מאסק לנחיתה המשולבת המדהימה של שני השלבים הראשונים זה לצד זה, “אקרובטיקת חלל”, מתאים אולי גם לכל שלבי הטיסה המסובכים. להלן פירוטם:

ההמראה, שנקבעה תחילה ל-13:30 שעון מקומי, נדחתה מספר פעמים עקב רוחות גבוהות ומהירות מדי, אך בסופו של דבר המשגר המריא מכן השיגור בשעה 15:45, לקול נפץ אדיר וענני פליטה כבירים.

שתיים וחצי דקות לאחר ההמראה, שני שלבי הצד של הפאלקון הכבד התנתקו מהשלב האמצעי, והמוטות המחברות ביניהם לבין השלב האמצעי התקפלו בחזרה אל גוף כל אחד משלושת השלבים.

בעוד השלב הראשון המרכזי ממשיך לבעור ולהאיץ את הרכב לחלל, שלבי הצד החלו את תהליך הנחיתה הכפול שלהם, שנראה קצת כמו מחזה מסרט מדע בדיוני. בתיאום כמעט מושלם, שני השלבים נחתו על שני מנחתים יבשתיים צמודים בבסיס חיל האוויר קייפ קנוורל (הסמוך למרכז החלל קנדי, ממנו המריאו דקות ספורות קודם לכן).

"אקרובטיקת חלל" - הנחיתה הכפולה, המזכירה קטע מסרט מדע בדיוני. מקור: SpaceX.
“אקרובטיקת חלל” – הנחיתה הכפולה, המזכירה קטע מסרט מדע בדיוני. מקור: SpaceX.

שלבי הנחיתה כללו ביצוע בעירה רקטית ראשונית על מנת לחזור לעבר כיוון קייפ קנוורל (Boostback Burn), בעירה שנייה במהלך החדירה לאטמוספירה (Entry Burn), ובעירה שלישית ואחרונה על מנת לבצע נחיתה רכה ביבשה (Landing Burn). בנוסף, על מנת להתכוונן במרחב, השלבים משתמשים במערכת של מדחפי גז קר וסנפירי רשת.

כ-40 שניות לאחר ההתנתקות משני שלבי הצד, השלב הראשון המרכזי התנתק מהשלב העליון, והחל לחזור גם הוא לכדור הארץ, הפעם לנחיתה על אסדה ימית ניידת בלב האוקיינוס האטלנטי. בניגוד לשני שלבי הצד שנחתו בהצלחה, אילון מאסק מסר שלשלב המרכזי נגמר הדלק לנחיתה על האסדה, והוא התרסק עקב כך על המים במהירות של כ-480 קמ”ש, כ-100 מטר ליד האסדה. לא ברור לגמרי מה מצב האסדה, אך מאסק אמר שההתרסקות הרסה שני מדחפים בה, ושלחה “מטר של רסיסים על הסיפון”.

במקביל, השלב העליון המשיך את שלבי השיגור הסופיים, באמצעות המנוע היחיד שלו, מסוג מרלין, המותאם לפעולה בריק.

בערך כארבע דקות לאחר ההמראה, מעטפת המטען, שמטרתה להגן על המטען המשוגר מפני האטמוספירה, התנתקה ושני חלקיה נפלו בחזרה לכדור הארץ.

נסיעה של מיליארד שנים בחלל העמוק

כדור הארץ ממושב הנהג של מכונית הטסלה. מקור: SpaceX.
כדור הארץ ממושב הנהג של מכונית הטסלה. מקור: SpaceX.

לאחר שמעטפת המטען התנתקה, נחשף המטען המפליא – יש יאמרו ראוותני – שהוצב בחרטום הטיל: מכונית טסלה רודסטר אדומה, שהיא רכושו הפרטי של אילון מאסק. איל ההון הדגיש כי מכיוון שזו הייתה טיסת ניסוי ראשונה של משגר חדש עם רמת סיכון גבוהה מאד, אף לקוח לא היה מסכים לסכן מטען יקר ערך כמו לוויין. לדבריו, האפשרות האחרת, של הצבת מדמה-מסה סטנדרטי כגון בטון – הייתה “משעממת מדי”.

אם לא די בשיגור מכונית לחלל – במושב הנהג הוצבה בובה המכונה “סטארמן”, שהולבשה בדגם אמיתי של חליפת החלל שפיתחה ספייס אקס. החליפה תשמש אסטרונאוטים של נאס”א שהחברה תשגר בעתיד לתחנת החלל הבינלאומית, באמצעות החללית דרגון 2 שהיא מפתחת בימים אלו.

אם נחזור רגע לשלבי הטיסה – הם עדיין לא נגמרו – לאחר בעירה ראשונה של השלב העליון, הוא מופעל מחדש לזמן קצר על מנת להכניס את המכונית ל”מסלול חנייה” סביב כדור הארץ. מסלול ראשוני זה העביר את המכונית והשלב העליון דרך חגורות הקרינה ואן אלן. מאסק ציין כי זהו אחד מגורמי הסיכון (הרבים) של הטיסה, וחששו היה שהקרינה העזה עשויה לפגוע באלקטרוניקה של השלב העליון.

לאחר שיוט בן שש שעות במסלול החנייה הראשוני, המנוע הרקטי של השלב העליון הודלק מחדש, והאיץ את מכונית החלל למהירות מילוט מכוח הכובד של כדור הארץ ואל יעדה הסופי – מסלול אליפטי סביב השמש.

אילון מאסק אמר כי הבעירה של השלב העליון הייתה חזקה מהמצופה, וכי המכונית תגיע במסלולה עד למרחק של חגורת האסטרואידים, מעבר למרחק המקסימלי שתוכנן במקור – הדומה לזה של מאדים מהשמש. יחד עם זאת, אסטרונומים מרחבי העולם הבחינו שהמספרים שסיפק לא עולים בקנה אחד עם מסלול כזה. ספייס אקס סיפקה מאוחר יותר עדכון על נתוני המסלול, שמראים כי בסופו של דבר מסלולה יגיע אמנם אל מעבר למרחק מאדים מהשמש, אך לא לחגורת האסטרואידים.

זווית ממצלמה נוספת - בה נראית היטב הבובה "סטראמן" לבושה בחליפת החלל של ספייס אקס. מקור: SpaceX.
זווית ממצלמה נוספת – בה נראית היטב הבובה “סטראמן” לבושה בחליפת החלל של ספייס אקס. מקור: SpaceX.

למכונית צורפו שלוש מצלמות ששידרו צילומים חיים מרהיבים, כמעט סוריאליסטיים, ממנה ומבובת האסטרונאוט במסלול הראשוני סביב כדור הארץ. למכונית בטריות שמספיקות רק למשך 12 שעות, ולאחר מכן לא תוכל להמשיך לצלם ולתקשר עם כדור הארץ.

הפאלקון הכבד – המשגר הפעיל החזק ביותר בעולם

לספייס אקס לקח שבע שנים להוציא לפועל את השיגור המוצלח, לאחר שאילון מאסק הכריז על פיתוח הפאלקון הכבד בשנת 2011. פיתוח הרכב העוצמתי לא היה משימה פשוטה עבור החברה, והשיגור, שתוכנן במקור ל-2013, נדחה שוב ושוב מאז.

בין הבעיות שעמן נאלצה החברה להתמודד בפיתוח המשגר, הייתה המספר הרב של מנועיו. לפאלקון 9 רגיל יש 9 מנועים, ומכאן שמו. לגרסה הכבדה, עם שלושה שלבים ראשונים, יש 27 מנועים בסך הכל – יותר מכל משגר אמריקאי אחר בהיסטוריה (הוא שני רק ל-N-1 הסובייטי – לו היו 30 מנועים, ועקב הסיבוך הרב בתפעול כל כך הרבה מנועים, כל שיגוריו הסתיימו בכישלון גמור).

בנוסף, שני שלבי הצד יצרו עומסים חזקים על השלב הראשון המרכזי, ועקב כך החברה נאלצה לעצב מחדש את שלדתו. “תחילה, זה נשמע מאד קל: פשוט תדביק שני שלבים ראשונים [של פאלקון 9] כמאיצי צד. כמה קשה זה יהיה? אבל אז הכל משתנה. כל העומסים משתנים, האווירודינמיקה משתנה לגמרי, ושילשת את הוויברציות והאקוסטיקה”, אמר בעבר מאסק על פיתוח הפאלקון הכבד.

מאסק חשף היום, במסיבת עיתונאים שקיים לאחר השיגור, כי החברה שקלה שלוש פעמים בעבר לבטל את תוכנית הפאלקון הכבד, עקב קשיי הפיתוח שלו. לדבריו, החברה השקיעה עליו לפחות חצי מיליארד דולר, ואולי אף יותר מכך.

למרות הקשיים הללו, החברה הוכיחה אתמול שעיצוב המשגר תקין, ושהוא מסוגל לא רק להרים עצמו מעל כן השיגור, אלא להכניס מטען לעבר מסלול בין-פלנטרי בחלל העמוק.

אז מה בדיוק יכולותיו של המשגר החדש? וכיצד הוא משתווה למשגרים אחרים בהיסטוריה? הפאלקון הכבד מסוגל ליצור דחף בעוצמה של 22,819 קילו-ניוטון, המשתווה בערך לזה שיוצרים ביחד 18 מטוסי בואינג 747. ביחס למשגרים פעילים היום, מדובר בכוח הדחף החזק ביותר עבור משגר כלשהו. יחד עם זאת, מעבורת החלל ומשגר הסטורן 5, שניהם כבר לא פעילים, היו חזקים ממנו מבחינה זו.

הסטורן 5 היה חזק ממנו גם מבחינת יכולת נשיאת מטען לחלל, עם כ-140 טון שיכל להכניס למסלול נמוך-ארץ, בעוד הפאלקון הכבד מסוגל לשגר 63,800 ק”ג למסלול נמוך-ארץ (בתצורה חד-פעמית בה הוא מכלה את כלל הדלק שלו, ושלושת השלבים נשרפים באטמוספירה). ראוי לציין, כי אם נחשיב את משקל מעבורת החלל עצמה, ולא רק את המטען שיכול להיפרס מתא המטען שלה לחלל, אז גם היא הייתה חזקה יותר מהפאלקון הכבד מבחינת כושר נשיאה.

27 המנועים של המשגר פאלקון כבד. מקור: דף הטוויטר של אילון מאסק.
27 המנועים של המשגר פאלקון כבד. מקור: דף הטוויטר של אילון מאסק.

ביחס ליכולתו של מי שהחזיק עד אתמול בתואר “המשגר הפעיל החזק ביותר”, דלתא 4 כבד של המתחרה ULA, שמסוגל לשגר 28,370 ק”ג למסלול נמוך-ארץ, לפאלקון הכבד יש כושר נשיאה יותר מכפול. חשוב יותר מכך הוא ההבדל במחיר בין השניים – בעוד הדלתא 4 כבד עולה בסביבות ה-350 מיליון דולר, ספייס אקס מוכרת משגר פאלקון כבד ב-90 מיליון דולר בלבד.

מה הלאה? והאם הפאלקון הכבד ישגר בני אדם לירח?

הפאלקון הכבד, כך לפי אילון מאסק, יאפשר לחברה לשגר את סוגי הלוויינים הכבדים ביותר שקיימים בתעשייה. למרות זאת, אין עדיין הרבה לקוחות שמחכים בתור לשיגור על גביו. בהמשך השנה מתוכננות עבורו רק שתי משימות – שיגור לוויין תקשורת של ערב הסעודית, ומשימת הדגמה של חיל האוויר האמריקאי.

החברה מתכוונת להשתמש במשגר כדי להתחרות בשיגורי לוויינים בטחוניים כבדים של משרד הגנה האמריקאי, שעד היום רק הדלתא 4 כבד יכל לשגר לחלל.

המחיר הנמוך, ביחד עם יכולותיו הגבהות, עשויים גם לעודד את נאס”א לבחור בפאלקון הכבד עבור משימות למאדים וגרמי שמיים אחרים במערכת השמש. גם הממשל האמריקני הגיב באופן חיובי לשיגור אתמול. הנשיא דונלד טראמפ עצמו צייץ בחשבון הטוויטר ברכות לאילון מאסק וחברת ספייס אקס:

אבל הפאלקון הכבד ככל הנראה לא ישגר בני אדם לחלל, לפחות לא לפי הכוונות הנוכחיות של אילון מאסק. שלשום, במסיבת עיתונאים טלפונית לפני השיגור, מאסק חשף שהחברה כבר לא מתכננת להשתמש בו עבור שיגור שני תיירים למסלול סביב הירח, תוכנית עליה הכריזה החברה בשנה שעברה. מאסק ציין כי פיתוח חללית הענק העתיד(נ)ית שלה, BFR – חללית רב-שימושית לחלוטין שמיועדת לשאת 100 איש לכוכב הלכת האדום – מתקדם במהירות, וכי החברה לא מעוניינת להשקיע את המשאבים הנדרשים להכשרת הפאלקון הכבד לצורך משימות מאויישות. לדבריו, רק אם פיתוח ה-BFR יתקל בעיכובים, הוא ישקול מחדש לשגר בני אדם על גבי הפאלקון הכבד.

מאסק לא מתכוון כנראה לעצור לרגע, גם לא לאחר ששיגר את הטיל העוצמתי ביותר בעולם, וכבר מתכונן לשיגור רכב חלל עוצמתי הרבה יותר. “[השיגור של הפאלקון הכבד] נותן לי בטחון שאנחנו יכולים לגרום לעיצוב של ה-BFR לעבוד. אני חושב שאנחנו יכולים להמשיך בקידום הטכנולוגיה כדי להשיג יכולת של רב-שימושיות מהירה ומלאה”, אמר במסיבת העיתונאים היום מאסק.

צפו בשידור המלא מהשיגור:

ראו עוד בנושא באתר הידען:

252 תגובות

  1. צריך אחת ולתמיד לסגור את השטות הזו של העולם השטוח לפני שעוד איזה תמהוני ישגר עצמו לחלל בטיל.

    אין דבר כזה עולם שטוח, אין! כמה אפשר לחזור על זה ילדים באמת, מה אתם לא מבינים, העולם אינו שטוח!!

    הוא מרובע.

  2. מתן
    נהניתי 🙂 אבל … הובכתי …. בחיי שחשבתי שאתה בן 12. בן 12 עם תסמונת דנינג-קרוגר. טעות שלי!!!

    מתן – הבאתי לך כמה ראיות לצורת כדור-הארץ, ולא הצלחת לסתור אף אחת מהן. אתה הבאת מספר ראיות לכאורה לזה שהארץ שטוחה – וסתרנו את כולן.

    בוא נתכרז בתצפית אחת ויחידה, ובבקשה – נסה להפריך אותה ביושרה. ובכן – איך אתה מסביר גאות ושפל. בפרט – למה יש 2 מחזורי גאות-שפל ביום?

    בוא נעזוב את העבר. אתה חושב שכל המדענים שקרנים, ואני חושב שכל אנשי הדת שקרנים – ויכוח על העבר לא יקדם אותנו. נסה רק לענות על השאלה שלי.

  3. היי מתן, הייתי שמח להמשיך את הוויכוח ביננו עד שנגיע להכרעה, כך או כך!

    מה אתה אומר?

    דרך אגב, לדעתי כדאי למצוא פלטפורמה אחרת לנהל את את הדיון הזה, זה יכול להיות פייסבוק או סתם מייל, אני פתוח להצעות.

  4. ישראל,

    אי אפשר לתת לך תשובה של כן או לא, מפני שכפי שאני מנסה להסביר לך – השאלה שלך לא מוגדרת היטב. הנחת היסוד שלך היא פוטון שנפלט מנקודה A לכיוון B. כלומר, פוטון שברגע התחלתי מסוים ממוקם ב-A *וגם* יש לו תנע מוגדר (אתה הרי טוען שהוא נפלט לכיוון B, לכן חייב להיות לו תנע מכיוון A ל-B). לפני ששאלת את השאלה, כבר סתרת את מכניקת הקוונטים. אי אפשר לענות על השאלה הזאת במסגרת התיאוריה.

    אם תסתכל על פוטון שנפלט ברגע מסוים עם תנע P, אז הוא לא נפלט מנקודה מסוימת במרחב ואין משמעות לשאלה איזה מרחק עבר. הוא היה מרוח במרחב ברגע הפליטה, והוא נשאר מרוח במרחב אחרי 3 שניות. אם תרצה לוותר דווקא על ההנחה השניה (כלומר, פוטון שנפלט מנקודה A אבל אין לו תנע מוגדר) אז כדי לתת משמעות למיקום שלו אחרי 3 שניות צריך לבצע מדידה, כלומר להתערב ולשנות את המערכת הקוונטית. תוצאת מדידה זו תמיד תניב נקודה כלשהי במרחק 3 שניות אור מ-A אבל כמו שאמרתי שוב ושוב, זוהי מדידה וככזו היא אקט הרסני למערכת. כלומר, ישנו עולם שלם של אפקטים פיזקליים שנמצאים מעבר למדידה(כמו ההתאבכות בניסוי שני הסדקים), ככה שהמדידה בנקודה A בזמן הפליטה והמדידה בנקודה במרחק 3 שניות אור אחרי 3 שניות ממש לא מספיקות לתיאור המערכת כולה. זה כן נכון בפיזיקה קלאסית, אבל לא במכניקה קוונטית. במהלך 3 השניות שהחלקיק לא נמדד הוא היה יכול לבצע אינטראקציות קוונטיות עם הסביבה שלו.

    כל קו השאלות הזה נענה כבר בעבר.

    1. אין לי מושג על מה אתה מדבר ואני לא הולך לנבור אחורה בתגובות. טרנספורמציית לורנץ היא סוג של סימטריה. מה זה אומר “מה הקשר בין CMB לטרנספורמציית לורנץ”? מה הקשר בין שבתאי לסימטריה לסיבובים?

    2. למיטב ידיעתי לא, אבל אני לא מומחה לתחום. אני עונה מהמותן ואולי מפספס משהו. באופן עקרוני, אם אתה מודד קרינה ממקור בלי לדעת מה הם מאפייני הפליטה שלו (כמו עוצמה), לא תוכל לדעת מה המרחק. יש מאפייני פליטה אחרים (למשל אם אתה יודע מה הספקטרום של הפליטה ואתה יודע מה המהירות היחסית אתה יכול להשתמש באפקט דופלר כדי למצוא את המרחק), אבל תמיד צריך איזושהיא כמות של מידע על הפליטה והסביבה.

    3. הסברתי לך כבר בעבר, כולל דוגמאות מפורשות. אין שום סתירה בין קיום של מערכת מנוחה לבין הומוגניות. אם תקח למשל נייר ותצייר עליו סריג של חלקיקים במרחקים שווים, הוא יהיה הומוגני (בכל נקודה צפיפות הנקודות שווה). אם תבצע טרנספורמציית לורנץ, הוא עדיין יהיה הומוגני, למרות שהמרחקים בין החלקיקים לא יהיו שווים יותר בשני הצירים. הצפיפות תשאר בלתי תלויה במיקום ולכן התווך יהיה הומוגני.

  5. ישראל
    במאמר הזה יש הסבר ונוסחה להבנת אפקט דופלר על קרינת הרקע – https://physics.stackexchange.com/questions/206100/velocity-of-solar-system-relative-to-cmb

    לא צריך התכווצות לורנץ, כי המהירות לא גבוהה במיוחד.

    אין כל דרך לדעת את המרחק לקורן. מה שאתה קולט זה פוטונים. הם לא מזדקנים עם הזמן.
    אם תדע תכונות של הקורן והתווך – תוכל להאריך את המרחק. למשל – אם זה קרן לייזר בעל פיזור מסויים וידוע, בריק, אז לפי העוצמה תוכל להאריך את המרחק.
    אם תנוע – אולי תוכל להשתמש בפרלקסה.
    אם תהיינה לך מספר נקודות קליטה, תוכל להשתמש בטריאנגולציה.

    בעולם הרדיו המצב מעט שונה. אם תשגר פולס אז צורת הפולס תשתנה עם המרחק. סיבה אחת לזה היא המהירות השונה של כל אורך גל בתווך (באוויר במקרה הזה).

  6. הי אלבנצו, שמח שאתה מגיב.

    בוודאי שאני זוכר שהצגתי את השאלה ואיפה, הרי משם העתקתי אותה..

    https://www.hayadan.org.il/researchers-discover-the-most-distant-supermassive-black-hole-07121704/comment-page-13#comment-723626

    אבל לא ראיתי תגובה ישירה שלך לשאלה הממוקדת הזו אחרי שהוצגה באותו שירשור.

    אז בוא נקבל תשובה חד משמעית לשאלה חד משמעית:

    אם פוטון יצא ברגע 0 – זמן מוגדר – מנקודה א׳ לנקודה ב׳ במרחק חמש שניות אור, אז אחרי 3 שניות הוא לא נמצא בהסתברות שווה לפני ואחרי נקודה ב? נעזוב כרגע מה תגלה המדידה.

    אם אפשר תשובה בכן או לא.

    ואם תוכל גם להתייחס ליתר השאלות:

    1. הקשר בין קרינת הרקע לטרנספורמציות לורנץ, הכי טוב נוסחה או קישור.

    2. האם יש דרך תאורטית כלשהי לדעת את מרחקו של גוף הפולט קרינה אלקטרומגנטית אם איננו יודעים את עוצמת הקרינה ואין לנו שום נתונים על סביבת הגוף? האם נוכל לדוגמה לדעת את מרחקו מאיתנו של פנס מהבהב או משדר רדיו שפולט פוסים מחזורים בחלל אם איננו יודעים את עוצמת הפנס או המשדר?

    3. ההגיון מאחורי ניסוי מ-מ. מערכת מנוחה ספציפית לאתר – כמו מערכת הקרינה – סותרת את הנחת ההומגניות מבחינת מערכות מנוחה (למה דווקא זו?).

    תודה.

  7. ישראל,

    דווקא כן ענו לך על השאלה הזו. אני הסברתי לך אותה בעצמי על פני מיליון תגובות. פוטון בעל תנע מוגדר מרוח על פני כל המרחב (הסברתי לך גם שזה לא באמת מצב פיזיקלי אלא אבן בניין מתמטית, אבל לא חשוב כרגע). על פוטון כזה לא ניתן לומר שהוא יצא בזמן t מנקודה A. הוא לא מרוח על פני כל המרחב *רק כשנוח לך*. הוא היה מרוח על פני כל המרחב גם בזמן t. בפרט, הוא לא יצא מנקודה A. אתה לא זוכר את כל הדיון הארוך הזה? שהתגלגל לאלף שאלות על לייזרים והסברים על מה זה חבילת גלים וקירוב סמי-קלאסי של אור?

    דיברנו על כך שבעוד שהפוטון מרוח, המדידה נותנת תמיד תוצאה בודדת והיא תמיד תקיים את המשוואות הקלאסיות שאומרות שהמרחק בין שתי מדידות שהפרש הזמנים ביניהן הוא t יהיה ct. דיברנו על כך שלא ניתן להתעלם ממה שקורה מחוץ למדידות כי זה רוב מכניקת הקוונטים (כמו למשל התאבכות כמו בניסוי שני הסדקים). איפה היית כשכתבתי את כל הדברים האלה? אחרי זה אתה מתעצבן כשאני אומר לך שאתה לא קורא מה אני כותב לך.

  8. אז מה אתה אומר, שאם פוטון יצא ברגע 0 – זמן מוגדר – מנקודה א לנקודה ב במרחק חמש שניות אור, אז אחרי 3 שניות הוא לא נמצא בהסתברות שווה לפני ואחרי נקודה ב?

    ובקשר לקרינת הרקע, השאלה היא איך היא קשורה לטרנספורמציות לורנץ. אני צריך להשתמש בהן כדי לדעת את מהירותי יחסית אליה, ולא יחסית לג׳פ שלך..

  9. ישראל
    אני כל הזמן אומר אותו דבר. אולי אני טועה, אבל אני אומר את מה שאני חושב שקורה: אם אתה יודע מתי הפוטון יצא ולאיזה כיוון, אז אתה לא בעולם הקוונטי.

    בקשר לקרינת הרקע, אגיד שוב את מה שאני מבין. קרינת הרקע מספקת מערכת יחוס. אם אתה נע במהירות מסויימת יחסית לקרינת הרקע אז אתה יכול לדעת את זה. לכן, כל אחד, בכל מהירות יכול חשב את גיל היקום, ויגיע לאותה תוצאה.

    בפרט – כדור הארץ נע יחסית לקרינת הרקע במהירות של כ-400 ק”מ לשניה, לכיוון מזל אריה.

    אבל (!!!) – זה לא אומר שקיימת מערכת יחוס מוחלטת. זה רק אומר שיש לנו מערכת כזו. גם הג’יפ שלי מגדיר מערכת יחוס….

  10. הפוטון לא יראה מאחרי הלייזר ויש סיבה טובה לכך, אבל זה עדיין לא עונה על השאלה המקורית:

    אבל אם הוא מרוח במרחב לפני כן, ועוד בהסתברות שווה, אז יש שתי אפשרוריות:

    1. מציאות הפוטון מעבר לB אינה משהו פיזי – זו רק פונקציית הגל או הפילוסופיה או הזברבירולוגיה, אך לא הפוטון עצמו.

    (קצת מוזר אם נחשוב על כך שריבוע פונקציית הגל היא ההסתברות למצוא את החלקיק בנקודה מסויימת).

    2. החלקיק עצמו נמצא גם מעבר לB.

    במקרה 1, אז אין כל ממשות לפוטון שנע מהר מc, איינשטיין צודק וקוונטים חרטה. אך להבנתי, לא זה המצב.

    במקרה 2, אם הפוטון עצמו נמצא מעבר לB בזמן פחות מct, אז אין מנוס מהמסקנה שהוא נע מהר מc ולמעשה בכל מהירות שהיא״.

    עדיין לא קיבלתי תשובה מאף אחד פה על השאלה המובעת בשורה האחרונה:

    ״במקרה 2, אם הפוטון עצמו נמצא מעבר לB בזמן פחות מct, אז אין מנוס מהמסקנה שהוא נע מהר מc ולמעשה בכל מהירות שהיא״.

    אז אם לטענתך הפוטון לא יראה מאחרי הלייזר, אז הוא אינו מרוח בהסתברות שווה גם מעבר לו, לא?

    ומה ענית בקשר לקרינת הרקע? מה הקשר שלה ללורנץ?

  11. ישראל
    “ידע” הכוונה לפעולת החלקיק על מערכת גדולה.
    אלקטרון בודד נראה בתא בועות, ופוטון בודד נראה בעין. אם יצרתי פוטון ב A והוא נע לכיוון B, הוא תמיד יגיע אליו. בוא נשגר טריליוני פוטונים בכיוון מסויים בעזרת לייזר. אף פוטון כזה לא יראה מאחרי הלייזר.

    בקשר לקרינת הרקע, לא עניתי?

  12. פלוני 123
    לפני 2400 שנה ארטוסטנס היווני מדד כך את היקף כדור הארץ. הוא גם חישב את גודלו של הירח והמרחק אליו, ובשביל הכיף גם את המרחק לשמש ואת גודלו.

    אם הייתי בונה סולם אינטליגנציה , אז בקצה העליון הייתי שם את ארטוסטנס ובקצה התחתון את נעלי הבית שלי. מתן וחבריו הם איפשהו מתחת לסולם…

  13. שלום לכולם, מצאתי הוכחה נפלאה לכדוריות הארץ (אך אין די מקום לכותבה… לא, סתם)

    זה הולך ככה: בימי השיויון (20-21 במרץ וב22-23 לספטמבר), אדם שיימצא על קו המשווה יראה בחצות היום את השמש היישר מעליו בזוית 90 מעלות מהאופק, עכשיו, בואו נסתכל על אדם אחר שנמצא במרחק 5,000 ק”מ מהאדם הראשון (הכיוון לא משנה אבל לשם הפשטות נניח שהאדם השני נמצא גם הוא על קו המשווה רק 5,000 ק”מ מזרחה/מערבה), הוא ייראה את השמש בגובה 45 מעלות מהאופק, מכאן נובע שאם העולם הוא שטוח מרחק השמש מפני הארץ חייב להיות 5,000 ק”מ.

    עכשיו, הבה נקפוץ לאדם שלישי שנמצא אף הוא על קו המשווה אך במרחק כפול מקודמו, 10,000 ק”מ -שהם 90 מעלות אורך בעולם כדורי- המציאות היא שהוא ייראה את השמש כמעט שוקעת, כחצי מעלה בלבד מעל האופק, אבל לפי מי שחושב שהארץ היא שטוחה חישוב פשוט בטריגונומטריה יראה שהשמש אמורה להיראות בגובה 26.5 מעלות מעל האופק !!!

    גם אם מישהו ינסה לטעון שהמרחקים על פני כדור הארץ הם שונים מהידוע לכולנו, זה לא ממש יעזור לו, הנה לדוגמא: אפילו אם מישהו יתרחק מזרחה 20,000 ק”מ מהאדם שרואה את השמש בזניט, עדיין הוא אמור לראות את השמש בגובה 14.03 מעלות מהאופק. 30,000 ק”מ והשמש תראה לו בגובה 9.45 מעלות.

    שימו לב, גם אם מישהו יטען שהשמש היא בצורת מנורה שולחנית / זרקור דהיינו מאירה רק אזור מסויים (וזה לא המצב כפי שכולנו רואים, השמש נראית תמיד ככדור בין אם היא באמצע השמיים ובין בשקיעה) או שהאטמוספירה הסמיכה היא זו שגורמת שלא נראה את השמש בלילה, זה לא יעזור, כי גם אם הטענות האלה היו נכונות, לא היינו אמורים לראות את השמש שוקעת באופק אלא מתכהה לאיטה בערפילי האטמוספירה עד שהיא נמוגה או שהיא הייתה קטנה מצורת כדור עד לפס דק של אור ואז נעלמת (כמו פנס שמכוון ישר לפנים ומסתובב לאט לכיוון התקרה) אבל כל זה היה קורה בגובה האמור, 26.5 מעלות.

    בקיצור, בעולם שטוח, לא משנה מה היה קורה לה, השמש לא הייתה שוקעת לעולם מתחת לאופק!

    נראה לי שזו הוכחה ניצחת שאין לה מענה לכך שהארץ איננה שטוחה אלא כדורית.
    מה אתם אומרים? אשמח לתגובות.

  14. ישראל
    לפי הבנתי – אם אין לך שום ידע על חלקיק מסויים, אז ורק אז, הוא יכול להיות בכל מקום בהסתברות שווה. אם יצרת אותו בזמן מסויים והוא נע לכיוון מסויים – אז אתה יודע עליו כמעט הכל. כנ”ל לגבי אלקטרון וגם פרוטון

  15. להבנתי על פי מכניקת הקוואנטים זה לא רק עניין של מדידה, לאלקטרון פשוט אין תנע או כל תכונה קוואנטית אחרת לפני המדידה.

    לפוטון ירוק במערכת מנוחה מסויימת יש תדר מדוייק בין אם מדדת אותו או לא, ולכן גם תנע מדוייק. זו הסיבה להבנתי שעל פי עקרון אי הוודאות הוא מרוח בהסתברות שווה ביקום לפני המדידה.

    מה עם שאר השאלות? מה עם הסתירה בין פוסטולט 1 וקרינת הרקע? ואיך בכלל קשורה קרינת הרקע לטרנספורמציות לורנץ? היא חייבת להיות קשורה, אחרת מקבלים סתירה מתמטית, אבל איפה הקשר ואיפה הקישור?

    ומה עם מקור הקרינה האלקטרומגנטית? תוכל לחשוב על דרך תאורטית או מעשית כלשהי לדעת את מרחקו למודד ללא ידיעת עוצמתו?

    הנה עוד שאלה שאיני יודע איך לפתור ואיני בטוח בנתונים:

    נאמר שיש לנו 3 חלקיקים שזורים זה עם זה, והם נמצאים ביחד בחדר אך נמדדים בזמנים שונים.

    אם המדידה מראה ששלושתם תמיד באותו מצב קוונטי, האם המדידה של הראשון מבחינת הזמנים היא זו שהוליכה לקריסה והמדידה של השניים האחרים רק גלתה את תוצאות המדידה?

  16. ישראל
    עקרון אי הוודאות. ככל שתדע בדיוק רב יותר את התדר, כך תיפגע הדיוק שלך בידיעת זמן יציאת הפוטון. להבנתי…

  17. ניסים

    אם פוטון יצא מנקודה A בזמן t אז אתה יודע הכל עליו. זה כמו רכבת (לא רכבת ישראל) ולכן יגיע לנקודה B בזמן המתוכנן.
    אם תעמוד מול הפוטון, אתה תראה אותו בעין.

    במה זה שונה מהאלקטרונים בשק”ק?

    האם אתה יודע מה התנע של אלקטרון בשק״ק? כי אתה כן יודע מה התנע של פוטון שיצא מנקודה A במערכת היחוס של הנקודה ועד שיגיע לנקודה B.

  18. ווקי

    אחרי שכתבתי לך את ה״בסדר״, ראיתי איזה משהו בטלוויזיה שגרם לי בכל זאת לענות לך.

    אתה כותב: ״אני לא מסוגל להסתדר עם ההתעקשות שלך למצוא כל מיני פרשנויות מוזרות לדברי, החיפוש שלך אחרי דברים שלא אמרתי בדברי, ההסטה שלך את כל הנאמר לכיוונים שלא היו שם ושאינם קשורים, החוסר מוכנות לשמוע, והאובססיביות הילדותית״.

    כתבת גם:

    ״על אנשים שלא תקועים במקום כבר עשור באותה נקודה בדיוק כי הם חושבים שהשמש זורחת להם מהמקום בו השמש לא זורחת ומסרבים להקשיב לאחרים ולנסות להבין או ללמוד מהם אני עוד מוכן לנסות להשקיע אנרגיה בלעזור להם. אתה אבל לא אחד מהם״.

    ״זו הבעיה הראשונה שלך. יש לי כבר מאות ראיות לכך שזו אמונה מחוסרת בסיס.
    השניה היא הקטיף דובדבנים שאתה עושה לדברים שאנשים אומרים לך, מה שרק מחריף את הבעיה הראשונה.

    עכשיו מכיוון שאתה מבקש כל כך יפה אז אשחרר אותך מהעול ואותיר אותך לדבר רק עם אנשים שלא פוגעים ב או מאתגרים את התפיסה העצמית העקומה והלקויה שלך״.

    והתחלת עם:

    ״אם אי פעם היה או יהיה לך ספק למה אנשים מפסיקים לדבר איתך, תדע שזה בגלל שאתה מתאמץ לא להבין או להבין לא נכון מה שהם אומרים לך(או לפחות גורם לזה להראות ככה)״.

    הצעתי לך מקודם להתמודד עם הטענה שלך שאיני מבין נכון מה אנשים אומרים לי (אפילו אם נתעלם מהטון המחנך שבביטוי ״שאתה מתאמץ לא להבין או להבין לא נכון מה שהם אומרים לך(או לפחות גורם לזה להראות ככה)״.

    סירבת, ואני מאמין שיש לכך סיבות טובות:

    1. אין שום דבר שלא הבנתי לא נכון במה שנאמר לי, בטוח שלא בכתבה זו.

    2. ״אנשים״ לא מפסיקים לדבר איתי. הנושאים היחידים כמעט בהם אני מגיב כמעט הם פיזיקה ובעלי חיים, בעיקר פיזיקה.

    שני המגיבים הקבועים פה בענייני פיזיקה שהם בעלי השכלה על תיכונית הם אלבנצו וניסים אני מאמין. רוב התגובות שלהם בעניינים אילו הן אלי, ובמקרה של אלבנצו שאי אפשר לחשוד בו בחיבה יתרה אלי, *כל* התגובות שלו אלי בשנתיים האחרונות הן יוזמה שלו.

    ואין צורך לומר שיודה שגם הוא מבין קצת בפיזיקה מתדיין איתי רבות, למרות חוסר ההסכמה ביננו.

    אז על אילו אנשים בדיוק אתה מדבר? למי בדיוק יש ידע או הבנה בפיזיקה? לך?

    אבל לא זו הנקודה.

    אני מזמין אותך להראות לי ולפורום פעם אחת ויחידה בעשר השנים האחרונות שבה לא הבנתי נכון מה שאנשים אמרו לי, שעשיתי ״קטיף דובדבנים״ ששיקרתי, שנתקעתי, שחיפשתי דברים שלא אמרת, שהסטתי, שיש לי ראיה עצמית לקויה, או כל מה שתרצה.

    רק בבקשה, בלי סבטקסט שכל אחד יכול לפרשו כרצונו.

    אני בתמורה אם תבקש, אוכל להראות לך עד כמה שאתה טועה בכל ויכוח שהיה ביננו אי פעם.

    גילווי נאות: איני מכיר אותך ואינני יודע אם אתה עולה חדש או שאולי מסיבה כלשהי אינך מבין כמה פלצני מתנשא וקשקשן אתה נשמע כאשר אתה כותב ״אנסה לעזור לך להבין״, ״אני מנסה להבין את צורת החשיבה שלך״, ״בייבי סטפס״, ו״אתה מתעקש לא להבין״. אולי בשפה זרה זה נשמע לגיטימי, בשפה מדוברת בארץ זה נשמע יהיר ומתנשא אפילו עם הידע וההבנה שלך עולות על אילו של בן שיחך, שלא לדבר על המקרה שזה איננו המקרה.

    מוכן לקחת את האתגר? רוצה לחשוף את טעויותי הרבות ואת כל יתר הדברים שאתה אומר עלי? רוצה להתחיל בכתבה זו?

  19. ישראל

    מצטער לא. אני לא הולך להתדיין איתך בשום אופן יותר, נקודה.
    לא מתוך עלבון. לא מתוך כוונה לא לפגוע בך.
    פשוט משום חוסר תכלית. אני לא מסוגל להסתדר עם ההתעקשות שלך למצוא כל מיני פרשנויות מוזרות לדברי, החיפוש שלך אחרי דברים שלא אמרתי בדברי, ההסטה שלך את כל הנאמר לכיוונים שלא היו שם ושאינם קשורים, החוסר מוכנות לשמוע, והאובססיביות הילדותית(לא בכוונה להעליב, באמת). מיציתי כבר מספר פעמים בעבר, ועכשיו פשוט מיציתי סופית.

    תבלה.

  20. עוד הפעם ממתינה.

    אולי ננצל את הזמן כדי לדסקס את השינויים שהוכנסו באתר. האם מישהו פה מוצא יתרון כלשהו בפורמט החדש? עם זה שהתגובות קופצות בסמרטפון? שתגובות אחרונות אינן מופיעות בצד ימין? ששטח התגובה מקוצץ? שהפרטים אינם נשמרים?

    אבי בליזובסקי, האם אתה קורא בכלל את כל התגובות השליליות בקשר לשינויים? האם לא ניתן לעשות משהו בנושא?

    תודה.

  21. ווקי

    הסרטון שלך טוב וענייני. אם תוכל בבקשה להמשיך בתגובות ענייניות כאילו, בלי ״בייבי סטפס״, אני מנסה להבין את צורת החשיבה שלך״ ״אני מנסה לעזור לך להבין״ ״אתה מתעקש לא להבין״ ושאר פלצינין – אז אין לי בעיה להתדיין איתך.

    אבל שים לב: בסרטון מצויין במפורש, לכל אורכו אך בעיקר בדקה 7, שההסתברות למציאת החלקיק אינה זהה בכל נקודה לאורך מסלולו, אלא מרוכזת באיזורים ספציפיים. זה נובע ישירות מעיקרון אי הוודאות (דקה 10).

    זה סותר את מה שנאמר בשאלה שהצגתי לגבי הפוטון:

    ״אתה יכול לפלוט פוטון מנקודה A וכעבור זמן מתאים לקלוט אותו ב-B.

    אבל אם הוא מרוח במרחב לפני כן, ועוד בהסתברות שווה״.

    ניסים

    לגבי הפוטון – ע״ע תגובה לווקי. מריחה במרחב בהסתברות שווה.

    ״איך מודדים את הטמפרטורה״ תאמין או לא, תוכל אפילו עם מדחום בצל, כמו שמודדים לתינוק. אם לא אז עם מד קרינה.

    תקבל סתירה לפוסטולט 1, כי אם תצלם בוידאו את שעון הצזיום ג׳ ושעון הטמפ׳ הצמוד אליו במשך שעה נאמר, אז תקבל וידאו שונה מצילום של שעון א׳ ושעון הטמפ׳ הצמוד אליו, ותוכל אפילו לדעת על פי גרדיאן השינוי את מהירותך בייחס לקרינת הרקע.

    אבל זה לא רלוונטי. מה שרלוונטי הוא שגיל היקום משתנה באותו קצב בכל נקודה ביקום, איך תמדוד את השינוי זה כבר עניין טכני.

    השאלה שלי היא: איך קשורה קרינת הרקע לטרנספורמציות לורנץ? לא שני טוען שהן אינן קשורות, אני רק מבקש הסבר, או עדיף קישור, לקשר בינהן.

  22. ישראל
    איך מודדים את הטמפרטורה? אם תסתכל על קרינת הרקע, אתה תראה שהמדידה תלויה בכיוון. כך למשל, אנחנו יודעים שלכדור הארץ יש מהירות יחסית לקרינת הרקע. לכן, אין כל סתירה.

  23. ישראל
    אם פוטון יצא מנקודה A בזמן t אז אתה יודע הכל עליו. זה כמו רכבת (לא רכבת ישראל) ולכן יגיע לנקודה B בזמן המתוכנן.
    אם תעמוד מול הפוטון, אתה תראה אותו בעין.

    במה זה שונה מהאלקטרונים בשק”ק?

    איפה כן יש מריחה? הידע שלך על כיוון הפוטון אינו מוחלט, ולכן אם תעמוד רחוק, אתה תגלה שהפוטון לא בדיוק איפה שציפית. זו בדיוק התופעה שקוראת בקרן לייזר. ככל שאתה מתרחק, שטח חתך העונה גדל.

  24. ממתינה

    עד שתשתחרר, שאלה שלישית:

    האם יש דרך תאורטית כלשהי לדעת את מרחקו של גוף הפולט קרינה אלקטרומגנטית אם איננו יודעים את עוצמת הקרינה ואין לנו שום נתונים על סביבת הגוף? האם נוכל לדוגמה לדעת את מרחקו מאיתנו של פנס מהבהב או משדר רדיו שפולט פוסים מחזורים בחלל אם איננו יודעים את עוצמת הפנס או המשדר?

    ואיני מתכוון שנשווה את עוצמת האור הנקלטת משתי נקודות במרחק שונה מהפנס. אני מתכוון באמצעות מדידות מנקודה אחת בארץ.

  25. ועוד אחת:

    1. יש לנו שני שעוני צזיום מסונכרנים א׳ וב׳ במרחק של נאמר 10 שנות אור.

    2. בזמן 0 בשעונים א׳ וב׳, שעון צזיום ג׳ חולף ליד א׳ בכיוון לשעון ב׳ כשהזמן בו גם כן 0. גאמה שווה ל10.

    3. שעון ג׳ מגיע לב׳ בזמן של כשנה והזמן בב׳ למעלה מ10 שנים.

    4. הזמן שחלף ביקום, שאותו ניתן למדוד גם עיי מדידת הטמפרטורה, הוא כ10 שנים.

    5. על פי פוסטולט 1 ביחסות כל המערכות האינרציאליות זהות, לכן אם נשים שעוני טמפ׳ ליד שעונים א׳, ב׳ וג׳, השינוי היחסי שלהם ביחס לשעוני הצזיום יהיה שווה בשלושתם.

    6. אך מכיוון שאנחנו מיד מגיעים לסתירה מתמטית אם מקבלים את סעיפים 1-6, ההסבר שקיבלתי עד כה הוא שפוסטולט 1 לא כולל את קרינת הרקע הקוסמית, ולכן אם שעונים א׳ וב׳ במנוחה יחסית לקרינה, הם יטקטקו באותו הקצב כמו שעוני הטמפ׳ הצמודים להם, ושעון ג׳ יטקטק לאט יותר משעון הטמפ׳ הצמוד לו. אם שעון ג׳ הוא הנייח יחסית לקרינה, אז שעונים א׳ וב׳ יטקטקו לאט יותר משעוני הטמפ׳ הצמודים להם.

    7. המסקנה היא ששעוני הטמפ׳ תמיד מטקטקים מהר יותר משעוני צזיום הצמודים להם, בקורלציה עם מהירותם יחסית לקרינת הרקע.

    והשאלה: מה דין שמיטה להר סיני? מה הקשר לקצב טקטוק שעונים לקרינת הרקע הקוסמית?

  26. בינתיים, שאלה לפורום שעדיין לא נענתה:

    ״אתה יכול לפלוט פוטון מנקודה A וכעבור זמן מתאים לקלוט אותו ב-B.

    אבל אם הוא מרוח במרחב לפני כן, ועוד בהסתברות שווה, אז יש שתי אפשרוריות:

    1. מציאות הפוטון מעבר לB אינה משהו פיזי – זו רק פונקציית הגל או הפילוסופיה או הזברבירולוגיה, אך לא הפוטון עצמו.

    (קצת מוזר אם נחשוב על כך שריבוע פונקציית הגל היא ההסתברות למצוא את החלקיק בנקודה מסויימת).

    2. החלקיק עצמו נמצא גם מעבר לB.

    במקרה 1, אז אין כל ממשות לפוטון שנע מהר מc, איינשטיין צודק וקוונטים חרטה. אך להבנתי, לא זה המצב.

    במקרה 2, אם הפוטון עצמו נמצא מעבר לB בזמן פחות מct, אז אין מנוס מהמסקנה שהוא נע מהר מc ולמעשה בכל מהירות שהיא״.

    עדיין לא קיבלתי תשובה מאף אחד פה על השאלה המובעת בשורה האחרונה:

    ״במקרה 2, אם הפוטון עצמו נמצא מעבר לB בזמן פחות מct, אז אין מנוס מהמסקנה שהוא נע מהר מc ולמעשה בכל מהירות שהיא״.

  27. למה שאעלב? תודה באמת על התגובה המושקעת והמלומדה. כל עוד אינך מקלל אז אין לי בעיה.

    את שדה ההיגס הבאתי רק כדוגמה למערכת הומוגנית באמת. השאלה שלי הייתה רק לגבי ההגיון מאחורי ניסוי מ-מ ב1887, ואת הפיזיקה של אותה תקופה אני מאמין שאני מכיר ומבין.

    אז אשאל עוד הפעם: נאמר שניסוי מ-מ היה מוצא את מה שביקש, דהיינו את מערכת המנוחה של האתר. נאמר אפילו שהיא זהה למערכת המנוחה של קרינת הרקע.

    אתה מסכים שיש עוד אינסוף מערכות מנוחה מלבד אותה מערכת? אז אם ליקום יש העדפה של מערכת מנוחה זו, אז הוא אינו הומוגני מבחינת מערכות מנוחה, לא?

  28. ישראל,

    שמע. לקח לי הרבה זמן לכתוב לך את התגובה. כתבתי אחת ארוכה, מחקתי, כתבתי אחת אחרת, מחקתי שוב. בסוף החלטתי על תגובה יחסית קצרה כי האמת היא שאני לא מאמין שיש טעם בדיון הזה. כנראה שאתה תקח את זה כעלבון, אבל חשוב לי להבהיר שאני לא אומר את זה כעלבון וממש לא מתכוון להעליב או להשפיל אותך, אבל אני חייב לומר זאת – ניכר מתגובתך שאתה פשוט לא מבין על מה אתה מדבר. אני מצטער, באמת לא אומר זאת כדי להעליב, אבל כדי לדבר על פיזיקה חייבים שפה משותפת ואתה פשוט מדבר שפה שאתה המצאת לעצמך והיא שגויה לחלוטין ואינה חופפת פיזיקה כפי שהיא במציאות.

    1. כל החלק על היות אוקיינוס אחיד לא הומוגני ואיזוטרופי הוא בלבול אחד גדול שנובע מזה שאתה כנראה לא מבין מה זה הומוגני ואיזוטרופי. תעשה את המבחן הכי פשוט שיש – קח מרחב תלת מימדי אינסופי המלאה בנוזל בצפיפות אחידה. בדוק את ההגדרות המתמטיות של הומוגניות, איזוטרופיות וקיום מערכת מנוחה. אל תשפוך פסקאות על גבי פסקאות של טקסט ותשחק אותה עו”ד. בדוק האם הוא הומוגני (אם יש סימטריה להזזה של מערכת ניסוי לכל נקודה בנוזל), האם הוא איזוטרופי (האם יש סימטריה מושלמת לסיבוב שאינה משנה אף שדה חיצוני) והאם קיימת טרנספורמציית לורנץ שבה ה-3-תנע של כלל חלקיקי הנוזל נסכם ל-0. זה הכל. כל אחת מהבדיקות האלה לוקחת שורה אחת בדיוק והתשובה לכולן היא “כן”. כנ”ל לגבי הדוגמא השניה שנתתי לך. נוזל שיש בו זרם קבוע לא מקיים זאת (מפר איזוטרופיות) ושום טענה על כך שבוואקום אין הבדל בין זרם קבוע לסטטיקה (טענה שהיא ביסודה נכונה אך מאפיינת את הוואקום ולא נוזל בצפיפות אחידה) לא תשנה זאת.

    2. כל מה שכתבת על היגס הוא לא בכיוון בכלל. שוב אומר שאני לא מנסה להעליב ואציין שתורת שדות היא חומר די מתקדם שנלמד לכל המוקדם בתואר שני, ורק על ידי חלק מתלמידי הפיזיקה. אז אין מה להתבייש בכך שאתה לא מבין אותה. אבל אתה חייב להבין שאתה לא מבין אותה, ואפילו בכלל. בתורת השדות כל החלקיקים מתוארים כשדות כתוצאה מאינסוף דרגות חופש שמוסיפים לחלקיק (הוא אינו ממוקם במרחב). “שדה היגס” אינו יותר שדה מאשר האלקטרון. אתה מתאר אותו בתור איזה שדה כוח מתכנית אנימציה לילדים שמתנגד לתנועה דרכו, אבל הוא שדה בדיוק כמו שאלקרון הוא שדה. אין דבר כזה “לעבור דרך השדה”, השדה הוא פונ’ מתמטית על המרחב שמגדירה את דרגות החופש החלקיקיות. בפרט כל מוד תנועה שלו הוא חלקיק, בדיוק כמו שאלקטרון הוא חלקיק. הוא לא איזה כישוף שיוצר אינרציה. הוא סתם עוד חלקיק שמסביר למה לחלקיקים במודל הסטנדרטי יש מסה למרות שהסימטריה של המודל מרמזת שלא יכולה להיות להם מסה. במילים אחרות, אינרציה קיימת גם במערכת שאין בה חלקיקי היגס בכלל. היגס חשוב למודל הסטנדרטי בגלל הסימטריות הספציפיות שמצויות בו והוא לא איזה כישוף שמאט אותנו כשאנחנו זזים. אתה אולי זרקת את המונח “שבירת סימטריה” בתור באזז-וורד אבל רק הבהרת שלמעשה אין לך שום הבנה של מהו מכניזם היגס ומה התפקיד שלו בהענקת מסה לחלקיקי המודל הסטנדרטי. ציירת לעצמך תמונה בראש שבה היגס הוא שדה כוח מיוחד הדומה לאתר ואתה עובד לפי ציור זה שבאמת לא יכול להיות יותר רחוק מהמציאות. היגס הוא חלקיק, כמו אלקטרון, בעל תכונות סימטריה ספציפיות ואינטרקציה מאוד מעניינת עם שאר החלקיקים. הוא “נדבק” אליהם ומעניק להם מסה למרות שסימטריות המודל הסנטדרטי מכתיבות שלא צריכה להיות להם מסה משל עצמם. שדה היגס הוא לא “מערכת”. הוא חלקיק, כמו אלקטרון, כמו פוטון, כמו פאיון או גלואון. ואמנם לשדה אין מערכת מנוחה כפי שאמרת, אבל זה רק כי אם היית מקפיד על ההגדרות של “מערכת מנוחה” היית יודע שזה משהו שמוגדר רק עבור אופן תנודה בעל תנע מוגדר (או קבוצה של כאלה), ולא עבור אופרטור שהוא סופרפוזיציה של אופנים. ואם מסתכלים על אופן של היגס (במילים אחרות, חלקיק בודד) אז ועוד איך יש לו מערכת מנוחה ואתה מוזמן לפתוח בעצמך מאמרים על פיזורים ולראות. כל החלק על היגס לא קשור בשום צורה לפיזיקה. הוא סיפור שסיפרת לעצמך כדי לנסות להבין את היגס, ולצערי הרב טעית ובענק.

    מצטער אם זה נשמע כאילו אני צוחק עליך או מנסה להשפיל אותך. באמת שלא. אבל אתה חייב להבין שמה שאתה עושה זה לא פיזיקה. זה להמציא לעצמך סיפורים. הסיבה שאני אומר את הדברים האלה היא כי אני בטוח שלפחות חלק מהדברים אתה אכן מסוגל לעשות. גם אם לא תלמד עכשיו תורת שדות, אין שום דבר שמונע ממך להזכר רגע במתמטיקה של יחסות פרטית ולהוכיח לעצמך בשתיים-שלוש שורות שיש מליון דוגמאות לתווך הומוגני, איזוטרופי ובעל מערכת מנוחה. זה ממש לא צריך להיות גדול עליך.

  29. זייגזונט סבא, אבל זה מה שניסוי מ-מ ניסה למצוא, וברור שאם היא הייתה נמצאת אז יש מערכת מנוחה מועדפת ליקום, ואז איפה ההומוגניות?

    קפיש?

  30. ישראל
    “אם ניסוי מ-מ היה מוצא את מערכת המנוחה של האתר והיא זו של מאדים, אז יש מערכת מועדפת ביקום, אז איפה ההומגניות?”

    Az di bobe volt gehat beytsim volt zi geven mayn zeyde

  31. אוקי, אז נעשה אותו הומוגני גם במבחן מהירות.

    ברור שאם לאוקיינוס אחד יש את מערכת המנוחה של הארץ, לשני של מאדים ולשלישי של אנדרומדה, אז יש לנו 3 מערכות מנוחה שונות.

    ואם ניסוי מ-מ היה מוצא את מערכת המנוחה של האתר והיא זו של מאדים, אז יש מערכת מועדפת ביקום, אז איפה ההומגניות?

    וזאת בניגוד לאוקיינוס ההיגס שלו אכן אין כל מערכת מנוחה מועדפת.

  32. ישראל
    הומוגני אומר שתכונה מסויימת לא משנה את ערכה כתלות במיקום. איזוטופי אומר שאין תלות בכיוון.
    על פניו זה נראה כאילו שההגדרות חופפות. אבל לא – קח שדה מגנטי אחיד (כמו ב-MRI). בכל מקום (בתוך המכשיר) ערך השדה הוא קבוע, אבל כמובן שבכל כיוון ערך השדה שונה.

    האתר יכול להיות הומוגני ואיזוטרופי, ובמקביל יכול להיות לו מערכת יחוס.

  33. שדה ההיגס הובא כדוגמה לאוקיינוס שהוא אכן הומוגני ואיזוטרופי, וזאת בניגוד לאוקיינוס השקט שמערכת המנוחה שלו היא הארץ, האטמוספרה של צדק שמערכת המנוחה שלה היא צדק, והאוויר בתחנת החלל שמערכת המנוחה שלה היא התחנה.

    אם לאתר יש מערכת המנוחה שאותה ניסה ניסוי מ-מ למצוא, אז הוא אינו הומוגני ואיזוטרופי כי עובדה, יש לו מערכת מנוחה – נאמר מערכת המנוחה של קרינת הרקע, ולכן הוא אינו הומוגני ואיזוטרופי.

    מאידך, מודל מקסוול בנוי על האתר שתכונותיו מתוארות בפרוטרוט, וזאת בניגוד לאיינשטיין שאינו מדבר על התכונות הפיזיקליות של התווך והתאוריה שלו בנויה על פוסטולטים.

    לכן אם ״נפתח״ את מערכת האתר של מקסוול למערכת אינסופית, נקבל את התכונות הידועות לנו של גלים אלקטרומגנטים שמהירותם זהה לכל מודד, אינרציה, גרביטציית לסאז׳ ללא חיכוך, והיתכנות אי לוקליות, דהיינו השפעה מרחוק ללא העברת אינפורמציה, כמו שבאוויר מהירות המולקולות יכולה להיות גבוהה בהרבה ממהירות הקול אך אינפורמציה עוברת רק במהירות הקול.

  34. ישראל
    איינשטיין דיבר על תכונות של המרחב עצמו. כלומר, הוא טען שהמרחב עצמו אינו הומוגני ואיזוטרופי. זה בניגוד למה שטען ניוטון.

    המנגנון של היגס שייך למכנקיה הקוונטית.

  35. אנסה עוד הפעם. ילך, ילך. לא, לא.

    אם אתה נמצא באוקיינוס אינסופי הומוגני ואיזוטרופי, ויש כיוון מועדף שבו עליך לשחות נגד האוקיינוס דהיינו האוקיינוס מפעיל עליך כוח, אז מערכת האוקיינוס אינה אחידה. עובדה, כאשר אתה נמצא במנוחה יחסית לאוקיינוס לא פועל עליך כוח, וכאשר אתה שוחה במהירות קבועה כן פועל עליך כוח.

    אך מה ההבדל בין מהירות קבועה למנוחה? האם בחלל יש הבדל בין השתיים? שלילי.

    לכן בדוגמה שהבאת המערכת אינה הומוגנית ואיזוטרופית. עובדה, במצב אחד אתה נמצא במהירות קבועה (מנוחה יחסית לאוקיינוס) ולא פועל עליך כוח, ובמצב אחר אתה נמצא במהירות קבועה יחסית לאוקיינוס וכן פועל עליך כוח.

    אז איפה ההומגניות והאיזוטרופיות? במה שונה מצב א׳ ממצב ב׳?

    חשוב על אוקיינוס ההיגס (בכלל, כיצד ניתן לדבר על שדה ההיגס הממלא את היקום ולהמשיך להגיד שהיקום ריק כמו הטענה שהייתה אחרי 1905).

    שדה ההיגס מתנגד לתאוצת כל מסה דרכו (שבירת סימטריה עלק), אך בניגוד לאוקיינוס, אין לו מערכת מנוחה. באוקיינוס רגיל אם אינך נמצא במנוחה יחסית לאוקיינוס, האוקיינוס יתנגד לכל תנועה דרכו, מואצת או אינרציאלית, ולכן ספינה ללא כוח מניע תאט ותיעצר. שדה ההיגס מתנגד אך ורק למסות בתאוצה ולא למסות בתנועה אינרציאלית. לכן שדה ההיגס הוא מערכת הומוגנית ואיזוטרופית, וספינה תיתקל בכוח הפועל עליה רק בתאוצה, אך אחרי שתפסיק להאיץ ותהייה במהירות קבועה לא יפעל עליה כוח יותר והיא תשאר באותה מהירות קבועה, וזאת בניגוד לאוקיינוס שיפעיל עליה כוח גם כאשר היא במהירות קבועה שאינה מערכת המנוחה של האוקיינוס.

    לכן אם אוקיינוס האתר הוא אינסופי הומוגני ואיזוטרופי, אז אין לו מערכת מנוחה מסויימת, כמו שלאוקיינוס ההיגס אין מערכת מנוחה מסויימת, ואין שום מהירות מסויימת שאותה ניתן למדוד ביחס לאתר, וזו הסיבה לכשלון ניסוי מ-מ.

    ולזאת התכוון איינשטיין (לדעתי) כשאמר שאי אפשר ללא האתר רק שאין לייחס לו מערכת מנוחה מסויימת (קישור אם יש ביקוש).

  36. שתי הדוגמאות שנתתי הן אינסופיות. בדוגמא של האוקיינוס לא כתבתי במפורש אבל התכוונתי כמובן לאוקיינוס אינסופי (אם הוא לא אינסופי הוא גם לא הומוגני, אפשר לשחות כלפי מעלה עד שמגיעים פתאום לאוויר). בדוגמא השניה כתבתי במפורש שהיא אינסופית וגם הסברתי איך מקבלים אותה, תמוה שפשוט החלטת להתעלם מזה.

    שמע, לא יודע איך להסביר את זה באופן יותר פשוט. לא רוצה להעליב אבל יש לי הרגשה חזקה שאתה חושב על המונח “מערכת מנוחה” באופן אינטואיטיבי ולא ממש מפנים מה זה אומר מבחינה פיזיקלית. מערכת המנוחה של אוקיינוס אינסופי היא לא “הארץ”, היא לא שום אובייקט. היא פשוט בחירה של מערכת שבה לאוקיינוס אין תנע תלת-מימדי, זה הכל. אין לזה שום קשר לארץ או לאי כזה או אחר, אין לזה שום קשר להיות האוקיינוס אינסופי. תכתוב פונ’ צפיפות של חלקיקי מים, שהיא הומוגנית ואיזוטרופית, ותבדוק אם אפשר לעשות טרנספורמציית לורנץ למערכת בה למים אין תנע קווי. ניתן לעשות זאת בקלות בשתי הדוגמאות שנתתי, שתיהן הומוגניות איזוטרופיות ואינסופיות. וכמו שאמרתי, דווקא במערכת לא הומוגנית זה יכול ליצור בעיה כי לכל חלק במערכת תהיה מערכת מנוחה לוקאלית אחרת.

  37. אלבנצו

    ״1. מצחיק, את התשובה לשאלה הראשונה נתתי לך לפני כמה שבועות ואתה נלחמת איתי עליה בשן ובעין. הסברתי לך שתורת היחסות הפרטית כפי שניסח אותה איינשטיין ב-1905 היא הרחבה למכניקה קלאסית ולכן באופן מפורש סותרת את מכניקת הקוונטים ולמעשה מהווה רק קירוב מסדר ראשון שלה (הקירוב הקלאסי של מכניקת הקוונטים)״.

    אני נלחמתי על הגנת תורת היחסות הפרטית? אני? אפשר קישור?

    כנראה לא הובנתי.

    2. ״אנשים יותר חכמים ממקסוול עשו טעויות יותר חמורות ממנו״.

    איפה הטעות של מקסוול? קראת בכלל את המודל שלו?

    ״3. אין כאן בעיה. דווקא בגלל שהיקום הומוגני ואיזוטרופי אפשר לחשוב שתהיה לו מערכת מנוחה. לכל מערכת מאסיבית של חלקיקים ניתן למצוא מערכת מנוחה לוקאלית. אם האתר לא היה הומוגני, אפשר היה למצוא מערכת מנוחה לחלקיקי האתר באזור אחד של היקום והיא הייתה שונה לגמרי ממערכת המנוחה באזור אחר. מערכת מנוחה אבסולוטית תתאפשר רק ביקום הומוגני. זה גם לא קשה לדמיין דוגמאות כאלה. למשל, אוקיינוס. קח אוקיינוס אינסופי בו המים מפוזרים באופן הומוגני ואיזוטרופי וקיימת מערכת מנוחה בה אין גלים בים״.

    שכחת להוסיף את התנאי הראשון והחשוב מכולם: אינסופי. זו לפחות הייתה האמונה ב1887 כאשר נערך ניסוי מ-מ.

    האם האוקיינוס היא מערכת אינסופית? תוכל לתת איזו דוגמה של מערכת אינסופית בכלל, ומערכת אינסופית בפרט שיש לה מערכת מנוחה?

    לאוקיינוס שהזכרת יש מערכת מנוחה, הארץ. לאוויר במטוס המטוס.

    אבל איזו מערכת מנוחה יש לאוקיינוס האתר האינסופי? ואם היא קיימת, מדוע דווקא זו ולא אחרת?

    אבי, במידה ואתה קורא, השינויים שערכת באתר אינם מוצלחים בלשון המעטה, והתקלות רק הולכות ומתגברות.

  38. ישראל,

    1. מצחיק, את התשובה לשאלה הראשונה נתתי לך לפני כמה שבועות ואתה נלחמת איתי עליה בשן ובעין. הסברתי לך שתורת היחסות הפרטית כפי שניסח אותה איינשטיין ב-1905 היא הרחבה למכניקה קלאסית ולכן באופן מפורש סותרת את מכניקת הקוונטים ולמעשה מהווה רק קירוב מסדר ראשון שלה (הקירוב הקלאסי של מכניקת הקוונטים). אחרי 1905 היו עוד הרבה עבודות על תורת היחסות, חלקן של איינשטיין אבל רובן לא, שזיקקו את הרעיונות של יחסות פרטית למבנה המתמטי הפשוט ביותר שלה, ואז ישמו אותו הדבר על מכניקת הקוונטים. מה שהתקבל זו תורת השדות הקוונטים, שהיא גם קוונטית וגם יחסותית. אם אתה מסתכל על יחסות פרטית כפי שנכתבה ב-1905 אז כן, היא בהחלט רק קירוב. אבל תהרוג אותי אם אני מבין למה למישהו לחקור את העולם סביבו אבל *רק* עד 1905, ולהתעלם מכמות הידע שנצברה אח”כ, שאני מניח שלא יפתיע אותך לגלות שהיא גדולה פי כמה מכל הידע שנצבר בין ניוטון לאיינשטיין (ממספר סיבות, החל מטכנולוגיה שמאפשרת לעשות הרבה יותר ניסויים הרבה יותר מורכבים, טכנולוגיה שמאפשרת לעבד מידע באופן הרבה יותר טוב, גדילה מאסיבית בקהילה האקדמית ביחס לגודל האוכלוסיה בעולם, וכו’). אני לא אוהב לנבור בתגובות ישנות, אבל אני יודע שאתה טוב בזה ומצליח בד”כ למצוא ציטוטים. אני מבטיח לך שאם תסתכל בהתכתבות האחרונה שלנו תראה שהסברתי לך שאת תורת היחסות הפרטית כפי שאיינשטיין ניסח אותה ב-1905 אפשר לקבל כקירוב קלאסי של תורת השדות הקוונטים (בפשטות, ניתן לחשוב על קירוב זה כמו לקיחת הגבול שבו קבוע פלאנק הוא 0).

    2. שם, לא שם, לא משנה בכלל. מצטער, כל הקטע שכתבת באמצע על זה שמקסוול הוא לא איזה טמבל לא רלוונטי בשום צורה לדיון. אנשים יותר חכמים ממקסוול עשו טעויות יותר חמורות ממנו. השם שלו לא קונה לו אף נקודת זכות ואינו מהווה סיבה להבדיל בין הדוגמאות שנתתי לך (שבהן באופן מפורש תיאוריה שגויה נותנת תוצאה נכונה) לבין הטענה שלך – שהמקרה של מקסוול מיוחד. את שימור התוצאה הנכונה אפשר להסביר בפשטות על ידי קיום סימטריית כיול U(1) גם במודל של מקסוול, אף על פי שהוא שגוי. דרך אגב, הדוגמאות שנתתי היו כבידה ניוטונית (ניוטון לא פחות בשום צורה ממקסוול, אז אם התיאוריה שלו נותנת תוצאות נכונות למרות שהיא שגויה, למה לא מקסוול?), פיזורים קוונטים (למשל, עבודותו של בורן שגם הוא אחד הפיזיקאים הגדולים של המאה ה-20 וזוכה פרס נובל). שתי הדוגמאות הנוספות אינן ממש שגיאות שמישהו עשה אלא תוצאות נכונות שמתקבלות מהצבה במודל שאינו מתאים להם, אז אי אפשר ממש לייחס אותם לאדם מסוים.

    3. אין כאן בעיה. דווקא בגלל שהיקום הומוגני ואיזוטרופי אפשר לחשוב שתהיה לו מערכת מנוחה. לכל מערכת מאסיבית של חלקיקים ניתן למצוא מערכת מנוחה לוקאלית. אם האתר לא היה הומוגני, אפשר היה למצוא מערכת מנוחה לחלקיקי האתר באזור אחד של היקום והיא הייתה שונה לגמרי ממערכת המנוחה באזור אחר. מערכת מנוחה אבסולוטית תתאפשר רק ביקום הומוגני. זה גם לא קשה לדמיין דוגמאות כאלה. למשל, אוקיינוס. קח אוקיינוס אינסופי בו המים מפוזרים באופן הומוגני ואיזוטרופי וקיימת מערכת מנוחה בה אין גלים בים. דוגמא נוספת (קצת יותר מורכבת) היא לקחת קליפה כדורית שהיקום נמצא בתוכה. הקליפה זורקת אתר באופן הומוגני ואיזוטרופי לתוך היקום. בגבול שבו הקליפה נמצאת במרחק אינסופי מהיקום (כלומר שרדיוס הקליפה הוא אינסוף) תקבל התפלגות הומוגנית ואיזוטרופית של אתר (בכל נקודה ביקום תגיע אליך צפיפות זהה של חלקיקי אתר מכל הכיוונים וכמובן שאפשר יהיה למצוא להם מערכת מנוחה פשוט על ידי הטלת תנאי אלגברי.

  39. ניסים

    ״הפרמיאביליות המגנטית ניגזרת ממהירות האור והמקדם הדיאלקטרי. תעשה 1 חלקי מהירות האור בריבוע חלקי המקדם האלקטרי …. ותראה מה תקבל״.

    הפרמיאביליות המגנטית והמקדם הדיאלקטרי היו ידועים עוד לפני מקסוול, וכמותן מהירות האור. אך אף אחד לפניו לא קישר את שלושת התופעות לתופעה אחת, וזאת באמצעות מודל האתר.

  40. “אך יש משהו שמייחד את משוואות מקסוול, לדוגמה שבניגוד לחוקי ניוטון שעודכנו עיי איינשטיין כקירובים בלבד, משוואות מקסוול לא רק שלא השתנו אלא הן המניע העיקרי לכתיבת תורת היחסות הפרטית…”.

    הדברים במידה ושמת לב, נאמרו כתגובה לG שכתב:

    ״אם משוואות מקסוול נכונות, ויש אתר, אז מדוע הן לא מצליחות לחשב את קרינת גוף שחור? אפילו את צבע האור של נורה פשוטה הן לא יודעות לחזות. למה?״

    ניתן אולי להבין מדבריו שהוא טוען שמשוואות מקסוול אינן נכונות, ואת זאת באתי לתקן. במסגרת הידע שהיה לאיינשטיין ב1905, חוקי ניוטון הם קירובים בלבד אך לא משוואות מקסוול, הן מושלמות כפי שהן. תורת היחסות הפרטית, שעליה אני טוען תמיד שאינה מושלמת, בנויה על משוואות מקסוול שנשארו בה ללא שינוי.

    אז כפי שאני רואה זאת, קיימות שתי אפשרויות:

    משוואות מקסוול מושלמות, וכמותן היחסות הפרטית.

    הן קירוב בלבד, וכמותן היחסות הפרטית.

    ואם האפשרות השניה היא הנכונה – אז על מה אנחנו רבים פה כבר 3 שנים?

    ״אתה טוען שהמצב פה שונה מהדוגמאות השונות שנתתי לתיאוריות שגויות שמניבות תוצאות נכונות, ואני מנסה להבין על סמך אתה אומר זאת״.

    כדי להבין, צריך להבין את מודל מקסוול.

    נתחיל מכך שמקסוול אינו איזה מדען חובב עלום שם, הוא מקסוול, מהטרילוגיה ניוטון מקסוול איינשטיין. והתאוריה שלו אינה איזו תאוריה איזוטרית חסרת בסיס שנועדה להסביר תופעות ידועות, המודל שלו הוא יצירת מופת שהמסקנה הנובעת ממנה, שגלים אלקטרומגנטים מתקדמים במהירות האור, לא הייתה ידועה מקודם, וזאת בניגוד לכל שאר המודלים האחרים שציינת שמסבירים כביכול תופעות קיימות אך אינם צופים תופעות חדשות משמעותיות שהתגלו כנכונות.

    כמוהו כמודל איינשטיין שחוץ מהסבר תופעות קיימות, חזה גם תופעות שלא היו ידועות כגון שקילות המסה והאנרגיה, התארכות הזמנים, יקום מתפשט ועידוש.

    אך מקסוול עושה יותר מזאת. בניגוד לאיינשטיין שקובע את פוסטולט 2 אך אינו מסביר מדוע (בגלל זה הוא פוסטולט – אקסיומה – ולא משפט), מקסוול מסביר בפירוט כיצד הוא מגיע למסקנות שלו.

    לא אכנס לפירוט אלא אם יש ביקוש, אך הדרך בה הוא עושה זאת היא כה מרהיבה ולוגית, וזאת בניגוד לאפיצקלים של תלמי (למה אפיצקלים?) או מודלים אחרים, שכמו במקרה של ההסבר של בונד על גולדפינגר הרמאי, קשה לא לקבל אותו.

    אך מקסוול לפי דעתי עשה טעות שאותה עשו כולם בזמנו.

    כתבתי כאן בעבר שעבורי שמות כמו מקסוול, לורנץ או ניוטון, הם כמו הרבי מלובביץ עבור אדם מאמין בהבדל אחד: המאמין מאמין שיש מישהו מעליהם, בשבילי הם הדבר האמיתי.

    ולכן אני חושב שכנראה יש לי טעות במה שאני אומר להגיד עכשיו, אך איני רואה איפה.

    כי אם יש אתר, דבר שכולם האמינו בו בזמנו, אז מדוע שתהיה לו מערכת מנוחה מסויימת? אם היקום הוא אינסופי, הומוגני ואיזוטרופי, דבר שגם בו כולם האמינו ב1887, אז האם מערכת מנוחה מסויימת לאתר אינה סותרת את הנחת
    ההומגניות? אם ניסוי מייקלסון – מורלי היה ״מצליח״, והם היו מוצאים את מערכת המנוחה של האתר – אז מדוע דווקא זו? איפה ההומגניות והאיזוטרופיות?

    והאם יתכן שלורנץ, מאך, קלווין, מייקלסון ומורלי פספסו סתירה כה בולטת?

    מכאן נובע כל השאר.

  41. “אך יש משהו שמייחד את משוואות מקסוול, לדוגמה שבניגוד לחוקי ניוטון שעודכנו עיי איינשטיין כקירובים בלבד, משוואות מקסוול לא רק שלא השתנו אלא הן המניע העיקרי לכתיבת תורת היחסות הפרטית…”.

    כשקראתי את המשפט הזה נראה לי נכון לציין שזה שמאמר ספציפי של איינשטיין לא “עדכן” כלשונך את משוואות מקסוול, זה לא אומר שהן אינן קירובים. אם מסתכלים על תיאוריה יותר כללית, יותר מתקדמת, יותר מדויקת ויותר נכונה מאשר יחסות פרטית (אלקטרודינמיקה קוונטית), רואים שמשוואות אלה הן אכן קירוב, ולא סתם שהן קירוב – אלא הן בדיוק אותו קירוב כמו החוק השני של ניוטון (שטרחת לציין שמעמדו שונה מאשר של משוואות מקסוול).

    קראתי מה אמרת. והבנתי. אתה טוען שהמצב פה שונה מהדוגמאות השונות שנתתי לתיאוריות שגויות שמניבות תוצאות נכונות, ואני מנסה להבין על סמך אתה אומר זאת. בינתיים הדבר היחיד שהבנתי ממך שמעיד על משהו מיוחד זו הטענה שלך שמשוואות מקסוול לא מתקבלות כקירוב של משוואות אחרות של תורה מתקדמת יותר, וזה פשוט לא נכון. אולי התכוונת רק באופן ספציפי שהן לא מתקבלות כקירוב של משוואה שמופיעה במאמר הספציפי של איינשטיין מ-1905 וזה כן נכון, אבל חסר משמעות בעליל. זה כמו להגיד שמישהו שהורשע בגניבה היום בבוקר שונה משאר הגנבים כי בניגוד אליהם, הוא לא הורשע בגניבה (עד היום בבוקר)! בעיני זה מצחיק מאוד, ולכן הנחתי שאתה לא מתכוון לטעון שרק במאמר הספציפי הזה המשוואות אינן קירוב, אלא באופן כללי. וזה פשוט לא נכון, אתה מוזמן לפתוח ספר על QED ולראות בעצמך.

    אין לי שום בעיה. ואין לי צורך “להוכיח תמיד לכולם שרק אני יודע פיזיקה וכל היתר מטומטמים”. אבל פיזיקה זו העבודה שלי וכשמישהו כותב משהו לא נכון בפורום פתוח, נראה לי חשוב להעיר על כך – בין אם בשבילו ובין אם בשביל אנשים אחרים שקוראים את תגובותיו. בנוגע לזה שאני “לא יכול לכתוב שני משפטים רצופים בלי לקלל או להעליב”, אתה סתם מנסה להכפיש את האופי שלי בשביל לא להתמודד עם הטענות. בתגובה הקודמת לא הייתה שום קללה ושום עלבון. לפני זה היו עלבונות (לא נראה לי שקללות), אבל מה לעשות – כשבנאדם שואל “האם יתכן שתוצאה נכונה תתקבל מתיאוריה שגויה?” ואז אתה עונה לו שכן, זה אפשרי, ואף מסביר איך זה יכול לקרות ונותן לו שלל דוגמאות ואז הוא מגיב ב”תסתכל על התיאוריה ותגיד לי אם זה אפשרי”, או במילים אחרות “אני יודע שהרגע כתבת לי תגובה מפורטת על כך שזה אפשרי ואף הרחבת על הסיבות והדוגמאות, אבל האם זה אפשרי?”, אז כן – אני מרגיש צורך חזק לבקש ממנו להתרכז ולוודא שהוא קורא מה שכותבים לו.

    אבל אני לא מתכוון לריב איתך. אתה כותב דברים, זכותך. אני כותב דברים, זכותי. רוצה להתייחס? תתייחס. לא רוצה, לא צריך. כאמור, אני לא רוצה לריב על פיזיקה ובטח ובטח שאני לא רוצה לריב על איך אתה מרגיש. לי עדיין אין מושג למה דווקא במקרה הזה העובדה שמודל שגוי נותן פרידקציה טובה היא פלא ובשאר מליון המודלים שזה קורה בהם זה לא. כפי שכתבתי בתגובה קודמת, תוצאה נכונה זו נובעת באופן ישיר מקשר בין חשמל ומגנטיות שנובע מסימטריית כיול כלשהי (U)1( לצורך העניין), והוא ימצא בכל מודל של אלקטרומגנטיות שיקיים סימטריה זו, לא משנה כמה הוא שגוי.

  42. ישראל
    הפרמיאביליות המגנטית ניגזרת ממהירות האור והמקדם הדיאלקטרי. תעשה 1 חלקי מהירות האור בריבוע חלקי המקדם האלקטרי …. ותראה מה תקבל.

    גם החוק השני של ניוטון הוא מדוייק, ולא קירוב. אז מה?

  43. אל בנצו

    איני יודע אם קראת, או הבנת, את שאני אומר בקשר למשוואות מקסוול. אז הנה ציטוט:

    ״האם הן מושלמות? במצבים מסויימים כן. ניתן לדוגמה לחשב באמצעותן את מהירות האור בלי לצפות בליקויים כמו רמר או לצפות בגלגל מסתובב כמו פיזו, אלא עיי מדידת קבועי החשמל והמגנטיות במעבדה״.

    האם משפט זה לא אומר שיש מצבים בהם משוואות מקסוול אינן מושלמות, אותם תיקונים עליהם דיברת בQED?
    האם אתה מכחיש שיש מצבים שבאמצעותן ניתן לקבל תוצאות מדוייקות לגמרי? לדוגמה אם נכפיל את קבועי החשמל והמגנטיות נוציא שורש ונעלה בחזקת מינוס אחד, האם לא נקבל בדיוק – אבל בדיוק – את מהירות האור?

    והאם לא הסכמתי איתך שיש מודלים שגויים שמפיקים זמנית ניבויים נכונים, אך לדעתי מודל מקסוול הוא שונה? קראת בכלל את מודל מקסוול? אולי אתה פשוט בורח מלהתמודד עם זה שאתה תמיד קורא חצי משפט וקופץ למסקנות בלי להבין בכלל מה מנסים להגיד לך?

    מה הבעיה שלך, הצורך הזה להוכיח תמיד לכולם שרק אתה יודע פיזיקה וכל היתר מטומטמים. מדוע אינך יכול לכתוב שני משפטים רצופים בלי לקלל או להעליב.

    אצל ווקי אנחנו כבר יודעים מה הבעיה. אבל מה אצלך?

    G

    אם אנחנו מסכימים, אז בסדר.

  44. הבהרה: חוקי מקסוול לא “שונו” ע”י איינשטיין ב-1905, אבל יש בפיזיקה עוד איזה מאמר או שניים שיצאו מאז. בפרט, הן שונו לחלוטין על ידי מכניקת הקוונטים.

  45. ישראל,

    צר לי לבשר לך, משוואות מקסוול מתקבלות ב-QED (אלקטרודינמיקה קוונטית, עליה זכה פיינמן בפרס נובל יחד עם שווינגר וטומונגה) רק כקירוב קלאסי. חלקיקים קוונטים בפירוש אינם מקיימים אותן (הכוונה היא כמובן לא לחלקיקים מיוחדים שהם “קוונטים” אלא לחלקיקים בסקאלות קוונטיות) ולא ניתן להשתמש בהן כדי לקבוע דינמיקה אלא במקרים בהם הקירוב הקלאסי תקף, בדיוק בדיוק בדיוק כמו מכניקה ניוטונית. היחס בין משוואות מקסוול לאלקטרודינמיקה קוונטית הוא *זהה* (הן מבחינה רעיונית והן מתמטית) ליחס בין החוק השני של ניוטון ומכניקה קוונטית. שתיהן מתקבלות כמשוואות תנועה קלאסיות, כלומר נקודת מינימום של הפאזה באינטרגל המסלולי הקוונטי. עזוב את זה שאני בכלל לא אמרתי כלום על תלמי או השוותי בינו לבין מקסוול, זה רק דברים שאתה ברחת אליהם כדי לא להתמודד עם הטענה שלי: תוצאות נכונות בתיאוריות שגויות זה משהו שקורה כל הזמן בפיזיקה (ואף הסברתי כיצד זה יכול לקרות, בפרט על ידי שימור סימטריה מסוימת בין המודל ה”נכון” והשגוי), ולכן אי אפשר לבסס מחקר או טענה פיזיקלית על כך שתוצאה אחת יוצאת נכונה. זו הסיבה שבמדע מחפשים הסבר למכלול התופעות, ולא מתייחסים לכל תוצאה בנפרד.

  46. י, אני רק רוצה לראות שהבינותי את התשובה שלך.. כתבת:

    ײלדעתי ניתן להרחיב את מודל האתר של מקסוול כך שיכלול גם אי לוקליות ואפילו הצעתי מודל שכזה – אתר אקטיבי. רוב הסיכויים הם שאני טועה כמובןײ.

    במילים פשוטות, זה אומר ש:

    1. מודל האתר המקורי הוא שגוי/חלקי ( אחרת מה האינטרס שלך להרחיב אותו בכלל)
    2. אולי אפשר לתקן אותו כך שלא יהיה שגוי/חלקי
    3. אבל רוב הסיכויים שהתיקון לא יצליח.

    האם זה מסכם נכון את מה שאתה אומר ( מינוס כל מיני הקדמות). כי אם כן, זה פחות או יותר מה שאני חושב וזה עונה לשאלה שלך.

  47. G

    התגובה שלך הומתנה וראיתי אותה רק עכשיו.

    השאלה שלך לגיטימית ופשוטה, אך התשובה אינה כה פשוטה.

    ראשית, בוא נראה מדוע מודל האתר של מקסוול שונה לדעתי ממודלים אחרים.

    אלבנצו הראה לי – כאילו שלא ידעתי מקודם – שגם מודלים אחרים ושגויים נותנים תוצאות נכונות לפעמים. אחד הידועים הוא המודל הגיאוצנטרי של תלמי, שבאמצעותו ניתן היה לחזות ליקויים למרות היותו שגוי. יש רבים אחרים.

    אך שים לב שאת מודל תלמי לא מלמדים בביה״ס ואת מקסוול כן. נכון, גם את מודל ניוטון השגוי מלמדים בביה״ס ומשתמשים בו יום יום באלפי שימושים בהנדסה וטכנולוגיה.

    אך יש משהו שמייחד את משוואות מקסוול, לדוגמה שבניגוד לחוקי ניוטון שעודכנו עיי איינשטיין כקירובים בלבד, משוואות מקסוול לא רק שלא השתנו אלא הן המניע העיקרי לכתיבת תורת היחסות הפרטית כפי שמעיד שם המאמר של איינשטיין מ1905: ״על האלקטרודינמיקה של גופים בתנועה״.

    בסרט ״גולדפינגר״ גולדפינגר הוא שחקן רמי שאין שני לו ובלתי ניתן להבסה. קיימים מודלים רבים של השגת יתרון במשחקי קלפים. ספירה, חישוב, קריאת הבעות הפנים של היריב, ורובם נכונים מהבחינה שהם נותנים יתרון למשתמש בהם.

    (אני יודע את זה מנסיון אישי. במשך 20 שנה הייתי סופר קלפים מקצועי בבלק ג׳ק, והיום רוב בתי הקזינו בנוודה וקליפורניה אוסרים עלי לשחק ופרצופי המכוער תקוע לנצח במערכת זיהוי הפנים של בתי הקזינו).

    לבונד, ג׳ימס בונד, יש השערה שונה לגבי מודל הנצחונות של גולדפינגר. גולדפינגר פשוט מרמה כפי שזכור למי שראה את הסרט.

    אז זה ההבדל לדעתי בין המודלים האחרים למודל מקסוול. כאשר אתה קורא ומבין אותו, אתה רואה שכמעט שאין סיכוי שמודל מורכב כזה הוא צירוף מקרים מוצלח בלבד כמו מודל תלמי.

    בדיוק כמו שכאשר בונד תופס את המשת״פית של גולדפינגר עם הטלסקופ והרדיו, זהו סלאם דאנק לגבי נכונות תאוריית הרמאות.

    אז אתה שואל ״האם אתה טוען שיש אתר? שאין אתר? שיש כישוף בעולם?״

    איני יודע עד כמה עקבת אחרי הדיונים בנושא תורת היחסות ואי לוקליות שהתנהלו כאן ובמקומות אחרים בשנים האחרונות. הטענה שלי היא שאי לוקליות ויחסות אינן יכולות להתקיים זו לצד זו, ושאף שאין עדיין סתירה ישירה או הוכחה לכך, אי לוקליות מציבה אתגר פיזי קשה, אולי קשה מנשוא, לתורת היחסות.

    לדעתי ניתן להרחיב את מודל האתר של מקסוול כך שיכלול גם אי לוקליות ואפילו הצעתי מודל שכזה – אתר אקטיבי. רוב הסיכויים הם שאני טועה כמובן, ויש גם אינדיקציות לא קטנות לכך, אך אני מקדיש הרבה זמן וכסף לניסויים שיאשרו או יפריכו את הרעיון שלי (הזכרתי פה אחד מהם).

    מתן אומר שהוא שם את הכסף איפה שהפה, ואפילו ציין סכום של 20,000 שקל שהשקיע בניסויים.

    אני מעריך שהסכום המשוקלל (כולל הפסד שעות עבודה) שהשקעתי בניסויים עובר את החצי מיליון – דולר.

    עכשיו תורך.

  48. מה קרה ווקי? ברגע שהיית צריך להתעמת עם המציאות השתפנת? הרי נתנו לך הזדמנות להוכיח לי שאיני יודע על מה אני סח, שאיני מבין, שאני משקר וכל יתר הכינויים החביבים עליך.

    אז בסוף מסתבר שאינך מבין בכלל בפיזיקה, אינך יודע מהו מודל מקסוול, מה ההבדל בינו למודלים האחרים..

    איפה ה״אני אעזור לך להבין״ השחצני הקבוע שלך? איפה הפטרוניות המתנשאת הקבועה שלך?

    תחפף חזרה למטכ״ל. זה המקום היחיד שבו מישהו בכלל סופר אותך, אפס מתנשא.

    ולמי שמתעניין: ישנה סיבה לכך שמשוואות מקסוול נחשבות לאחת היצירות הגדולות ביותר של הרוח האנושית (למרות שהן לא בדיוק של מקסוול אלא מיחזור של גאוס ואמפר). לא תמצאו סטודנטים מסתובבים עם משוואות תלמי על החולצה, אבל של מקסוול כן, אפשר לקנות אותן באמאזון.

    האם הן מושלמות? במצבים מסויימים כן. ניתן לדוגמה לחשב באמצעותן את מהירות האור בלי לצפות בליקויים כמו רמר או לצפות בגלגל מסתובב כמו פיזו, אלא עיי מדידת קבועי החשמל והמגנטיות במעבדה.

  49. ישראל,

    עזוב את WD הוא לא ראוי לתגובה, כולנו רואים מי כאן התינוק בסיפור וזה לא אתה.

  50. ל- WALKING DEATH האפס

    הידען הוא לא המקום לתגובות מבזות וקטנוניות כשלך.

  51. ישראל

    אתה תינוק. כל פעם שמישהו אומר לך משהו שלא נעים לך לשמוע אתה ישר נכנס לטילט וקופץ שהוא התחיל איתך ועכשיו בעולם העקרונות התינוקי שלך אתה מחוייב להראות לו.

    צר לי לבשר לך.
    על ה**** שלי.
    אין לי זמן לבזבז עליך.

    על אנשים שלא תקועים במקום כבר עשור באותה נקודה בדיוק כי הם חושבים שהשמש זורחת להם מהמקום בו השמש לא זורחת ומסרבים להקשיב לאחרים ולנסות להבין או ללמוד מהם אני עוד מוכן לנסות להשקיע אנרגיה בלעזור להם. אתה אבל לא אחד מהם.

  52. מתן,

    אני רוצה להבהיר דבר חשוב שאולי הוא זה שבלבל אותך, אתה לא צריך להבחין בשן אחת ספציפית בגלגל! כל הקטע זה מיקום יחסי של השיניים בכל מהירות סיבוב!

    1. אם במהירות סיבוב מסויימת (סל״ד) אור שיוצא ממרווח בין שתי שיניים מצליח לחזור לעינו של הצופה אז זה יהיה נכון ***עבור כל מירווח בין שתי שיניים בגלגל***, אור שיצא דרכן החוצה תמיד יצליח לחזור לעינו של הצופה!

    2. ואם נגביר לאט ובהדרגה את מהירות הסיבוב כך שאור שחוזר חזרה נתקל בשן שחוסמת את דרכו ***אז זה יהיה נכון עבור כל מיקום בגלגל*** ומרגע זה הצופה יראה כל הזמן חושך!!

    3. ואם נמשיך להגביר בהדרגה את המהירות ופתאום שוב נראה אור (כאמור 50% מהזמן אם גודל כל שן שווה לגודל המרווח בין השיניים) אז מרגע זה והלאה במהירות זו תמיד נראה אור! וגם נדע ****שבאופן יחסי**** היה מעבר של שן אחת בודדת ולא יותר!

    האם ההסבר הזה מובן לך?

    כל הקטע זה מיקום יחסי, ולא הבחנה בשן ספציפית!

  53. ווקי, לא אמרת שאתה הולך ומשחרר אותי?

    אני בטוח שיש מגיבים שנזקקים בדחיפות לשרותיך. שתעזור להם להבין, שתסביר להם את הנקרא, שתתעלק עליהם, שתתנשא עליהם, שתגיד להם כמה קשה איתם ואיזה מזל שיש להם שאתה פה בשביל לעזור.

    נכון, אף אחד לא פונה אליך אף פעם בבקשת יעוץ או עזרה, אך מה זה משנה? למי פה חשובות העובדות? העיקר שתוכל לזקוף חוטמך למרום ולהרגיש עד כמה אתה נעלה על כל הנמושות.

    כי אצלך אין דבר כזה דיעה שונה, לך ניתנה האמת ממעל וכל המעז לא להסכים איתך הוא בהכרח טועה וכחן וקשה תפישה.

    אבל אתה טועה. אילו היית פעם אחת מקלף מעצמך את כל הצדקנות המתחסדת ומפסיק לחנך פה את כולם, והיית מוכן להתייצב פעם אחת גבר מול גבר בלי פסיכולוגיה והטפות מוסר, הייתי משתדל להראות לך שאינך כה חכם כפי שאתה חושב את עצמך ושאתה טועה כמעט בכל נושא, כולל הנושא שבו אתה מחנך אותי בכתבה זו.

    מוכן לקחת את האתגר? זכור, בלי פסיכולוגיה, בלי שמות תואר, רק לוגיקה ופיזיקה.

    אתה.. אתה מבין בפיזיקה, נכון? אין לך בעיה לקרוא את מודל מקסוול אני מאמין? אתה שולט בהידרודינמיקה ומשוואות דיפרנציאליות כמובן?

  54. WALKING DEATH

    אתה פשוט אדם נבזי

    הידען – אולי תשקלו פשוט לא לפרסם תגובות כאלה?

  55. ישראל

    אוי איזה קורבן מסכן אתה, רחמים עליך.

    אתה לא עובד על אף אחד, כולם כאן רואים ומבינים בדיוק מי ומה אתה.

  56. נו טוב, עוזר להבין, דביל היית ודביל תשאר.

    אבל כל עוד אתה משחרר אותי ומתעלק על קורבנות חסרי ישע אחרים עם הפלצנות המתנשאת שלך – אתה עוזר להבין, אתה, האדם המפגר בשכלו, אז בסדר.

    חזרה לפקודות מטכ״ל.

  57. ישראל

    ‘אני דווקא מאמין שאני מבין מה אנשים אומרים לי.’

    זו הבעיה הראשונה שלך. יש לי כבר מאות ראיות לכך שזו אמונה מחוסרת בסיס.
    השניה היא הקטיף דובדבנים שאתה עושה לדברים שאנשים אומרים לך, מה שרק מחריף את הבעיה הראשונה.

    עכשיו מכיוון שאתה מבקש כל כך יפה אז אשחרר אותך מהעול ואותיר אותך לדבר רק עם אנשים שלא פוגעים ב או מאתגרים את התפיסה העצמית העקומה והלקויה שלך.

  58. מתן

    ‘שוב יש לי טענה אחת – אתה יכול לראות פעם חושך ופעם אור, אתה יכול לראות הבהוב – אבל בגלל שזה היה כל כך כל כך מהר 18,000 פריימים בשנייה – אין סיכוי שאתה תזהה שן אחת שעברה! זה כל הסיפור.הסיפור.’

    אני אקח את זה כדבר הכי קרוב שאני אקבל ממך לתשובה לשאלות ששאלתי אותך.

    אז קודם כל אתה צודק שאף אחד לא מסוגל להבחין במעבר של שן אחת או אחרת במהירויות הרלוונטיות כאן.
    זה צריך להיות הרמז הכי ברור עבורך שלא זה מה שנעשה (וכבר אמרו לך מספר פעמים שזה אכן לא מה שנעשה).

    מה שאתה צריך לשאול את עצמך זה איך מבינים שהאור עובר שן אחת בלי שיש צורך שבן אדם יבחין במעבר של שן אחת. וזה בדיוק מה שמנסים להסביר לך כאן.

    תנסה לחשוב איך היית מבין, במצב זה שאתה לא מסוגל להבחין במיקום של השיניים בכלל, שהאור נחסם על ידי שיניים שעוברות.

  59. מתן

    ‘שוב יש לי טענה אחת – אתה יכול לראות פעם חושך ופעם אור, אתה יכול לראות הבהוב – אבל בגלל שזה היה כל כך כל כך מהר 18,000 פריימים בשנייה – אין סיכוי שאתה תזהה שן אחת שעברה! זה כל הסיפור.הסיפור.’

    אני אקח את זה כדבר הכי קרוב שאני אקבל ממך לתשובה לשאלות ששאלתי אותך.

    אז קודם כל אתה צודק שאף אחד לא מסוגל להבחין במעבר של שן אחת או אחרת במהירויות הרלוונטיות כאן.
    זה צריך להיות הרמז הכי ברור עבורך שלא זה מה שנעשה (וכבר אמרו לך מספר פעמים שזה אכן לא מה שנעשה).

    מה שאתה צריך לשאול את עצמך זה איך מבינים שהאור עובר שן אחת בלי שיש צורך שבן אדם יבחין במעבר של שן אחת. וזה בדיוק מה שמנסים להסביר לך כאן.

    תנסה לחשוב איך היית מבין, במצב זה שאתה לא מסוגל להבחין במיקום של השיניים בכלל, שהאור נחסם על ידי שיניים שעוברות.

  60. מתן
    העניין של זמן הליקוי הוא לא כמה זמן היה הליקוי – אלא המועד שלו. רמר הסביר את התופעה שבה הליקויים מתעקבים כשצדק רחוקה לעומת הזמנים שבהם צדק קרובה.

    1. ההבדל בין המרחק הכי קרוב לצדק למרחק הכי רחוק לצדק הוא 300 מיליון ק”מ. את זה כבר ידעו היוונים הקדמונים (בקירוב מפתיע).

    2. המרחק הזה הוא 1000 שניות אור, כלומר – הליקויים הרחוקים מתעכבים בכ-17 דקות.

    3. המדידות אכן הראו את העיקובים האלה.

    שים לב שלא חשוב כאן המרחק לצדק. וגם שים לב שלא חשב כמה זמן נמשך הליקוי – כל מה שמעניין זה תחילת הליקוי.

    בקשר לניסוי פיזו – תחשוב על הדוגמה הבאה: ניקח 2 גלגלי שיניים כאלה ונשים אותם על ציר אחד. נעשה מספר ניסויים, שבכל ניסוי נסובב גלגל אחד יחסית לגלגל השני בחלקיק של מעלה. זכור – שן מכסה רבע מעלה.

    1. כשהחפיפה מלאה (שן מעל שן) – יעבור מחצית מהאור.
    2. כששן מכסה בדיוק את הרווח – לא יעבור כל אור. סיבבנו סה”כ רבע מעלה.
    3. בוא נסובב עשירית מעלה ממצב (2): עכשיו יעבור 40% מהאור.
    4. נסובב מאית מעלה ממצב (2): עכשיו יעבור רק 4% מהאור.

    כלומר – קיבלנו מכשיר רגיש למדי למדידת זווית, בטכנולוגיה שהייתה קיימת במאה ה-19.

    מתן – אני לא מבין למה אתה חותר. בוא נעזוב את העבר. האם אתה לא מסכים שמהירות האור היא 300 אלף ק”מ לשנייה?

  61. פלוני 321 – נעלמתי כי השקעתי בזה הרבה שעות כל יום.
    (וגם מבאס אותי שהם מפרסמים את כל ההודעות שלי רק פעם אחת ב-24 שעות )

  62. מתן,

    בהנחה שגודל השיניים בגלגל היה זהה לגודל המרווחים ביניהן, אז אם מהירות הגלגל היא כזאת שאור שיצא ממרווח בין שתי שיניים (ופגע במראה הרחוקה) יצליח לחזור ולעבור גם הוא דרך מרווח בדרכו חזרה, זה אומר ש 50% מהזמן אתה תראה אור. או אם נלך לפי דוגמת הסרט המצוייר שלך, יהיו בסרט 9,000 תמונות רצופות של אור, ואז 9,000 תמונות רצופות של חושך (כאשר אור שמנסה לצאת החוצה נחסם ע״י שן של הגלגל) וחוזר חלילה.

    ואם תגביר לאט לאט את המהירות עד שכל פוטון של אור שיוצא ממרווח בין שתי שיניים יפגע בדרכו חזרה בשן וייחסם, אז 100% מהזמן יהיה לך חושך.

    ואם תמשיך להגביר בהדרגה את מהירות הגלגל, הוא שוב יגיע למהירות שבה הפוטונים החוזרים יצליחו לעבור דרך מרווח (המרווח שנמצא מיד אחרי השן שחסמה קודם את הפוטונים החוזרים) ואז שוב תראה 50% מהזמן אור, ותדע בוודאות שהיה מעבר של שן אחת בודדת ולא יותר.

  63. שלום מתן וברוך השב,

    לגבי מדידת מהירות האור ע”י ליקויי הירח של צדק איו, אני לא בטוח בזה אבל לפי דעתי אתה צודק,

    (בדומה לשתי הדוגמאות שהבאת, הנה דוגמא משלי: אם מישהו יטוס בחללית אל השמש, וממש סמוך אליה יפרוס מניפה עצומה שתסתיר מאיתנו את השמש לחלוטין, נגיד שסיכמנו איתו מראש שמשך הזמן שהמניפה תהיה פתוחה הוא 10 דקות בדיוק, אם לא דיברנו על השעה המדוייקת שבו יפרוס את המניפה אלא רק על משך הזמן, זה לחלוטין לא יעזור לנו לדעת את המרחק לשמש / את מהירות האור מכיוון שמשך הזמן בו השמש תוסתר מאיתנו יהיה בדיוק 10 דקות לא משנה מה מרחקה מאיתנו)

    אם אני מבין נכון המדידה מבוססת על זה שהמרחק בינינו לאיו משתנה *תוך כדי הליקוי* דהיינו אם בתחילת הליקוי המרחק הוא איקס, בסוף הליקוי, כשאיו מתגלה שוב לעינינו, המרחק שלו כבר שונה בכמה מאות אלפי ק”מ ולכן אורך הליקוי יהיה ארוך או קצר יותר בשנייה או שתיים.

    רק מה שלא מסתדר לי זה – א. צריך לדעת את מסלולו של צדק ואת מסלולי ירחיו בדיוק עצום כדי להבחין בשינוי של שנייה אחת מהערך הצפוי (משך הזמן של ליקויי איו הוא לא קבוע אלא נע בערך בין 2.5 ל-3.5 שעות), קשה לי להאמין שלפני 350 שנה מסלוליהם של ירחי צדק היו ידועים בדיוק כה גדול.

    ב. קשה לומר מתי התחיל ומתי נגמר הליקוי ברמת דיוק כזו ללא מצלמות ועם טלסקופים של לפני 300 שנה, מכיוון שלעיתים איו לא נכנס לצל של צדק ונעלם אלא ממשיך להאיר כל הזמן והוא רק מוסתר ע”י צדק עצמו, הבעיה היא שכאשר הוא מתקרב אליו ומתחיל להתכסות על ידו קשה מאוד להבחין ביניהם.

    גם אני אשמח לשמוע תשובות ממי שמבין בנושא.

    לגבי ניסוי פיזו, (אקדים ואומר שאני לא באמת מתמצא בנושא אבל אני חושב שהבנתי את ההסברים של המגיבים האחרים) אני אנסה לעשות את ה פשוט, התמונה היא כזו: אתה יושב על הקרקע, לפניך, במרחק כמה מטרים ניצב לו גלגל שיניים גדול, מאחוריך מאיר פנס היישר קדימה אל גלגל השיניים, ובמרחק כמה קילומטרים מממך מעבר לגלגל השיניים עומדת לה מראה,

    הפנס שמאחוריך נדלק ומה שאתה רואה זה גלגל שיניים מואר באור יקרות, אבל גם המקום של החריצים-היכן שאין שיניים- “מואר”, בגלל שהאור שחלף דרך החריצים הגיע למראה וחזר משם לאחורה באותה דרך היישר לעינך, ולכן תראה עיגול שלם של אור שמורכב מהשיניים שמוארים ישירות מהפנס והחריצים שדרכם אתה רואה את המראה שמחזירה את האור שעבר דרך החריצים ופגע בה.

    כעת מישהו מתחיך לסובב את גלגל השיניים, והוא מגביר את המהירות אט אט, בהתחלה לא תראה שום שינוי כי האור כל כך מהיר שהוא מספיק ללכת למראה ולחזור דרך אותו סדק שהוא עבר בו, אבל בשלב מסויים תראה שכמות האור קצת מתעמעמת כי חלק מהאור שיצא דרך החריץ כבר נתקל בדרכו חזרה בשן ההסמוכה לחריץ ממנו הוא יצא, ככל שהמהירות תגבר כך האור יילך ויתעמעם עד שבשלב מסויים הוא יגיע למינימום, זה יקרה כאשר מהירות הסיבוב היא כזו שבזמן שהאור נוסע אל המראה וחוזר, גלגל השיניים הספיק להסתובב כך שבמקום בו היה לפני מיליונית השניה חריץ פתוח, יש עכשיו שן אטומה, והאור שחוזר מהמראה נתקל בשן הזו ולא יגיע לעיני, (כמובן שעדיין אראה אור מהפנס שמאיר את שיני גלגל השיניים ישירות, אבל האור שעבר דרך החריצים לא יוחזר לעיני כך שכמות האור הכללית תרד, (מדובר רק כאשר אורך השיניים שווה או גדול מאורך החריצים, אחרת לא כל האור ייחסם).

    שאלת איך אני יודע שעברה שן אחת ולא יותר, התשובה היא שמגבירים את מהירות הסיבוב באיטיות רבה ורואים שכמות האור מתחילה לרדת בשלב מסויים עד שהיא מגיעה למינימום, ואז שוב עולה באופן רציף וחוזר חלילה. במינימום הראשון אני יודע שבמהירות הסיבוב הנוכחית, השן מספיקה לזוז בדיוק את המרחק שלה עצמה (אם אורך השיניים והחריצים שווה) בזמן שהאור הגיע למראה וחזר, כשכמות האור חוזרת לתקנה אני יודע שמהירות הסיבוב כרגע היא כזו שבזמן שלוקח לאור להגיע למראה ולחזור, הגיע החריץ השני למקום בו היה הראשון כשהאור יצא דרכו, ושוב, מעלים את המהירות לאט לאט, ורואים שכמות האור משתנה באופן רציף בלי קפיצות.

    מקווה שהייתי ברור.

    (סתם סקרנות, הפסקת להגיב כי השתכנעת או שהתייאשת מלשכנע אותנו?)

  64. אני רוצה לענות לך, ואני מבטיח לענות לך מיד אחרי שתענה על מה ששאלתי אותך. וזו שאלה פשוטה.. הרי אתה כתבת את זה נכון? “אז אם אין אתר והמודל שגוי – איך הצליח לו החישוב?”
    רק את זה, למה התכוונת? האם אתה טוען שיש אתר? שאין אתר? שיש כישוף בעולם? האם אתה יכול להסביר את זה במילים פשוטות שגם ילד יוכל להבין?

  65. ווקי

    אני דווקא מאמין שאני מבין מה אנשים אומרים לי.

    למשל את זה שאמרת לפני כמה זמן שנפסיק להגיב זה לזה. טעיתי? להראות לך איפה?

    אני בהחלט מכבד את המשימה החשובה שלקחת על עצמך לעזור לאנשים להבין, רק מבקש שלא תכלול אותי בין אותם ברי המזל שאתה עוזר להם להבין. אתה.

    ואני – אני כבר אמצא איכשהו אנשים שיגיבו לי, אבל עוד הפעם, תודה.

    G

    אולי השאלה שלי לא היתה ברורה, אז אחזור עליה:

    האם אתה טוען שמשוואות מקסוול שגויות?

    בבקשה, רק את זה.

  66. לא הבנתי. כתבת “אז אם אין אתר והמודל שגוי – איך הצליח לו החישוב?”

    מה אתה טוען במשפט הזה.. יש אתר? אין אתר? או שאתה סתם זורק נקודות למחשבה?

  67. ישראל

    אם אי פעם היה או יהיה לך ספק למה אנשים מפסיקים לדבר איתך, תדע שזה בגלל שאתה מתאמץ לא להבין או להבין לא נכון מה שהם אומרים לך(או לפחות גורם לזה להראות ככה).

  68. אלבנזו

    כמובן שדיברת רק על פיזיקה, כמו תמיד. וזה אני שכתבתי:

    ״בבקשה תעשה מאמץ. תתרכז. תקרא מה שכותבים לך״.

    ״אם אתה רוצה להתעסק במילים ולא בפיזיקה, אז בבקשה אל תיתמם ותעמיד פנים״.

    פיזיקה נטו, ללא ספק.

    וכמובן שלא התייחסת למה שאני אומר כלל וכנראה לא הסתכלת במודל מקסוול, ואם הסתכלת לא הבנת את ההבדל המהותי בינו לבין יתר המודלים השגויים שמפיקים זמנית תוצאות נכונות (תלמי, פלוגיסטון, וכו).

    נוכל להתייחס אליך בטיפת כבוד ולא כמו הקשקשן הפתטי שאתה, אם תתחיל לעמוד מאחורי מה שאתה אומר ולא תכנס מיד לנימות אישיות.

    תתחיל בזה שתביא את ההוכחה המתמטית שאתה טוען שקיימת לזה שבין חלקיקים שזורים לא עוברת אינפורמציה . הרשימה עוד ארוכה.

    אתה יכול כמו תמיד להתנצל על התוקפנות שלך ולסיים פה. אם תמשיך, תחטוף חזרה כמו תמיד.

    G

    ״אם משוואות מקסוול נכונות, ויש אתר, אז מדוע הן לא מצליחות לחשב את קרינת גוף שחור? אפילו את צבע האור של נורה פשוטה הן לא יודעות לחזות. למה?״

    לא הבנתי, אתה טוען שמשוואות מקסוול אינן נכונות? תוכל להגיד את זה בצורה ברורה?

  69. איזה ג’וקר. תגיד, לדעתך יש פה מישהו חוץ ממני וממך שקורא את התגובות ולא היה לו ברור שתתעלם מכל מה שאמרתי (העובדה שזה קורה כל הזמן, ההסברים לאיך זה יכול לקרות, הדוגמאות הרבות, ואפילו ההתייחסות לסימטריה בבעיה הספציפית עליה מדובר) ובמקום רק תעלב ותאשים אותי בכך שאני “מדבר על חינוך במקום פיזיקה”, למרות שבשלוש התגובות האלה יש יותר פיזיקה מכל התגובות שאתה כתבת באתר בשנה האחרונה?

    אבל כמו שאמרתי, תמשיך בשלך. אני מקווה שברור לך שבאמת לא איכפת לי מה אתה חושב ושאין לי אינטרס לשכנע אותך. את התגובות כתבתי רק למקרה שמישהו שצופה מהצד יחשוב שאולי יש משהו בנקודה שאתה אומר, ונראה לי שעכשיו זה די ברור.

  70. אם משוואות מקסוול נכונות, ויש אתר, אז מדוע הן לא מצליחות לחשב את קרינת גוף שחור? אפילו את צבע האור של נורה פשוטה הן לא יודעות לחזות. למה?

    תקלה זמנית באתר, מיד נשוב?

    כישוף שהוטל ע”י הקוסם הרשע פלאנק?

    קטסטרופה על סגולה?

    נקודה למחשבה.

  71. אלבנצו

    אני רואה שאתה שוב חוזר לחינוך במקום לדבר פיזיקה. כרצונך.

    אם תתרכז ותקרא את מה שכותבים לך, תוכל לקרוא את מודל מקסוול מהקישור שהבאתי. ואז אולי תוכל לראות את ההבדל בין המודל המורכב והעדין הזה למודלים כגון מודל תלמי או מודל האטום הראשוני.

  72. רשמתי מוקדם קודם 2 הערות על פיזו. אנא עיינו.

    דבר אחר,מישהו העלה את אולה רמר על מדידת מהירות האור על בסיס הירח איו שחג מסביב לצדק.

    אני באמת רוצה להעלות שאלה בצורה כנה וללא הקנטה:
    יוצאים מנקודת הנחה שתמיד הליקוי של איו הוא אותו זמן מקומי, פשוט המרחק שלנו משתנה מצדק ולכן יש הבדל של 22 דקות בליקוי, וככה חישבו את מהירות האור.
    שאלתי – מה זה משנה מה המרחק שלנו מצדק? אם הליקוי של איו הוא זמן קבוע X, אז ככל שתהיה יותר רחוק אתה תראה את הליקוי בזמן אוחר אבל עדיין הליקוי יהיה זמן X.
    אתן 2 דוגמאות –
    1. בעבר שדיברנו בטלפון עם חו”ל הייתה השהייה של נניח 2 שניות. אם מישהו בארהב מדבר 10 שניות, אז אתה תתחיל לשמוע אותו לאחר 2 שניות, אבל עדיין תשמע אותו במשך 10 שניות.
    2. נניח שהשמש עושה סופר נובה של 10 שניות (סתם רק בשביל הדוגמה). אם אנחנו בארץ נראה זאת לאחר 8 דקות אבל עדיין נראה 10 שניות את הפיצוץ. ואם מישהו בצדק אז הוא ייראה זאת לאחר 40 דקות אבל עדיין ייראה 10 שניות את הפיצוץ.
    לכן, אשמח להסבר מה זה משנה מה המרחק שלנו מצדק, אם יוצאים מנקודת הנחה שהליקוי של איו מסביב לצדק הוא קבוע.
    באמת שאלה כנה.
    מתן

  73. אולי בתגובתי הקודמת לא נתתי מספיק אינטואיציה לאיך דברים כאלה יכולים לקרות, ולכן זה עדיין פלא מבחינתך ולכן אתה לא משוכנע. אז בנוסף לדוגמאות שנתתי מקודם, אוסיף הסבר קצרצר.

    לכל מודל או מערכת מתמטית יש אוסף תכונות שמגדירות אותה. האובייקטים שהיא מכילה, הקשרים ביניהם וכו’. לפעמים יתכנו שני מודלים שונים (לצורך העניין, אחד מהם “נכון” והשני לא, וברור שכשאני אומר “נכון” הכוונה היא רק בקונטקסט מסוים, למיטב ידיעתנו, בהתאם לתצפיות הידועות וכו’) שחולקים חלק מהתכונות. לכן, תוצאות שנגזרות באופן ישיר מתכונה (או סט של תכונות) שהמודלים חולקים יהיו זהות בשני המודלים, וגם זה שאינו “נכון” יספק תשובה “נכונה”. למשל, רוב התכונות המיוחסות למהירות האור (היותה חסם עליון למהירות של כל גוף מאסיבי, היותה המהירות היחידה בה חלקיק חסר מסה יכול לנוע, שלל קשרים בינה לבין אלקטרומגנטיות) נובעות מסימטריה פשוטה של תורת היחסות. מודל שונה, גם אם הוא שגוי לגמרי, יכול לקיים גם הוא את אותה הסימטריה ולכן לפחות חלק מהתכונות האלה יתקיימו גם במסגרתו.

    זהו לא ההסבר היחיד למודל שגוי שמניב תוצאות נכונות, אבל הוא פשוט למדי ורלוונטי במקרה הזה. שוב, זה לא במקרה שהשיטה המדעית מחפשת רק מודלים שיכולים להסביר את כל התופעות. הסברים חלקיים (שמתאימים לתופעות מסוימות, אפילו בדיוק מופתי) יש כמו מים בים.

  74. ישראל,

    בבקשה תעשה מאמץ. תתרכז. תקרא מה שכותבים לך.

    כן, זה אפשרי שמודל שגוי יתן תוצאה נכונה. זה קורה כל הזמן. זה לא חייב להיות צירוף מקרים (לפעמים זה כן): זה יכול להיות שתי טעויות במודל שמקזזות זו את זו, זה יכול להיות שהשגיאה של המודל בדיוק לא באה לידי ביטוי בשאלה הספציפית שנשאלה, זה יכול לקרות ממגוון סיבות. זה קורה כל הזמן ואף קיבלת דוגמאות לכך, בעיני אפילו יותר מרשימות.

    זה ממש לא משנה מה איינשטיין אמר. אבל אם אתה רוצה להתעסק במילים ולא בפיזיקה, אז בבקשה אל תיתמם ותעמיד פנים שאינך יודע שאיינשטיין דיבר על כך שלמרחב יש תכונות פיזיקליות (בפרט, צפיפות אנרגיה). הוא אינו אתר כמו האתר של מקסוול שמצוי במרחב. הוא המרחב עצמו. אבל ממש לא מעניין אותי לעשות איתך מלחמת ציטוטים ופרשנויות. תאמין במה שבא לך. שאלת שאלה, קיבלת תשובה. תוצאות נכונות במודלים שגויים זה דבר שמדענים נתקלים בו מדי יום. בהצלחה.

  75. אלבנצו.

    עבור בבקשה על המודל עד משוואה 136 שבה מקסוול גוזר את מהירות האור מתוך קבועי החשמל והמגנטיות, ואמור לי אם צירוף מקרים לא יאומן כזה הוא אפשרי.

    בעבר נשמעה פה הטענה שמקסוול כבר ידע מראש שהוא יקבל את מהירות האור מתוך המודל, והוא רק סימן את המטרה במקום פגיעת החץ. מקסוול עצמו הכחיש זאת, וטען שהופתע כשגילה שמהירות גל באתר שלו היא מהירות האור.

    אך למה ללכת רחוק? איינשטיין עצמו אמר שללא אתר היחסות אינה אפשרית, רק שאין לייחס לו מערכת מנוחה מוגדרת (קישור אם יש ביקוש).

  76. ישראל,

    התופעה שבה תיאוריה לא נכונה מביאה לתוצאה מסוימת שהיא כן נכונה היא מ-מ-ש לא דבר נדיר. בגלל זה במדע מחפשים תיאוריה המסבירה את *מכלול* התופעות ולא משהו נקודתי. ישנן דוגמאות רבות, למשל הקשר הנכון בין המסה של חור שחור לבין הרדיוס שלו בפיזיקה קלאסית (תוך התעלמות מיחסות כללית), על אף פי שאנו יודעים מניסוי שכבידה ניוטונית היא תיאוריה שגויה שפשוט נותנת פרדיקציות לא נכונות. ישנן תוצאות פיזור מסוימות אשר מכניקת הקוונטים חוזה בדיוק גבוה למרות שהיא אינה יחסותית ושוב, ישנם ניסויים שמראים שהיא נותנת פרדיקציות ממש שגויות במקרים מסוימים. ישנן דוגמאות גם יותר מתמטיות, כמו העובדה שפיתוח בפרמטר N גדול (לא חשוב כרגע מי הוא בדיוק N, הוא קשור לכמה סימטריה יש במערכת קוונטית) נותן תוצאות טובות גם עבור N=3 או שהצבה של q=2 טור הנדסי (שכמובן לא מתכנס עבור ערכים גדולים מ-1) נותנת בדיוק את התוצאה הנכונה של הרגולרזיציה של הטור.

  77. בגלישה דרך סמרטפון הכתבה והתגובות כל רגע קופצים למעלה ולמטה וזה ממש משגע.

    תגובות אחרונות הרבה פעמים לא באמת מראות את התגובות האחרונות וגם יש כפילות בתצוגה (אותה הודעה מופיעה כמה פעמים) וגם חסר מאוד המלבן שמראה את התגובות האחרונות כבר בדף הראשי.

    לדעתי מאוד חסרה גם תצוגה ליד כל כתבה בדף הראשי שמראה כמה תגובות יש בה.

    אבל הקפיצות של הדף זה ממש משגע וחייב להיפתר (זה נגרם בגלל הכתובית הרצה בראש הדף).

    האם אבי בליזובסקי מודע בכלל לבעיות האלו? האם מישהו שם מנסה לטפל בזה?

  78. אני קורא את התגובות של כולם .

    שוב יש לי טענה אחת – אתה יכול לראות פעם חושך ופעם אור, אתה יכול לראות הבהוב – אבל בגלל שזה היה כל כך כל כך מהר 18,000 פריימים בשנייה – אין סיכוי שאתה תזהה שן אחת שעברה! זה כל הסיפור.

    ניקח לדוגמה סרטון אנימציה של דיסני עם 18,000 תמונות מצויירות של נסיכה בשנייה ,
    אם אני אשים לכם 1 תמונה (מתוך 18,000 בשנייה) של משהו אחר לדוגמה מגדל האם תזהו את המגדל? ממש לא. זה זמן של 0.00005 שניות
    אם אני אשים לכם 10 תמונות רצופות של משהו אחר לדוגמה מגדל האם תזהו את המגדל? ממש לא. זה זמן של 0.0005 שניות
    אם אני אשים לכם 100 תמונות רצופות של משהו אחר לדוגמה מגדל האם תזהו? גבולי. אני מאמין שרוב הסיכויים לא. זה זמן של 0.005 שניות. זה 5 אלפיות השנייה.
    אם אני אשים לכם 1000 תמונות רצופות (מתוך 18,000 בשנייה) של משהו אחר לדוגמה מגדל האם תזהו? כן נתחיל לזהות כי זה 5 מאיות השנייה, והעין יכולה להבחין בזה.

    עכשיו בוא נמשיך,
    אם אני אשים לכם 1 תמונה (מתוך 18,000 בשנייה) של מגדל ואז 1 תמונה של נסיכה ואז 1 תמונה של מגדל, ואז 1 תמונה של נסיכה, וחוזר חלילה – מה נזהה?

    העין שלנו תראה לפעמים מגדל ולפעמים נסיכה, אבל שוב (!!!!!) לא תוכל להגיד כמה זמן עבר בין 1 נסיכה ובין 1 מגדל! העין שלך לא יכולה לקלוט זמנים של 0.00005 שניות

    וזה כל הסיפור.
    ושוב אני אומר בפעם המיליון – ה-1 שן היא קריטית כי אם עברו בטעות 2 שיניים – אז המהירות היא רק 150,000 ק”מ בשנייה.

  79. וכדי לקלקל את השמחה, השאלה הקבועה שלי:

    פיזו חישב את מהירות האור באמצעות ניסוי. מקסוול, שנים אחרי ניסוי פיזו, חישב את מהירות גלים אלקטרומגנטים באמצעות מודל הידרודינמי של האתר, ולהפתעתו גילה שזו מהירות האור אותה מצא פיזו.

    https://en.wikisource.org/wiki/On_Physical_Lines_of_Force

    אז אם אין אתר והמודל שגוי – איך הצליח לו החישוב?

    ניחוש מוצלח?

    קסם?

    כישוף?

  80. עוד הפעם הפרטים לא נשמרים והכתבות קופצות באייפון..

    אבי, מה היה רע בפורמט הישן, שבו גם ניתן היה לראות את כל התגובות מימין?

  81. ניסים,

    זה בדיוק מה שרשמתי בתיאור שלי, שברגע שמגבירים את מהירות הסיבוב (אחרי שראינו קודם אור) ורואים חושך אז יודעים בוודאות שזו השן שמיד אחרי החריץ דרכו חזר קודם האור, וכשהמהירות גוברת ושוב רואים נצנוצים של אור אז יודעים שזה החריץ שנמצא מיד אחרי החריץ הקודם שראינו דרכו אור.

  82. יריב

    כן, הבעיה כאן היא לנסות להסביר למתן בצורה שהוא יבין. הוא משום מה חושב שצריך לצפות בשיניים ולהבחין בשן אחת ספציפית באיזה אופן במהלך הניסוי.

  83. יריב
    כן, אתה צודק. הרעיון שרציתי להדגיש הוא שאם מתחילים לאט ומאיצים, אז אפשר לדעת שמה שחוסם זה השן הקרובה ביותר לחריץ. כלומר – בצורה הזו, התוצאה היא חד-חד-ערכית.

  84. ישראל
    נניח 10 סיבובים לשנייה. יש 720 שיניים, אז הזמן בין שן לחריץ הוא 1 חלקי 14400.
    אם תסובב את הגלגל (כמובן, בניצב לציר הסיבוב) – הזמן הזה לא ישתנה.

  85. ניסים, WD וישראל שפירא,

    כן ברור שבמהירות סיבוב נמוכה האור יכול לצאת ולחזור דרך אותו חריץ, אבל אם מהירות הגלגל גוברת והאור כבר חוזר דרך חריץ אחר, אז התיאור שלי לגבי אופן החישוב הוא דיי מדוייק לא?

  86. לדעתי, פיזו היה יכול לבצע את הניסוי כשהמראה קרובה יותר, וזאת עיי הצבת הגלגל באלכסון, מה שיקטין את רוחב החריצים.

    אך יש לכך גבול בגלל הנפיצה.

  87. לכולם
    אני מבין טוב טוב מה כולם אומרים, ושוב על הרוב אין לי וויכוח.
    ראו סרטון מדהים של בלון סבון ב- 18,000 פריימרים בשנייה.

    https://www.youtube.com/watch?v=ktvZ2Z_s4Bo

    בזמן 00:43 הוא מנפח את בלון הסבון ואחרי שבריר זמן מנפץ.
    לכו לזמן של 01:14 – איזה יופי ובאיזה איטיות רואים ב -18,000 פריימים. כמה מידע חזותי אתה יכול להבחין בכל השלבים של הפיצוץ של הבלון.
    חיזרו שוב לזמן של 00:43 – אתם יכולים לזהות בכלל משהו? רואים את הבלון מתפוצץ וזהו. כלום אתה לא יכול לראות בזמן קצר כל כך.

    עכשיו,
    תסתכלו על כך שפיזו ראה 1 פריים מתוך 18,000 פריימים!
    נראה לכם הגיוני????
    אתם מבינים מה אני שוב אומר – אם היה רואה רק 2 פריימים מתוך ה- 18,000 פריימים, אזי המהירות היא כבר רק 150,000!
    רשמתם על אור שעבר בחריצים 41, 42, 43 – שוב אני מתרגם לכם זאת לפריים 41 מתוך 18,000, פריים 42 מתוך 18,000 וכו. זה הגיוני שמישהו יזהה?
    סורי – עיניים אנושיות לא יכולים להבחין בזה!

  88. יריב
    ישראל הסביר נכון. בהתחלה הגלגל מסתובב לאט, וכמובן שהאור מספיק לצאת ולחזור דרך אותו חריץ. לפיזו כבר הייתה הערכה של מהירות האור מהממצאים של רמר, 180 שנה לפני הניסויים שלו.

    בוא נסתכל רגע על מספרים. טווח של 17,266 מטר זה זמן – כלומר 57.8 מיקרו שנייה (האור נע 300 מטר במיקרושנייה.
    יש 720 שיניים (וחריצים), ואנחנו רוצים שהזמן מחריץ לשן יהיה 57.8 מיקרו. נקבל – סיבוב ב-83.2 מילישניה. זה כמעט עשירית שנייה!
    אז מהירות הסיביב שפיזו היה צריך זה 721סל”ד. זה כלום….. אם כמה גלגלי שיניים פשוטים, קל להגיע למהירות כזו ביד. פיזו השתמש במערכת שבנה איש בשם פרומן, שהיה מאין שעון – ומסוגל להסתובב מאות סיבובים בשנייה.

    כדי לוודא – פיזו הגביר את המהירות יותר, וראה שב-1442 סל”ד, שוב ראו את האור בצורה טובה.

    אפשר לסכם את הדיון הזה: פיזו הוא גאון, מתן ….. לא בדיוק….
    ודרך אגב – לחתוך 720 שיניים בדיסקת פליז זה גם קל מאד.

  89. כמובן, אך בסרטון השיניים תופסות איזה 90%..

    המסקנה – שמספיק חריץ אחד בשביל הניסוי וכל שאר הגלגל הוא שן – ממוטטת לגמרי את טיעון ראיית השן הבודדת.

  90. ישראל

    לגבי סעיף 3. העלמות מוחלטת של האור תלויה בגלגל השיניים. אם השיניים תופסות חמישים אחוז ומעלה מהאזור של השיניים והרווחים, אז תהיה העלמות אם לא רק התעמעמות.

  91. יריב

    הסרטון אכן אינו מדוייק – אך לא מהסיבות אותן הזכרת.

    1. פיזו לא עיין בכתבי מקסוול בשביל עריכת הניסוי – להיפך, מקסוול השתמש בתוצאות ניסוי פיזו במאמר שלו מ1861 שבו הוא הראה שהאור הוא גל אלקטרומגנטי.

    2. המנורה בסרטון היא בצד הנכון, כמו שהיה בניסוי.

    3. לא בכל מהירות סיבוב של הגלגל יראה האור, במהירויות מסויימות הוא יעלם.

    4. בניסוי פיזו המקורי הרוחב של השיניים והחריצים זהה, אך בסרטון השיניים רחבות בהרבה מהחריצים.

    5. אין צורך לאור שעבר בחריץ 41 לחזור דרך חריץ 42. במהירויות סיבוב איטיות הוא חוזר דרך חריץ 41, והאור שעבר דרך 42 חוזר גם הוא דרך 42.

    זה מספיק בשביל מדידת מהירות האור. המעבר דרך החריצים הבאים – אור שעבר דרך 41 חוזר דרך 42, 43, וכן הלאה – רק מאשש את תוצאות המדידה.

    רואים אפוא שבשביל למדוד את מהירות האור מספיק חריץ אחד בלבד שרק רוחבו קובע. המספר הגדול של החריצים הוא כדי להעצים את עוצמת האור והקלה בחישובים.

  92. יריב

    הסרטון אכן אינו משקף נכונה את ניסוי פיזו, אך מסיבות אחרות מאילו שהזכרת:

    1. פיזו לא עיין בכתבי מקסוול לצורך הניסוי – ההיפך הוא הנכון. מקסוול השתמש בתוצאות ניסוי פיזו במאמר מ1861 שבו הוא הסיק שהאור הוא גל אלקטרומגנטי.

    2. המנורה בסרטון היא בצד הנכון, כמו בניסוי.

    3. לא בכל מהירות סיבוב רואים אור, במהירויות מסויימות האור נעלם.

    4. בגלגל המקורי המרחקים בין השיניים והחריצים הוא זהה, שלא כמו בסרטון שבו השיניים גדולות בהרבה מהרווחים.

    5. אין צורך לאור שעבר דרך חריץ 41 לחזור דרך חריץ 42, הוא חוזר דרך אותו החריץ – 41 – והאור שעבר דרך חריץ 42 חוזר גם כן דרך חריץ 42.

    זה מספיק בשביל למדוד את מהירות האור. בפעמים שהאור עובר דרך החריץ הבא – או זה שאחריו – רק מקבלים אישור למה שכבר נמדד.

    רואים אפוא שלמעשה בשביל הניסוי מספיק חריץ אחד בלבד, שמה שחשוב הוא הרוחב שלו. המספר הגדול של החריצים והשיניים הוא רק כדי להעצים את עוצמת האור הנקלט והקלה בחישובים.

  93. מתן,

    הניסוי שרואים בסרטון מבלבל ולא משקף בצורה נכונה את הניסוי של פיזו. הבעיה בסרטון שאור מגיע כל הזמן מכיוון הדף (הדף הוא כמו מקור אור) ולכן אתה רואה אותו כל הזמן בכל מהירות סיבוב של הגלגל, בניסוי של פיזו לעומת זאת מקור האור היה ממוקם בצד השני של הגלגל (יכל להיות אפילו אור שמש שמוחזר ממראה) כלומר בצד שלנו הצופים.

    אני אסביר לך איך אני מבין את הניסוי ואם אני טועה אז יתקנו אותי:

    1. פיזו מסובב את הגלגל במהירות מסויימת (נגיד 1000 סל״ד, סתם זרקתי מספר) ורואה חושך.

    2. הוא מגביר בהדרגה את מהירות הסיבוב של הגלגל עד שפתאום הוא רואה נצנוצים של אור, נגיד ב 1200 סל״ד. כלומר הוא יודע שהאור יצא דרך אחד החריצים כאשר זה היה ממוקם בדיוק מול מקור האור שלו, והצליח לחזור חזרה דרך חריץ אחר, לא משנה איזה, נגיד חריץ 41 אם נקבע שהחריץ דרכו האור יצא הוא חריץ 0.

    3. פיזו ממשיך להגביר בהדרגה את מהירות הסיבוב עד ששוב הוא רואה חושך, כלומר האור החוזר נתקל עכשיו במתכת שנמצאת מיד לאחר החריץ הקודם (חריץ 41) שדרכו קודם האור החוזר הצליח להשתחל ולהגיע לעינו הבוחנת של פיזו.

    4. פיזו מגביר את מהירות הסיבוב בצורה הדרגתית עד שבמהירות 1270 סל״ד הוא שוב מבחין בנצנוצים של אור, כלומר הוא יודע שבמהירות זו היה מעבר של שן אחת בודדת, ושהאור החוזר הצליח להשתחל עכשיו דרך חריץ 42.

    5. פיזו מחשב כמה זמן לוקח לשני חריצים צמודים בגלגל להתחלף במקומם במהירות זו (כלומר כמה זמן עובר מהרגע שחריץ כלשהו נמצא מול הצופה, ועד שהחריץ הבא אחריו נמצא מולו).

    6. עכשיו יש לפיזו מרחק (המרחק בין מקור האור והמראה הרחוקה, כפול 2 כי זה הלוך וחזור) וגם יש לו את זמן התחלופה בין שני חריצים צמודים, חישוב פשוט של דרך חלקי זמן נותן לו מהירות.

  94. מתן

    בדרך כלל אומרים הבוס הוא עדי, אך יפה שכיוונת לדעת גדולים.

    גילוי נאות: ביום כיפורים בבית הכנסת אני שומע הרבה אנשים אומרים לי: שמע, ישראל – אתה ה׳ אלוהינו אלוהים אחד!

    כדי לחסוך את כל הווג׳ראס:

    אתה שואל איך פיזו הבחין בשן אחת בניסוי.

    אז אם תקח מתקן פיזו שבו השיניים כה קטנות שאי אפשר לראותן ללא מיקרוסקופ – אז לא יהיה ניתן למדוד את מהירות האור באמצעות המתקן?

    ואם התשובה היא שניתן – אז מה הרלוונטיות של ראיית שן אחת לניסוי אם ממילא לא ניתן לראותה?

  95. מתן

    טוב, אני מבין שלא הבנת את ההסבר בכלל. אנסה שוב ואשאל שאלה שאלה בשביל לנסות להבין מה לא הבנת.

    נראה לי כרגע שהכי סביר שלא הבנת מה בכלל רואה מבצע הניסוי ועל מה הוא צופה בניסוי. בבקשה תענה על כל השאלות בשביל שנוכל להבין יחד את נקודת אי ההבנה.

    א) כאשר אנחנו נבצע את הניסוי האור או יפגע בגלגל השיניים(בשן כלשהי) ויחסם בכיוון היוצא או יעבור פעם אחת דרך גלגל השיניים(דרך רווח כלשהו) יפגע במראה ויעשה את דרכו חזרה, ואז שוב או יעבור דרך גלגל השיניים או יפגע בגלגל השיניים ויחסם בכיוון הנכנס?
    1) נכון?
    2) מובן?

    ב) במהירות נמוכה יש אור שנחסם בדרך החוצה והאור שיוצא ולא נחסם יספיק לחזור דרך אותו רווח?
    1)נכון?
    2) מובן?

    ג) אם השיניים והרווחים זהים בגודלם חצי מהאור תמיד יחסם וחצי יצא בדרך החוצה?
    1) נכון?
    2) מובן?

    ד) כשנגביר את המהירות, בנקודה מסויימת חלק מהאור הזה שיוצא ולא נחסם בדרך החוצה, יחסם ע”י גלגל השיניים בדרך חזרה.
    1) נכון?
    2) מובן?

    ה) החל ממהירות סיבוב מאוד נמוכה צופה שמביט בגלגל השיניים לא מסוגל להבחין בכלל באף שן.
    1)נכון?
    2) מה הוא כן רואה כאשר הוא מביט על הנקודה בגלגל השיניים דרכה עובר האור?

  96. WD
    תאמין לי שאני חד ועקבי בכל מה שאומר.
    אמרתי זאת כבר רבות, אין לי אף בעיה עם המטודה, ואמרתי זאת רבות, המספרים מסתדרים מעולה! אני מבין שהוא מעלה כל הזמן את המהירות של הגלגל. גם לזה אין לי טענה.
    התהייה שלי בנוגע לניסוי הזה היא איך הוא הבחין בעין אנושית ב-1 שן (מתוך 18,744 שיניים שעברו בשנייה) או בזמן של 0.000053.
    זה המנטרה שלי מההודעה הראשונה ועד עכשיו. ישראל שפירא הוא עדי.

    ואתה – אתה הולך סחור סחור. אתה עו”ד במקרה?
    בהודעה הראשונה לך ב-1 למרץ:
    “מכאן שכל מה שצריך הוא לעקוב אחר ההתעמעמות והבוהק של האור ואין צורך להבחין בשיניים כלל”

    ועכשיו אתה משפץ בהודעה ב-2 למרץ:
    ” מה שאפשר לו להבחין הודות להתעמעמות של האור בעקבות החסימה של האור בדרך חזרה ע”י השיניים, מתי היה מדובר במעבר של שן אחת”

    כלומר,
    WD טוען שהוא כן הבחין ב-1 שן.

    עכשיו לענייננו,

    הטענה היחידה שלי שהוא לא יכל להבחין ב-1 שן בעין אנושית. ושוב, אם זה היה בטעות 2 שיניים, אז המהירות רק 150,000 ק”מ לשנייה.

    בוא ננתח את הסרטון ששלחת לי (שאותו אני מכיר כמובן) – אני לא מנתח מה הבחור אומר, אלא רק את הסיבוב של השיניים:
    בזמן 02:00 רואים את הגלגל – 12 שיניים
    בזמן 02:08 רואים שהגלגל מסתובב. אני לא יודע להאריך כמה סיבובים בשנייה אבל נניח ואני לארג’ איתך 10 סיבובים בשנייה (אין סיכוי אבל אני לארג’ איתך)
    כלומר אנחנו מדברים על 120 שיניים שעוברות בשנייה.

    עכשיו, לך לזמן של 02:08-02:09 (ב02:10 זה כבר מאט) –
    תתמקד בחץ הירוק – האם אתה יכול להגיד לי כמה שיניים עוברות כל הבהוב מעל החץ הירוק? (כלומר בזמן של 0.1 שנייה (אמרנו שיש 10 סיבובים בשנייה))?
    רואים את החץ הירוק, לא אמרתי שלא, האם אתה יודע להגיד לי כמה שיניים עברו בהבהוב אחד? בסיבוב אחד? זה נטו ניחושים!!! ניחושים!!
    כלומר, ב-120 שיניים ל-1 שנייה קשה לך להגיד כמה שיניים עברו מעל החץ הירוק בכל הבהוב או בכל סיבוב. בסרטון כל שן עוברת זמן של 0.008 השנייה.

    עכשיו, תחלק 18,744 שיניים ב-120 שיניים – יוצא 156.2.
    כלומר, פיזו ראה את המציאות הזאת פי 156 יותר מהר!!!!
    אז אתה רוצה להגיד לי שאנחנו עם הסרטון הזה בקושי יכולים לקבוע כמה שיניים (כל שן עוברת 0.10 שניות.) , ופיזו ראה בעינו האנושית פי 156 שיניים והבחין בשינוי/הבדל של 1 שן בזמן של 0.000053 שניות?
    מחדד שוב בפעם המיליון – אצל פיזו ב-1 חור הוא רואה, וב-1 שן הוא לא רואה, ב-1 חור הוא רואה. (אני רושם זאת בשפה של ילד בן 10 שיבין ולא את הפלפולים שלך על “להבחין הודות להתעמעמות של האור בעקבות החסימה של האור בדרך חזרה ע”י השיניים, מתי היה מדובר במעבר של שן אחת”)
    אוקי? רושם שוב – 1 חור הוא רואה, וב-1 שן הוא לא רואה, ב-1 חור הוא רואה.
    אז אתה רוצה להגיד לי שהוא יכל בסוף התהליך (להגביר לאט לאט את המהירות של הגלגל) עד שהבחין בעינו ב-1 שן? הראיתי קודם בסרטון שצירפת שאתה או ניסים או ישראל או כל בן אדם אחר בקושי יכול להבחין כמה שיניים עוברות – בסרטון כל 1 שן עוברת זמן של 0.008 השנייה.
    אז אתה רוצה להגיד לי שפיזו הבחין בפי 156 יותר מהר? בזמן של 1 שן בזמן של 0.000053?

    חחח – ואני המשוגע. אין סיכוי שבעולם!

  97. ניסים

    Baby steps.

    מקסימום במקרה הכי גרוע אולי עזרנו למישהו אחר שכן מסוגל להסתכל מעבר לבור הבורות והטפשות הזה ששוטף חלקים נרחבים מהעולם.

  98. WD
    מתן הסביר שסיבוב מישור המטוטלת ובע מזה שהיא תלויה על כבל גמיש. ראיתי מספר מטולטלות כאלה (בסמית’סוניון ב-DC ובכנסיה בבולוניה, ב UNSW באוסטרליה – ושניים בישראל). כולם תלוים על כבל פלדה. יפה לראות שזה בסידני מסתובב הפוך 🙂 (אין לו כל מנגנון מכני, והוא מתנדד כמה שעות כל פעם – ותמיד 7 מעלות לשנייה).

    הסברתי לו שכשטסים גבוה קו האופק מנמיך, בצורה מדודה שאינה קשורה לראות. הוא טוען שהסיבה זה אובך.

    הסברתי לו שמעליה על מאונה קיה ניתן לראות את הר הגעש על מאווי (הליאקאלה) – ורואים בבירור את הפסגה אך לא את הבסיס.

    הסברתי לו שמגדלי גשר הזהב הם מאונכים – אבל לא מקבילים. הוא מתעלם.

    הסברתי לו שקולטים GPS בלב ים, אך הוא מתעקש שהמשטרה מאכנת לפי אנטנות סלולריות (לא שייך וגם בד”כ לא נכון).

    הסברתי לו שאי-אפשר לנבא ליקויי חמה בלי להניח שהעולם עגול – הוא מתעלם.

    הסברתי לו על הסרטון של הטסלה.

    מתן טוען שסטיבן הוקינג מבלף ואינו חולה. הוא טוען שתאונות הצ’לנגר והקולומביה לא קראו.

    אתה באמת חושב שניסוי פיזו קשור לעניין? כל מכ”ם מתבסס על הערך של מהירות האור – הבעיה שזה סותר את ה”אמונה” שלו. לכן – כל מי שטוען שמהירות האור הוא c, בעולמו המעוות של מתן, הוא שקרן.

    יש פה משוגעים נחמדים באתר, אז למה לדון עם אחד מרושע ומעוות?

  99. WD
    מתן הסביר שסיבוב מישור המטוטלת ובע מזה שהיא תלויה על כבל גמיש. ראיתי מספר מטולטלות כאלה (בסמית’סוניון ב-DC ובכנסיה בבולוניה, ב UNSW באוסטרליה – ושניים בישראל). כולם תלוים על כבל פלדה. יפה לראות שזה בסידני מסתובב הפוך 🙂 (אין לו כל מנגנון מכני, והוא מתנדד כמה שעות כל פעם – ותמיד 7 מעלות לשנייה).

    הסברתי לו שכשטסים גבוה קו האופק מנמיך, בצורה מדודה שאינה קשורה לראות. הוא טוען שהסיבה זה אובך.

    הסברתי לו שמעליה על מאונה קיה ניתן לראות את הר הגעש על מאווי (הליאקאלה) – ורואים בבירור את הפסגה אך לא את הבסיס.

    הסברתי לו שמגדלי גשר הזהב הם מאונכים – אבל לא מקבילים. הוא מתעלם.

    הסברתי לו שקולטים GPS בלב ים, אך הוא מתעקש שהמשטרה מאכנת לפי אנטנות סלולריות (לא שייך וגם בד”כ לא נכון).

    הסברתי לו שאי-אפשר לנבא ליקויי חמה בלי להניח שהעולם עגול – הוא מתעלם.

    הסברתי לו על הסרטון של הטסלה.

    מתן טוען שסטיבן הוקינג מבלף ואינו חולה. הוא טוען שתאונות הצ’לנגר והקולומביה לא קראו.

    אתה באמת חושב שניסוי פיזו קשור לעניין? כל מכ”ם מתבסס על הערך של מהירות האור – הבעיה שזה סותר את ה”אמונה” שלו. לכן – כל מי שטוען שמהירות האור הוא c, בעולמו המעוות של מתן, הוא שקרן.

    יש פה משוגעים נחמדים באתר, אז למה לדון עם אחד מרושע ומעוות?

  100. ניסים

    יכול להיות, אבל נראה לי שעדיין כדאי לנסות לעזור לאנשים להבין במקום שנראה שהם לוקים בהבנה ושתיקון פשוט של שגיאה או מיסקונספציה יכול לשנות את זה. בגלל זה פיניתי ספציפית לניסוי של פיזו ולמטוטלת של פוקו כיוון שנראה ששם באמת הוא הבין לא נכון את העקרון של הניסוי.

  101. עוד נקודה למי שחושב שכל משימות החלל הם זיוף ושום חללית או לווין לא עזבו את כדור הארץ:

    ברית המועצות ורוסיה שלחו למאדים 19 חלליות במהלך השנים, בהם חלליות מתקדמות ושאפתניות מאוד לזמנם, הכולללות מקפות נחתות רוברים ועוד, כולם עד האחרון שבהם כשלו במשימתם, חלקם התפוצצו בשיגור, חלקם לא הצליחו לעזוב את המסלול סביב כדור הארץ או שאבדו בחלל, וגם אלה שהצליחו להגיע למאדים התרסקו עליו או שהקשר איתם ניתק זמן קצר מאוד אחרי הנחיתה. (ארה”ב הצליחה גם הצליחה ושיגרה לא מעט חלליות למאדים, היום אגב סובבות סביבו 6 מקפות אם אני זוכר נכון ועוד 2 רוברים משוטטים להם על פני השטח)

    אם כל החלליות הם המצאה וגרפיקה למה רוסיה לא זייפה גם בדרך הזאת??? זו הרי מכה אנושה לכבוד הלאומי שלה, 19 כשלונות ואף הצלחה אחת!!! איך היא הסכימה להודות בזה? כולם מזייפים ורק היא לא???

  102. WD
    להיפך! רציתי להראות שהנתונים של ניסוי פיזו הם לא חריגים.

    הבעיה של מתן שהוא שקרן פתולוגי. הוא יספר לך שכשמגדילים תמונה של אוניה מעבר לאופק – רואים את כל האוניה.

    והוא עוד יותר טיפש….. אני לא מוצא עניין בדיון עם שקרנים שהם טיפשים

  103. מתן

    קודם כל לא צריך לשחק במשחקי נדמה לי, המספרים של פיזו בניסוי היו:

    Distance travelled by light = 2d
    Transit time = 1/2nN
    Speed of light (c) = 4nNd

    Fizeau’s values were:
    2d = 17.26 km, N = 720, n = 12.6 revs per second, giving a value of 3.13 x 108 ms-1 for the speed of light.

    שנית אני לא בטוח שאני מסוגל להסביר את זה יותר טוב משעשיתי בתגובה שלי(או אם בכלל מישהו אחר יכול) ובאמת קשה לי להבין אם קראת ובאמת ניסית להבין את התגובה שלי בהתחשב בזה שיצאת ממנה בזה שהתאור מדהים ומשם קפצת לכל מיני נקודות שלא ממש קשורות אליה ורומזות שלא הבנת אותה.

    בכל זאת אנסה להתייחס לתגובה שלך. ואנסה לעזור לך להבין שוב.

    ‘בטח שחשוב מספר השיניים’

    מי אמר שלא?

    מספר השיניים שעוברות בזמן שהאור עושה את דרכו למראה וחזרה חשוב ביותר לחישוב. בדיוק בגלל זה כל כך חשוב היה לוודא שמדובר במעבר של שן אחת בדיוק. (כמובן שבהמשך ניתן לצפות ולעשות את החישוב על מעברים נוספים אבל זה פחות חשוב להכנס לזה)

    ‘מה זה ” לעקוב אחר ההתעמעמות והבוהק”? מה זה לעקוב?’

    לעקוב במקרה הזה זה לצפות עם העין. זה מה שהיה לו בזמנו, וזה בהחלט פגע ביכולת הדיוק שלו.

    אם תביט כאן –

    https://youtu.be/h_UMabvPrws?t=119

    תראה שכאשר אתה מביט דרך החלק המשונן כאשר הוא מסובב אותו אתה רואה את הדף מאחור במעומעם. (הסרטון הסברה הזה בכללי לוקה באי אלו אי דיוקים אז הוא קצת מעצבן אבל הוא בסדר קונספטואלית)

    ‘אז איך אתה אומר לי ש –
    “מכאן שכל מה שצריך הוא לעקוב אחר ההתעמעמות והבוהק של האור ואין צורך להבחין בשיניים כלל”‘

    כי אם היית קורא בשקיקה את התגובה שלי היית אמור להבין ממנה שהוא דאג שהמדידה תהיה של מעבר שן אחת וכאן השאלה הבאה שלך היא אכן מאוד נכונה וחשובה.

    ‘איך פיזו הוכיח שעברה 1 שן?’

    זו בעצם הייתה כל המטרה של התגובה שלי. להבהיר את הנקודה הזאת. וכמו שכתבתי מקודם התשובה היא שהוא העלה את המהירות בהדרגה מה שאפשר לו להבחין הודות להתעמעמות של האור בעקבות החסימה של האור בדרך חזרה ע”י השיניים, מתי היה מדובר במעבר של שן אחת. ההעלאה ההדרגתית ממהירות למהירות מהירה יותר וכך הלאה היא שאיפשרה את וידוא העניין שמדובר במעבר של שן אחת ולא פתאום שתיים או שלוש.

    עכשיו בבקשה תנסה לקרוא את התגובה הקודמת שלי עוד פעם בנסיון אמיתי להבין אותה ולא בנסיון לרפרף מעליה ולחזור על שאלות של בעיות שלא קיימות. אם אתה לא מבין לפחות תנסה לעזור לי להבין מה אתה לא מבין בהסבר ע”י שאלת שאלות רלוונטיות ולא ע”י ניסיון להציג בפני את מה שאתה חושב שהבעיה בניסוי.

  104. מתן

    אפשר למדוד את מהירות האור באמצעות ניסוי פיזו אפילו אם השניים כל כך קטנות שאי אפשר לראות אותן בלי מיקרוסקופ כשהגלגל במנוחה, שלא לדבר על תנועה.

  105. WD
    אני עדיין פה…..
    שתבין, התיאור שרשמת מדהים, והמספרים המתמטיים של הניסוי הזה גם מדהימים ומסתדרים מעולה!
    אבל איפה הבעיה ה”ממש-קטנה”?
    בטח שחשוב מספר השיניים!
    1 שן שעברה שווה למהירות של 313,000 ק”מ בשנייה.
    2 שיניים שעברו שווים למהירות של 150,000 ק”מ בשנייה!! (בקירוב)
    3 שיניים שעברו שווים למהירות של 100,000 ק”מ בשנייה!! (בקירוב)

    המהירות שהוא קבע לפי החישובים המתמטיים, 313,000 ק”מ בשנייה, גם לפי החישובים אמור לעבור רק 1 שן!
    בוא אראה לך את המתמטיקה ה”מדהימה” שמסתדרת:

    שלב א:
    בוא נחשב כמה גלגלי שיניים עוברים בשנייה:
    נתון 720 שיניים
    נתון 1562 RPM
    סיבובים לשנייה
    60 / 1562 = 26 סיבובים לשנייה

    26*720 = 18,744 שיניים עברו בשנייה אחת!

    שלב ב:
    הוא הגיע למהירות של 313,000 ק”מ בשנייה על בסיס מה שרשמת לכאורה.
    בוא נחשב כמה זמן עבר:
    V= S / T
    T= 16/313,000 = =בקירוב 0.000053 sec
    שלב ג:
    אמרנו שעוברים 18,744 שיניים בשנייה.
    בוא נראה כמה גלגלי שיניים עוברים בזמן של 0.000053 שניות:
    18,744*0.000053
    0.993 = בקירוב = 1 שן!

    שלב ד:
    בוא נגיד שעברו 2 שיניים (הכוונה האור עבר בחור, עברו 2 שיניים, והאור חזר בחור).
    נעשה חשבון מחדש:
    18,744*0.000106
    = 2 שיניים!
    נחזור למהירות ואז נקבל שהמהירות היא
    V= S / T
    V=16/0.000106
    V= 150,000 km/sec (בקירוב)
    כלומר, אם עברו 2 שיניים אזי המהירות היא חצי – 150,000 ק”מ בשנייה

    אז איך אתה אומר לי ש –
    “מכאן שכל מה שצריך הוא לעקוב אחר ההתעמעמות והבוהק של האור ואין צורך להבחין בשיניים כלל”
    מה זה ” לעקוב אחר ההתעמעמות והבוהק”? מה זה לעקוב?
    מה המדיד האמפירי שהוא מדד?
    והראיתי לך במתמטיקה שעברה 1 שן. איך פיזו הוכיח שעברה 1 שן?

    כל כל הניסוי הזה המשתנה המרכזי הוא השיניים – מספר השיניים שעוברות משתנה – המהירות משתנה.

    לפיכך, בטח שהוא צריך להבחין בשיניים!!!! הרי הוכחתי לך למעלה – כל שן שמתווספת המהירות קטנה!

    מתן

  106. ניסים

    ניסיתי להסביר למתן מדוע אין צורך להבחין בשן מסויימת שנעה במהירות מאוד גבוהה בניסוי כמו שהוא חושב(או לפחות נראה שהוא חושב).

    הניסוי מבריק לחלוטין ומעשי לגמרי .

    לא ברור לי אם ולמה התרשמת שהתכוונתי לומר שהוא אינו מעשי.

  107. תאורטית, במהירות סיבוב מסויימת האור יעלם לגמרי וזה אכן מה שקורה אם השיניים גדולות יותר מהרווחים בשיעור מסויים.

    אך בגלל העקיפה, בניסוי פיזו האוריגינל שבו גודל השיניים והרווחים זהה פחות או יותר, תמיד קיימת דליפה של אור, אך זה עדיין לא מונע חישוב מדוייק למדי של מהירות האור גם באמצעים פשוטים.

    מה שניסיתי לבדוק בניסוי אותו ערכתי לפני כמה שנים היה את הרעיון על פיו האור נע ביותר ממהירות אחת. לא אכנס לפרטים, אך בגלל אותה עקיפה ואפקטים קוונטים לא הצלחתי לקבל ״סלאם דאנק״ לכאן או לכאן (קיבלתי סלאם דאנק כמעט מושלם בניסוי אחר, די מעניין).

    באותה וריאציה של ניסוי פיזו, ניצלתי בנס כאשר גלגל השיניים אותו סובבתי במהירות של כ20,000 סל״ד, פשוט נחתך לשניים בגלל הכוח הצנטריפוגלי, וחציו שחלף כחצי מטר ממני חתך את דלת המתכת הסמוכה. ארוע מפחיד, אך מה לא עושים למען המדע?

  108. WD
    אם מתחילים לסובב את גלגל השיניים לאא, ומגבירים עוד ועוד, עוצמת האור תרד למינימום ותתחיל לעלות שוב עד למקסימום, ואחרי זה שוב יירד למינימום וכן הלאה.

    בוא ניקח 50 סיבובים בשניה ו-100 שיניים. הזמן מחור לשן הוא 100 מיקרו שנייה. זה זמן הלוך חזור, והאור נע 150 מטר בכל מיקרו (הלוך, חזור). כלומר – טווח של בסה”כ 15 ק”מ עבור הדוגמה שנתתי.

    לכן – הניסוי של פיזו הגאון הוא מאד מעשי.

  109. מתן

    מצטער על האיחור בתגובה ומקווה שתקרא אותה.

    מכיוון שאף אחד לא נתן לך הסבר לעניין של פיזו אז אני אעשה מאמץ להסביר בצורה פשוטה.

    פיזו לא היה צריך לראות או לספור אף שן.

    איך הוא ידע שהאור עובר דרך שן אחת בכיוון היוצא וחוזר מהזאת הבאה בדרך חזרה?

    העקרון הוא כזה.
    הוא התחיל לסובב את גלגל השיניים במהירות איטית ועם הזמן הגביר מהירות. הדבר אחריו הוא עקב הוא עוצמת האור שהוא ראה בחזרה. (מדובר באור שמעומעם ע”י הגלגל שיניים כיוון שחלק מהזמן האור עובר(כאשר הוא בין השיניים) וחלק לא(כאשר האלומה היוצאת פוגעת בשיניים), ובשום שלב לא מדובר כאן על חוסר אור מוחלט או על תאורה שעוצמתה כעוצמת המקור ללא הפרעה)

    כל עוד המהירות לא גבוהה מספיק האור מספיק לצאת ולחזור דרך אותו מרווח בין השיניים.

    כאשר המהירות גבוהה מספיק חלק מהאור היוצא לא מצליח לחזור חזרה כי בדרך חזרה הוא נתקל שן. התוצאה של זה היא שהאור נעשה מעומעם יותר. למה?

    כמות האור היוצאת דרך הרווחים בין השיניים נשארת זהה. או שהאור פוגע בשן ולא חוזר או שהוא עובר במרווחים.

    אז נשאלת השאלה מה קורה עם האור שיצא.
    הוא יכול או להתקע בשן בדרך חזרה או לחזור דרך מרווח.
    אם הוא עובר דרך מרווח אנחנו שנארים עם אותה עמימות של התאורה.
    אם הוא נתקל בשן בדרך חזרה אנחנו מקבלים פחות אור שמצליח לחזור מה שמתבטא בתאורה עמומה יותר.

    מכיוון שהתחלנו עם מהירות נמוכה והגברנו ואנחנו יודעים שבהתחלה האור שחוזר עושה זאת דרך אותו מרווח ממנו יצא ושכאשר האור יחזור להיות בוהק יותר כמו מקודם זה בגלל שהוא עובר חזרה דרך המרווח הסמוך(לפיכך מעבר לשן אחת).

    איך אנחנו יודעים שמדובר במעבר של שן אחת בלבד ולא מספר שיניים בגלגל שיניים?

    אין מדובר במעבר של מספר שיניים כיוון שהאור מהיר מדי בשביל שזה יהיה אפשרי בשלב זה. (כמובן שכשנגביר מהירות בהרבה בהחלט אפשר שהאור יצא ויחזור תוך כדי מעבר של מספר שיניים)

    מכאן שכל מה שצריך הוא לעקוב אחר ההתעמעמות והבוהק של האור ואין צורך להבחין בשיניים כלל.

    ומשם זה הכל חישובים שתלויים ברדיוס הגלגל שיניים, מהירות הסיבוב שלו והמרחק שהאור עובר.

    מקווה שזה היה ברור.

    בהסבר קצר יותר על המטוטלת של פוקו, שנראה לי שהחמצת לגמרי את כל הקטע שם. זה לא משנה למה היא ממשיכה לזוז, הפואנטה היא שהמטוטלת ממשיכה כל הזמן לנוע באותו כיוון(ע”ע תנועת מטוטלת) הלוך ושוב ישר. התנועה הסיבובית שאנחנו במבחינים בה היא התנועה של כדור הארץ.

  110. מתן
    קבוצות הזודיאק השתנו בחודש בערך מתקופת הרומאים. היום, אם נולדת במזל טלה, אז השמש נימצאת דווקא במזל דגים.

    אז הנה סתרנו עוד שקר שלך. אני אקפיד לסתור שקר אחד כל תגובה, ולא אתייחס לשאר השקרים – עד שתענה לי על השאלה שלי.

    אשאל שוב כדי שלא תוכל לשקר שלא ראית את השאלה: התוכנה שלי מניחה שהעולם עגול ומסתובב סביב השמש. אני יכול לחשב את המיקום של כל גוף נראה בשמיים, בכל שעה, בכל מקום בעולם, בכל תאריך – בדיוק של אלפיות מעלה. איך זה ייתכן אם אני כל כך טועה?

  111. בדקתי את הנושא של השתנות השמיים הצפויה בעקבות הנקיפה, עקבות תנועת השמש סביב מרכז הגלקסיה וכן בשל התנועות העצמיות של הכוכבים וזה הולך ככה: הנקיפה לא אמורה להשפיע על בכלל על מישור המילקה, זאת אומרת אותם 12 מזלות שבהם עוברים השמש הירח וכוכבי הלכת לא ישתנו בגלל הנקיפה, מכיוון שמסלול כדור הארץ סביב השמש לא משתנה אלא כיוון הסחרור של כדורינו

    מה שכן, הנקיפה גורמת זה להשתנות של נקודות השוויון במעלה אחת כל 72 שנה ואת זה בהחלט רואים.

    אנחנו מסתובבים סביב מרכז הגלקסיה אבל כך גם כל שאר הכוכבים ולכן לא נראה שינוי בשמיים בשל כך.

    אמנם לכוכבים יש תנועות עצמיות בכיוונים שונים אבל התנועה הזו קטנה עד מאוד, הכוכב עם התנועה הצידית המהירה ביותר שנמצא עד היום הוא כוכב ברנארד ואפילו הוא יזוז רק כ-5 מעלות ב-2,000 שנה (כוכב ברנארד הוא במגניטודה 9.5 דהיינו חלש מלהיראות לעין בלתי מצויידת) , כל שאר הכוכבים נעים לאט אף יותר.

  112. למתן

    לא הבנתי מה הבעיה שלך עם ליקוי הירח האמור, הליקוי מתחיל בכיוון התנועה של הירח, בדיוק איפה שהוא אמור לפגוש את הצל של כדור הארץ, וגם הצורה שלו בסדר גמור כמו 2 עיגולים חופפים בחלקם.

    אני מחכה לתשובה איך אתה יכול להסביר את היעלמות חלק מספינה או מגדל באופק לפי מודל העולם השטוח. וכמו שהסברת לך בתגובה הקודמת, מה שאתה קורא לו חוק ההיעלמות נובע כל כולו מכדוריות הארץ. או במילים אחרות: בעולם שטוח אם אני רק צופה מעל גובה הגלים, ובאמצעות טלסקופ, איזו סיבה בעולם יש שחלק ממגדל רחוק יוסתר על יד המים אם אני רואה בלי בעיה את החלק העליון שלו? חלק בן עשרות מטרים !!

    לגבי השמש, נתת דוגמא של פנס באיצטדיון בגובה 2 מטר מהקרקע שיאיר רק את האזור מתחתיו, יש עם הדוגמא הזו 2 בעיות א. פנס איצטיון מכוון להאיר אזור מוגבל, תחשוב על הצורה של מנורה שולחנית, השמש לעומת זאת דומה יותר לנורת אגס – היא כדורית ומאירה לכלכ הכיוונים, אם היתה נורת אגס בקצה האיצטדיון היו רואים אותה בלי בעיה מהקצה השני.

    ב. 2 מטר הם סדר גודל של מאית מאורך האיצטדיון כולו, השמש נמצאת אפילו לפי דעתך בגובה שהוא די דומה לאורך כל העולם.

  113. יריב – בכוונה בחרתי מישהו שלא קשור לעולם השטוח שצילם את הכוכבים הקרובים.
    זה שהוא טוען שבסוף עבר לויין ולא באמת רואים משהו – אמחייה.

  114. ניסים

    הבבלים לא האמינו בעולם הליוצנטרי שעוד לא הומצא בכלל.

    מתוך מאמר מהידען מלפני 13 שנים:
    https://www.hayadan.org.il/babylonastronomy-220305

    1.
    “לא ידוע מתי , אך הבבלים גילו גם את מחזורי הסארוס (Saros Cycles), החלים במרווחי זמן של כ-6585 ימים, שהם מחזורים בהם חוזר הירח כמעט לאותה עמדה ביחס לשמש. הזמן הנ”ל שווה לכפולה של 223 חודשים סינודים או 18 שנים ו-11.3 ימים. בעזרת הסארוס ניתן לחשב ליקויי שמש וליקוי ירח באופן מדויק, והם באמת חזו ליקויים”.
    והנה ניסים, רואים שהם יודעים לחזות ליקויי שמש וירח ללא התוכנות המדהימות שלך.
    גם היום אתה יכול לחזות ליקויי שמש וירח על בסיס נתונים שידועים לנו כבר אלפי שנים.

    2.
    “על-פי “לוחות נוגה” של עמצדק מהמאה ה-17 לפנה”ס, ניתן לראות שנערכו תצפיות שמיים כבר בתקופת בבל הקדומה, כולל רישומים רבים שעשו הבבלים במשך מאות שנים, תיעדו את תצפיותיהם ביומנים מיוחדים, וקטלגו כוכבים באלמנכים שלמים. בזכות הרישומים המדויקים והתצפיות התאפשרו כל החישובים המדויקים הללו. ביומנים תיעדו הבבלים יום אחר יום ורשמו נתונים לגבי מיקומי הירח, כוכבי הלכת, ליקויים, היפוכים ושיוויונים, ולעיתים אף תיעדו את זמני זריחת סיריוס (כנראה תוך קשר לתרבות המצרית, בה היווה סיריוס את הכוכב החשוב ביותר).”

    “מכאן אנו למדים שגלגל המזלות של ימינו, הומצא כבר אז על ידי הבבלים.”

    גלגל המזלות לא השתנה כבר כמה אלפי שנים – רואים זאת ביומנים שלהם – העולם ההליוצנטרי שלך קובע שזה זז 60 מעלות – אבל ביומנים שלהם המזלות אותן מזלות!
    יש לך הוכחה שלפני 4000 שנה הבבליים אמרו שהמזלות נמצאים במקום אחר? אשמח לראות. מחכה.

    3.
    “הבבלים ידעו להבדיל בין הפלנטות לכוכבים, ושמו לב לתנועתן הלא אחידה בשמיים ביחס לשאר הכוכבים”.
    גם בעולם שטוח יש הבדל בין הכוכבים הקרובים לנו – חמה, מאדים, נוגה וכו’ בין הכוכבים הרחוקים.

    אין קשר להשפעה של השמש. זה פשוט סוג אחר של כוכבים.

    ויריב – ברור שהכוכבים הקרובים אלינו נראים אחרת מאשר הכוכבים הרחוקים המנצנצים.
    הנה – ראה מצלמה P900 איזה יופי צילמה את כוכבי הלכת הקרובים:
    https://www.youtube.com/watch?v=BdKd20pP8Cc

    מתן

  115. פלוני 123
    הכוכבים בשמיים אכן זזים, בגלל מספר סיבות. הסיבה העיקרית היא הנקיפה של כדור הארץ. ציר כדור הארץ נוטה על צידו כ 23.5 מעלות. הציר הזה נמצא בתנועה, והיום פונה בערך לכיוון כוכב הצפון. הציר נע כמו ציר של סביבון שמעט, ומשלים סיבוב כל 26 אלף שנים. כלומר, בתקופת הבבלים, הציר היה בערך 60 מעלות מכיוונו היום. מתן כמובן לא ידע להסביר את זה.

    בנוסף, יש השפעה למיקום שלנו סביב השמש, השפעה קטנה, ורק על הכוכבים הקרובים ביותר. כמובן שגם את זה מתן לא יודע להסביר.

    בנוסף, לחלק מהכוכבים רכיב תנועה שמשפיעה על המיקום של כוכבים מסויימים. לזה מתן מתכחש לחלוטין.

    מתן לא מסוגל להסביר כלום… או לנבא כלום. שאל אותו מתי ליקוי החמה הבא…

  116. לפלוני 321
    https://www.youtube.com/watch?v=VW2xRR75lKE
    סרטון של נשיונל ג’יאוגרפיק המסביר על סוגי ליקויי הירח
    ראה דקה 01:56 – המסביר על ליקוי חלקי
    “Appears very dark on the side of the moon facing earth”

    facing earth

    וראה גם את האנימציה – הירח נכנס לתוך הצל החלקי שכדור הארץ עושה מהשמש, וכדור הארץ עצמו עושה צל קמור על הירח.

    מה שהראיתי לפני מספר שבועות היה צל בחלק העליון של הירח וגם קעור! – כלומר גם צורה הפוכה.
    פיזית אין מצב שכדור הארץ עושה צל הפוך שכזה – גם למעלה וגם קעור. פיזית זה לא מסתדר.

    מתן

  117. למתן, היקום, וכל השאר

    למה אתה חושב שבליקוי ירח הוא אמור להתכסות מכיוון מטה? ככל הידוע לי אין שום עדיפות בכיוון, בנוסף, כיוון הירח מתהפך בכל לילה בין החצי הראשון לשני וכן בחלק מסוגי הטלסקופים התמונה מתהפכת.

    מפות הכוכבים המפורטות העתיקות ביותר שבידינו הם בנות כ-2,000 שנה אני לא מתמצא מספיק כדי לדעת האם או בכמה הם שונות מהמצב כיום אבל אין לי ספק שאסטרונומים עוסקים בכך.

  118. יריב
    וואוו … השמש במרחק 6000 מייל. אך איך המטומטמים מסבירים טיסות של 15 שעות בקו ישר?

    האמת שהסרטון נורא מצחיק.

  119. מתן
    תתיחס להוכחה שלי. מצידי שלא יהיה אף לווין או אף טיל – זה לא משנה את מה שאמרתי. שכל התמונות מהחלל יהיו מזוייפוץ מצידי – זה עדיין לא משנה בכלום אתה מה שאני אומר. פיזו שקרן, איינשטיין שקרן, אילן רמון חי וקיים, סטיבן הוקינג מבלף – למי אכפת?

    אנחנו לא בתחרות כאן. תסביר לי מה לא נכון בהוכחה שלי.

    לא אתייחס לאף מילה שלך לפני זה.

  120. מתן,

    אני חושב שקיבלת תשובות על רוב הטענות שלך, אני לא מתכוון להמשיך לבזבז את זמני על הדיון המגוחך הזה.

  121. יריב
    חשבתי על ההוכחה שלי (עולם עגול). נראה לי שאי אפשר לסתור אותה. כל אחד יכול להסתכל בתוכנה ולהבין שרק עולם עגול יכול להתנהג ככה.

    קראתי גם עוד הסבר נחמד. יש נקודה בשמיים שכל הכוכבים מסתובבים סביבה, נכון? ליד זה יש את כוכב הצפון.
    איך יכול להיות שבאוסטרליה (דרום אמריקה, דרום אפריקה…) כיוון הסיבוב הוא הפוך?

  122. ניסים – תראה עד כמה זה מגוחך – ראית מי צילם את התמונה הזאת של תחנת החלל?
    Joel Kowsky – צלם ועורך תמונות בנאסא…….
    https://twitter.com/jakowsky

    ניסים ויריב – אני עדיין מחכה שתענו לי:
    מה גרם לטסלה לזוז?
    למה היה fade-away של האסטרונאוט ועוד פעלולי מחשב בתחנת החלל?
    למה אמרו שטיל אטלס וטיל ספייס איקס אירודיום 4 עמדו במשימתם שרואים שהטילים פשוט התפוצצו?
    איך פיזו הבחין בעינו ב-1 שן או בזמן של 0.000053 שניות (ישראל שפירא זה גם בשבילך)?
    למה בליקוי ירח חלקי לפני שבועיים ראו את הצל של כדור הארץ בחלק העליון של הירח?
    איך זזנו 3 שנות אור ב-4000 שנים האחרונות, וגם שאר הכוכבים ביקום זזו להם שנות אור לכיוונים אחרים, אבל עדיין המזלות נראים אותו דבר בדיוק כמו שמדדו לפני 4000 שנה?

    אחרי זה אענה על הדברים שרשמתם אתמול

    בברכת שבת שלום

    מתן

  123. מתן, אני שמח שאתה לוקח את הדברים ברצינות ויישמת את המלצתי לגבי המספור.

    ראשית בקשר להקדמה שלך המדמה את המודל ההליוצנטרי לשורת מפסקים שאם אחד מהם נופל אין חשמל ולא משנה ש-99 כן פעילים, זה כל כך לא נכון, זה בדיוק מה שניסיתי להסביר לך בתגובה הקודמת, כשאתה רוצה לדון האם האמת כתאוריית הארץ השטוחה אם לאו או בכל נושא אחר, אתה צריך להסתכל על כלל הראיות, ולזכור שאתה לא יודע הכל ואתה כמו כל אחד אחר מועד לטעויות ולכן גם אם אתה חושב שמצאת ראיה או 2 או 8 לתאוריה שלך אתה מוכרח לקחת בחשבון את שאר הראיות, ואת המשקל ומידת הביטחון שלך בהם, בדיון שלפנינו הראיות לכדוריות הארץ ולקיומם של לווינים הם כ”כ רבות וכ”כ איכותיות, שגם אם יש אי אילו ראיות לצד השני (ואין), הם אמורות לכל היותר לזרוע ספק קל, אולי בכל זאת הארץ שטוחה.
    אצלך, לעומת זאת אין שום ספק, זה ברור לך כשמש. ומה אם כל הראיות לצד השני? אה מה הבעיה, הכל מזויף…

    בנוסף, כל המדידות שאתה מנסה לפקפק בהם (ניסוי קוונדיש, מטוטלת פוקו, הקבוע הקוסמולוגי וכו’) אושרו שוב ושוב ושוב על ידי מדענים שונים במקומות שונים, ה-א’ ב’ של מדע זה שחזור של ניסויים, אם אתה חושב שעולם המדע כיום מתבסס על ניסוי אחד ויחיד של קוונדיש -לשם הדוגמא- ועוד שבוצע כשהיה לבדו, אין לך שמץ שמצו של מושג איך עובד מדע או שאין לך מינימום של ביקורת על מה שמספרים לך, כל עוד זה מחזק את התאוריה שלך, אתה קונה הכל.

    נקודה דומה: לגבי מכונית הטסלה כתבת: “הטסלה בחלל היא המפסק הפתוח, ולמרות ש-99 המפסקים סגורים הנורה לא דולקת.
    אין אין טסלה, אז אין פלקון האווי, אין פלקון 9, אין כל שיגורי הלוויינים ואין את הזרוע הביצועית של נאס”א.
    אם תפסתי פעם מישהו על שקר, הוא משקר הרבה.” סוף ציטוט

    (טיעון 101) לגבי הטענה עצמה, קראתי מישהו שטוען שלפעמים בצילום וידאו (היה שם משהו עם קידוד או דווקא בשידור חי, אני לא זוכר) החלקים הדוממים בתמונה מוצגים כתקנם ורק החלקים שזזים מתחרבשים, זה גם מסביר את הסרטון שהבאת עם האסטרונאוטים שהופכים לבליל צבע לאיזה שניה. אבל אדגיש שאין לי מושג בעריכת וידאו כך שאין לי דרך לשפוט את זה. בכל אופן, אתה חושב שספייס איקס היו מפספסים כזו פשלה שמציגה אותם כשקרנים? ובכלל, זו קפיצה אדירה (לאנושות…) להגיע מאיזה ספק-ראיה כזו למסקנה שאין שום שיגורי לווינים, אפילו אם נניח שהטסלה היתה זיוף, איך זו ראיה לכל השאר??? נכון, זה מעלה חשד, אבל להגיד שזה מפסק שאם הוא נופל אין כלום? איך הגעת זה???

    (טיעון 102) בגדול התייחסתי, לגבי רשימת הטענות הנוספות, תביא קישורים ונדבר.

    (טיעון 103) -ככל שאני יודע קרני השמש כמעט מקבילות כי היא רחוקה מאוד מאיתנו ביחס לגודלה (ניסים גם מסכים איתי), בדיוק להיפך ממה שכתבת.
    לגבי ניוטון, אני לא מתמצא מספיק.

    לגבי ניסוי פיזו, ישראל כבר ענה לך שלא היה שום צורך למדוד שיניים, הוא בעצמו עשה את הניסוי בלי למדוד שום שן. אגב, לי עצמי שום מושג בנושא הזה אבל הנה מישהו שמבין בו מצוין ואפילו עשה זאת בעצמו! למה אתה מתעקש? היתה לך טעות, הבנת, נגמר. ואם לא הבנת תברר הלאה, אולי תשאל את ישראל פרטים בדיוק אבל אל תשאר תקוע עם השאלה הזו כשיש לה תשובה טובה.

    (טיעון 104) זה הטיעון החביב עלי, הוא כולו רצוף טעויות שנובעות מחוסר ההתמצאות שלך בנושא, שנתחיל?
    בתור התחלה הנה עוד סרטון שמציג את אותו שיגור של טיל אטלס מה – 02.09.15 https://www.youtube.com/watch?v=6EbBij-Vrtk זה עדיין נראה לך כמו פיצוץ?
    רואים בבירור איך הטיל ממשיך לטוס כרגיל כל הזמן, שום פיצוץ. בהמשך נראים 2 מאיצי הצד ניתקים מהגוף המרכזי וכן מה שנראה כמו חלקי החופה (שמכסה את הלווין), כמה הסברים: זה מה שקרה שם, השיגור התרחש לא הרבה זמן אחרי השקיעה, ולכן כאשר הטיל הגיע לגובה של כמה עשרות קילומטרים הוא “ראה” שוב את השמש, הטיל עצמו בקושי ייראה מהמרחק הזה, אבל גזי הפליטה שלו נראים היטב. הסיבה שהוא נראה כאילו הוא נופל לקרקע קשורה גם לכיוון הצילום וגם לכך שכשטילים משוגרים לחלל הם מתחילים במצב מאונך ואט אט משנים כיוון לכוון מאוזן, זה בגלל שהגובה הוא הבעיה הקטנה בטיסה לחלל, הנושא העיקרי הוא להביא את המטען למהירות אופקית סביב כדור הארץ של כ-28,000 קמ”ש, ולכן הטיל נראה כנופל כי הוא התקדם מאוד בכיוון הרחק ממני, בדיוק כמו שמטוס שיטוס ממני והלאה לכיוון האופק ייראה כאילו הוא הולך ומנמיך עד שייעלם לגמרי מעבר לאופק (עוד ראיה לכדוריות הארץ, בעולם שטוח היית אמור להיות יכול לראות מטוס למרחקים גדולים בהרבה מאשר בעולם כדורי (בניכוי השפעות אטמוספריות)).
    ואותו סיפור עם השיגור של ספייס איקס, בא תראה סרטון: https://www.youtube.com/watch?v=JRzZl_nq6fk&t=208s
    כאן הכתב מדבר על זה מראש! זו הסיבה שהוא הגיע בכלל לצלם את השיגור, כי הוא יודע שבשיגורים סמוכים לשקיעה נוצר מחזה יפה כזה! אגב, גם שם ניתן לראות בבירור שהטיל טס כל הזמן כרגיל בלי שום פיצוצים, ניתן גם לראות את הפרדת השלב הראשון, וכן את סילוני החנקן שיורה השלב הראשון כדי לייצב ולנהג את עצמו חזרה לאתר השיגור (בתוספת הפעלת המנוע הראשי בהמשך כמובן) ואפילו את שני חצאי החופה יורים סילוני גז קטנטנים משלהם –SPACEX מנסה להחזיר גם אותם לשימוש נוסף-, אני בטוח שיהיו לך עוד שאלות אי הבנות אם תצפה בסרטון בעיון, אבל אל תחשוב שאם אתה לא מבין משהו זו הוכחה שכל הסיפור מזוייף, תשאל את המומחים. אגב את התשובה הזו כמו רוב האחרות שכתבתי ידעתי עוד לפני ששאלת, זה לא שחיפשתי תשובה ובסוף מצאתי משהו.

    לגבי אריאן 5 לא בדקתי בעיון, אבל תשובה מתקבלת על הדעת היא שטילי אריאן 5 משוגרים כולם מבסיס אחד ויחיד בגינאה הצרפתית כידוע, אולי לכן זה נראה דומה כ”כ אחד לשני.

    ומתן, שים לב זו כבר ה”הוכחה” ה-10? 20? שלך שמופרכת בקלות על ידי מי שמבין יותר ממך בתחום האמור, אולי תלמד מזה ולהבא תתייחס לסרטונים שאנשים מאכילים אותך קצת יותר בחשדנות? ותבין שגם מה שנראה לך עדיין כראיה רוב הסיכויים שיתברר כטעות מביכה.

    (טיעון 105) זה באמת לא רציני, אתה מבין בטנקים לא בתחנת החלל ולכן אין לך שום הערכה מה דרוש לתחזוק השוטף שלה, איך אתה אומר? הכל השערות. אבל כמי שמבין קצת יותר ממך בנושא אני יכול לומר לך א. המצב הרגיל הוא 6 אסטרונאוטים, פרק הזמן שצוות התחנה מונה 3 אסטרונאוטים הוא שבועות ספורים ואף פחות. ב. תחנת החלל היא קודם כל הרבה הרבה יותר יקרה ומושקעת מטנק או צוללת, בנו אותה מראש כך שצוות מצומצם של אסטרונאוטים יוכל לתחזק אותה, ודבר נוסף לא מופעלים עליה כמעט בכלל אותם לחצים ועומסים שמופעלים על כלים ארציים.

    (טיעון 10) לגבי הוקינג, א. מה שניסים כתב ב. כן זה נדיר יחסית שאדם חולה ALS ישרוד כל כך הרבה זמן, אז מה? בעולם קורים גם דברים לא שיגרתיים, מה הקשר בין זה לנושא שלנו? כל ממוצע מורכב גם מנקודות קצה נדירות יותר. תסלח לי מתן אבל אתה מייחס לכל דבר קצת לא רגיל משמעות זדונית, כך עם הוקינג, כך עם המחדל שהוביל למחיקת התיעוד המקורי של אפולו 11 (רק הסלילים המקוריים ורק אפולו 11, זוכר?) ועוד.

    לגבי 1. לא הצלחתי להבין אותך. לגבי 2. ממש לא הבנת מה קורה שם, המסוק טס בקו ישר כל הזמן עד שהוא הגיע אל הגדה השנייה, ורק אז כשהוא ריחף מעל הקרקע הנגדית הוא החל להנמיך. הוא מנמיך כדי לנחות, אילו שום סיבה לעשות את זה מעל המים.

    (טיעון 11) תגיד, אתה צוחק עלי? מאיפה המצאת את המושגים האלה “קו ההיעלמות” ו “חוק ההיעלמות?
    אתה דומה למישהו שמנסים להוכיח לו שקיים כח המשיכה (נניח שהוא חושב שהאויר דוחף הכל כלפי מטה) אז לוקחים אותו למעבדה, מוציאים את האוויר ממיכל ומפילים חפץ כלשהוא, אתה מצפה שהוא יודה שאכן קיים כח משיכה אבל אז אתה שומע אותו אומר: לאאאאאאא, זה לא כח המשיכה זה “הכח שגורם לדברים ליפול למטה”…
    ככה אתה נשמע. חוק ההיעלמות הוא הוא עקמומיות הארץ!

    תחשוב רגע, אני נמצא בחוף ימה של תל אביב בגובה 5 מטר מעל פני הים, ומתצפת עם טלסקופ לכוון אנייה רחוקה או ארובות תחנת הכח בחדרה, איזו סיבה בעולם יש שאני לא אראה אותם (את כל הגובה שמעל 5 מטר של הגלים (נניח שהגלים הם בגובה הזה, אם יותר אתצפת מגבוה יותר ולהפך))? הרי לפי דעתך -שהעולם שטוח- האנייה או הארובות נמצאים היישר מולי, הרי שום דבר לא מסתיר לי אותם=נמצא בין העיניים שלי לבינם??? הסיבה היחידה (מלבד האטמוספירה, שעליה אנחנו לא מדברים כי שום דבר לא ישתנה בה אם אטפס עוד כמה מטרים) שמשהו יתגלה לעיני אם אצפה מגובה גבוה יותר הוא כדוריות הארץ!
    (אם טעיתי בהבנת כוונתך, אשמח לשמוע תיקון אבל בבקשה תסביר את זה בבהירות רבה, עם הסברים ודוגמאות כי עד עכשיו זה היה נורא מעורפל)

    (טיעון 13) מה שניסים אמר.

    (טיעון 14) א. כמו שניסים הסביר לך יפה, אם אתה לא יודע בדיוק רב את מיקום הספינות אין שום אפשרות לבנות איתן מערכת איכון. ב. GPS עובד גם כאשר לכל מלוא האופק אין ולו אונייה אחת.

    (טיעון 16) אתה טוען שהשמש נמצאת במרחק רק של אלפי ק”מ מאיתנו… זה מגוחך בכל כך הרבה רמות שאין לי כח להתחיל לפרט, אבל ק דבר אחד – אם השמש הייתה כ”כ קרובה (נגיד 100,000 ק”מ) אז הגודל שלה צריך להיות כ 1,000 ק”מ והירח צריך להיות עוד יותר קרוב ועוד יותר קטן…

    (טיעון 18) מה שניסים אמר.

    (טיעון 19-20) כתבת: תראה, בטוח משהו עובר, אני לא אומר שלא, הרי ראיתי זאת במו עיניי. אולי יש טכנולוגיה לשים טיל או מטוס בגובה 30-40 ק”מ , אני לא יודע, יש גם טכנולוגיה שנקראת “קרן כחולה” אני לא הבנתי מזה בדיוק, אבל אומרים שאפשר להקרין על השמים אז אולי זה.
    שוב, איני יודע מה בדיוק, אבל האור שעובר בשמיים אני לא חושב שזה לווין אלא משהו אחר, שאיני יודע בוודאות מה. סוף ציטוט.

    מתן, הוכחתי לך באופן ברור שמדובר בלווינים, כל אדם מכל נקודה על פני כדור הארץ יכול לראות בעין עשרות לווינים בעין, אנשים שמאות ק”מ מפרידים ביניהם רואים שניהם את אותו לווין בשמיים, ואם מפרידים ביניהם עד כ-50 ק”מ שניהם יראו אותו כמעט ישר מעליהם בו זמנית. זה מוכיח בבירור שגובה הטיסה של הלווין הוא מאות ק”מ, אחרת אחד מזוג האנשים שמפרידים ביניהם עשרות ק”מ היה רואה אותו הרבה יותר נמוך -קרוב לאופק- מחברו. ומה שאמרתי על זה שניתן לראות זאת מכל מקום, בכל יום, ועשרות לווינים, עונה על השאלה אולי מדובר בטיל או משהו, בנוסף להוכחה הקודמת. “הקרן הכחולה” היא במקרה הטוב טכנולוגיה בפיתוח/נסיונית ובמקרה הגרוע שטויות מוחלטות (לא בדקתי לעומק). בכל אופן, הרי אפשר לראות לווינים מכל מקום בעולם כולל מלב הים וממעמקי המדבר, ואפשר היה גם לפני עשרות שנים. ובכלל, שמת לב איך אתה עורם קונסיפרציות על גבי קונספירציות, העיקר להציל את התיאוריה שלך.

    ואחרון ממש לא חביב הסרטון היפני של הכוכבים עם מצלמת ניקון וכו’, זה כל כך מגוחך שאני לא בטוח אם בכלל כתבת את זה ברצינות, זה דומה למישהו שיטען שחלב פרה הוא בעצם ירוק ולראיה יפנה לסרטון ביוטיוב שמראה מישהו חולב פרה ומה שיוצא הוא חלב ירוק…

    כל אדם עם טלסקופ או אפילו משקפת טובה יכול להסתכל בעצמו לשמיים ולראות את כוכבי הלכת ככדורים ברורים, כולל פרטים רבים על פני השטח שלהם, אני עצמי ראיתי את זה בלי סוף פעמים.
    יש לך מושג כמה כאלה יש? להערכתי רק בארץ ישנם אלפים שמחזיקים ברשותם טלסקופ שלא לדבר על משקפות… אין לך שום ביקורת על מה שאתה רואה, יש סרטון אז זהו זו הוכחה מושלמת.

    בקיצור מתן, יש לך אולי ידע מסויים מאוד במדע, וכנראה שאתה באמת מאמין למה שאתה אומר אבל אתה מוכיח שוב ושוב ושוב שאין לך שמץ של מושג איך מוכיחים דברים, מהי ראיה ומה בדיחה.

  124. מתן,

    ״לפני 8-9 חודשים דיברתי עם מנהלת בכירה ב”חלל תקשורת” (לפני כל הבלאגן העסקי שלהם) וביקשתי מהם תמונה של הלווין בחלל בשביל עבודה של בתי בבית ספר. היא אמרה לי שאין להם תמונה של הלווין בחלל אלא רק תמונה שהלויין על הקרקע. נכון, המרחק הוא 36,000 ק”מ אז שאלתי אותה למה לא מצלמים מלווין אחר, והיא אמרה שאין להם תמונה כזאת.
    אז אם לחלל תקשורת אין תמונה של לווין, מה אני אגיד״

    הדברים שאתה כותב כאן מעידים על ילדותיות ועל חוסר הבנה ברמה הכי בסיסית! מה זה לצלם אותו מלווין אחר?? אתה רציני? מדובר בלווין תקשורת שטס במהירות של עשרות אלפי קמ״ש בחלל בגובה ספציפי ומספק שירותי תקשורת! איזו סיבה יש להם לצלם אותו כשהוא בחלל??? אתה רוצה אולי שהם ישלחו במיוחד בשבילך לווין נוסף שיטוס בצורה מתואמת באותו מסלול ובאותה מהירות, יכוון את המצלמות שלו לכיוון של הלווין השני ויצלם אותו רק כדי להפריך את הטענות המוזרות שלך על כדור ארץ שטוח? אתה חושב שעד כדי כך משעמם להם? אין להם מה לעשות?

    וואו אתה בן 40 אבל חושב ממש כמו ילד!

  125. דרך אגב ניסים, נדמה לי ששוחררה הודעה נוספת שלך (הארוכה) שקודם לא ראית אותה, אני רושם למקרה ומתן יפספס כי ראיתי שענית לו שם על הרבה דברים 🙂

  126. 🙂

    נראה לי שמתן הוא חלק מקונספירציה כלל עולמית שמנסה לשכנע אותנו שכדור הארץ שטוח. למה? ואיזה אינטרס יש להם? רק למתן פתרונים.

  127. יריב
    אם יש לך משקפת שדה טובה, אתה יכול לראות לפחות 2 דברים יפים. הראשון זה נוגה. נוגה קרובה ומוארת, ורואים אותה כעיגול במשקפת.
    הדבר השני עוד יותר מעניין – הסתכל על צדק: רואים 2-3 ירחים שלה!!!

    אבל – אל תספר מה שראית כאן. מתן יגיד שאתה משקר, שאתה חלק מהקונספירציה!!!!

  128. מתן
    בוא תסביר לי משהו.
    במסגרת העבודה שלי אני עכשיו בונה מאין פלנטריום. יש לי טבלה עצומה שמכילה מידע על כוכבים: מיקום זוויתי (right ascension ו-declination), מרחק, צבע, גודל, וכן הלאה. ה”כן הלאה” כולל מהירויות של הכוכבים – יש כוכבים בשמיים שאכן זזים, וכדי למקם אותם במדוייק, צריך להתחשב במהירויות האלה ובתאריך.

    בנוסף – ש לי נתונים על מסלולי הפלנטות סביב השמש, כולל ההשפעה ההדדית ביניהם.

    בנוסף – יש לי חישוב מדוויק של סיבוב כדור הארץ, נקיפת הקטבים ונוטציות

    בנוסף – יש לי חישוב מדוייק של מיקום הירח (החישוב הכי מסובך כאן, בגלל הרבה מאד סיבות).

    עכשיו שים לב – איך אני מצליח לחשב את תמונת השמיים מכל מקום בעולם, בכל תאריך, ובכל שעה? בפרט – איך אני יודע שיראו ליקוי חמה מלאה ב-2 ביולי, 2019? (יראו אותו בדרום אמריקה).

    אולי זה לא יקרה? אז איך אני יודע לחשב בדיוק איפה יראו את הליקוי מ-21 באוגוסט 2017? זה ליקוי שראיתי בעיניים לשמחתי 🙂 נסעתי חצי יום למקום שידעתי שבו יהיה ליקוי מלא (ולא הרבה אנשים). איך מישהו (נאס”א במקרה הזה) ידעו לחשב את זה?

    מתן – בוא נעזוב את כל הסרטים השיקריים ואת כל הסיפורים. בבקשה – תסביר לי איך התוכנה שלי מדייקת כל כך, אם כדור הארץ הוא לא כדור.

  129. מתן

    אענה רק על חלק מהדברים שלך, אבל זה לא אומר שיש משהו חכם בדברים האחרים.

    SpaceX היא חברה פרטית. בין היתר, הם מספקים, בתשלום, שירותים לנאס”א. נאס”א, בנוסף, תרמה הרבה ידע לחברה הזו, בהיותה חברה ממשלתית.

    מה הכוח שגרם להזזת המכונית? דחף מזרמי סילון מהחלק השני של הטיל (יש לו מדחפי גס שמרחיקות אותו מהמטען – ככה זה בהנדסת חלל מתן). אבל, מצידי שהסרט הוא באמת מזוייף. לא נראה לך רמה נמוכה מידי של זיוף??

    כתבת “הרבה פעמים רואים את הידיים שלהם עם פיקסלים, חסר להם אצבעות בידיים במיוחד שהם מעבירים את המיקרופון, לפעמים יש להם אקסטרה אצבעות”
    נכון. זה קורה בגלל שיטת הדחיסה של המידע. אם תרצה – אוכל להסביר את זה בפירוט – אבל קודם תלמד בבקשה מה זה הרמוניות ספריות. בלי זה לא תבין.

    רואים שסוגרים בתחנת החלפה את אחד הפתחים העגולים, והפתח מברזל מתעקם כאילו הוא עשוי מספוג.” שקר. זה פשוט שקר.

    ארתוסטנס – היוונים היו ימאיים גדולים וכבר אז הקיפו את אפריקה מדרום. הם ראו שהשמש נמצאת שם צפונית להם וידעו שהעולם העגול. ארתוסטנס ביצע מדידה יפיפיה שהתבססה על זה שהמש רחוקה.
    בוא אספר לך סוד: היוונים מדדו לא רק את גודל כדור הארץ. הם ידעו את הטוול לירח ואת גודלו, ואת הטווח לשמש ואת גודלו!!!

    שיגורים לחלל. מתן, גרתי בפלורידה בימי המעבורות, וראיתי מספר שיגורים, ביום ובלילה. אני מצטער להרוגד לך את סנאה-קלאוס – אבל הטילים האלה עולים מאד מאד מאד גבוה. הם לא מתפוצצים ולא נופלים חזרה (חוץ מאחד לצערי הרב, אני יודע שלך לא אכפת שאנשים מתו – לא רק שאין לך מוח, גם אין לך לב).
    ראיתי גם שיגורים בממערב ארה”ב – ממרחק מאות ק”מ מאתר השיגור (וחברים היו באתר והם ראו בעיניים).

    אתה חושב שהוקינג מבלף? אין לך בושה?

    תסביר לי – איך האופק מתרחק עם הגובה בעולם שטוח.

    מטולטלות פיקו הם על כבלי פלדה. וואוו – אתה שקרן חולני….

    GPS – איך זה עובד בלב ים מתן? בים אין ספינות עם אנטנות GPS. כל אחד שסיים תיכון יכול להבין למה, אבל לך אסביר (אני מאמין שכל התעודות שלך זה פייק – ואני יכול להוכיח זאת!!!) הסיבה היא שאחד העקרונות של GPS הוא שחייבים לדעת בדיוק איפה המשדר. איך אוניה כזו בים תידע את מיקומה?? הרי לה אין GPS….

    פולאריס אינו בדיוק בקוטב הצפוני השמימי – הוא מוסט כחצי מעלה. מיקומי הכוכבים דווקא כן השתנו – ופולריס לא היה “כוכב הצפון” בימי הבבלים. כל הגלקסיה מסתובבת, ולכן לא רואים שינויים גדולים.

    בגובה 20 ק”מ אתה לא רואה עדיין עקמומיות, אבל אתה כן רואה דברים שלא רואים מגובה נמוך. הסיפורים שלך על הפרעות ראות הם לא נכונות.

    אין לי כבר כוח לענות לך על כל שטות שאתה אומר, אז אכתוב תגובה נוספת ובוא שאלה אחת.

  130. מתן,

    ועוד משהו:

    ״הכוכבים הם נקודות אור מבולגנות שמנצנצות. בדיוק כמו מה שלמדנו שהיינו ילדים על כוכבים מנצנצים״.

    ממש לא, לחבר שלי יש טלסקופ, דיי פשוט האמת, ורואים בצורה ברורה מאוד את צורתם העגולה של כוכבי הלכת! נכון שכאשר מביטים בכוכב הלכת באופן ישיר דרך הטלסקופ קשה לראות זאת כי הטלסקופ רועד והמרחק גדול, אך כאשר אוספים מספר גדול של פריימים, מייצבים אותם באמצעות תוכנה לאותו מיקום יחסי בפריים ומחברים אותם לתמונה ממוצעת רואים יופי של צורה עגולה! אפילו מבחינים בפרטים רבים על פני השטח שלהם!!!

  131. מתן,

    תראה איך האמונה העיוורת שלך בשטות של עולם שטוח גורמת לך לראות דברים הפוכים ממה שהם באמת.

    תסתכל שוב בסרטון שלך:

    https://www.youtube.com/watch?v=hceMXAQWWN8

    אתה טוען שזו עדות לעולם שטוח? באמת? עזוב את זה שבשני הסרטונים שהבאת במשך רוב הזמן לא רואים בכלל את האופק בצורה ברורה ולכן לא ניתן להסיק מסקנות לכאן או לשם, אבל יש בכל זאת כמה פריימים שניתן לראות בהם בבירור את צורתו העגולה של האופק. ראה לדוגמא בדקה 02:04:18, ובדקה 02:05:30. תסתכל לא דרך מסך קטן של סמרטפון אלא דרך מסך גדול של מחשב ביתי, ותעביר את הסרטון לרזולוציה גבוהה.

    עשה צילום מסך, העתק לתוכנת עריכת תמונות ומתח קו ישר שמשיק לאופק. אתה לא רואה שהאופק עגול? אתה לא רואה שהקו הישר משיק לאופק בנקודה אחת בלבד? בדiק ותראה!

  132. לכולם – היום התפרסם ניתוח מעניין
    לפני 7 שנים מישהו טס מניו-יורק למוסקבה וצילם מהחלון את הטיסה עם time-lapse
    https://www.youtube.com/watch?v=0pliyY5S11w
    אני מאמין שהסרטון לא מזויף כי זה מלפני 7 שנים שאף אחד לא חלם על העולם השטוח.
    המעניין בסרט כי רואים שהשמש שוקעת, באמצע הטיסה יש חושך, אבל לקראת סוף הטיסה פתאום חוזר אור השמש.
    השמש שקעה אחרי 25% (1 דקה מתוך 4 דקות סרט) זמן טיסה והתחילה לזרוח לאחר 62.5% זמן הטיסה (2.5 דקות מתוך 4 דקות סרט)
    אז איך זה אפשרי אני שואל?

    הנה הסרטון “my perceptive” שפירסם ממש היום את זה ואת ההסבר איך בעולם שטוח זה הגיוני
    https://www.youtube.com/watch?v=EHp1RzPdoN0

    אבל נניח להסבר שלו –
    האם יכול להיות שבטיסה של 9 שעות מזרחה אפשר לראות את השמש שוקעת לאחר 2.25 שעות, במשך 3.5 שעות יש חושך, ואז לאחר מכן חוזר האור למשך עוד +3 שעות?
    האם זה אפשרי בעולם עגול?
    אשמח לתשובה

    מתן

  133. חלק שני

    נתחיל לענות על הדברים שלך:

    (טיעון 10)
    סרטון של המסוק – שים לב מי עשה זאת? הוקינג.
    המותג “הוקינג” גם שותף למזימה. (אתם לא שואלים את עצמיכם איך חולה ALS יכול להחזיק 50 שנה ועדיין לחיות? תוחלת החיים מרגע המחלה היא 3-5 שנים. רק 10% מהחולים מגיעים לחיות מעל 10 שנים, אבל רובם נפטרים אחרי זה בגלל סיבוך בריאות. אז איך הוקינג חי כבר 50 שנה?)
    את הסרטון הזה אני מכיר ממש מתחילת החקירה שלי.
    הסרטון ערוך.
    2 ההוכחות המרכזיות:
    1. ראה דקה 7:24 – ראה את הגבעה שמצולמת התמונה. בעינית בכלל לא מזהים אותה.
    2. ראה 07:58 ש הן צוהלות שיש 24 רגל. יש מאחוריהן חול קרוב וגבעה במקום מים. הם אמורות להיות מעל הים ולא מעל החוף.
    בקיצר הכל ערוך בסרטון.

    בקשר לסרטון הארובות:
    זה מתקשר לנושא הפרספקטיבה שאענה עליו בסוף.

    (טיעון 11)
    ההבנה שלך לא נכונה. זה שבארץ שטוחה יש אלפי ק”מ לא רלוונטית. מה שרלוונטי מה הגבולות שאתה יכול לראות. יש מגבלת ראייה – זוהי עובדה. על חוף הים אתה יכול לראות מספר ק”מ.
    אותו דבר בנוגע לשקיעה – השמש שוקעת בקו שנקרא “קו העלמות” – הקו שכל הדברים נעלמים לך.
    ואם אתה עולה יותר למעלה בגובה, השטח נפתח ואתה יכול לראות יותר הרבה. אם אתה בחוף הים ועולה על מנוף של 10 מטר נניח תוכל כבר לראות עד נניח מרחק של 20 ק”מ.
    ומנוף של 100 מטר תוכל לראות נניח עוד עשרות ק”מ.
    וממטוס תוכל לראות למאות ק”מ.
    השמש נעלמת לך מהראייה גם אם אתה במשטח אין סופי ולכן היא שוקעת.

    (טיעון 12)
    מטוטלת פוקו היא פיקציה.
    הסיבה הראשונה היא למה היא צריכה התנעה? אתה חייב לתת לה פוש ראשון כדי שתסתובב.
    הסיבה השנייה היא ממשיכה להתנועע רק בגלל האלסטיות של הכבל/חבל. זהו. אין פה שום קשר לסיבוב כדור הארץ. כל תנועה “מטעינה באנרגיה” את האלסטיות של הכבל/חבל וזה מה שגורם לה להמשיך.
    בוא נעשה כזה דבר – תשים על מוט ברזל את המטוטלת – האם הוא ימשיך לעולמי עד? לא, הוא ינוע כל הזמן לאט לאט עד לעצירה מוחלטת כי המוט לא אלסטי. בדיוק כמו בנדנדות מברזל של ילדים – למה הם לא ממשיכות לעולם ועד? כי האנרגיה שלהם נגמרת.

    (טיעון 13)
    סורי, אין עשרות אלפי צילומים. יש עשרות אלפי הדמיות ו CGI.
    תרשום בגוגל satellite רוב התוצאות הן הדמיות.
    לפני 8-9 חודשים דיברתי עם מנהלת בכירה ב”חלל תקשורת” (לפני כל הבלאגן העסקי שלהם) וביקשתי מהם תמונה של הלווין בחלל בשביל עבודה של בתי בבית ספר. היא אמרה לי שאין להם תמונה של הלווין בחלל אלא רק תמונה שהלויין על הקרקע. נכון, המרחק הוא 36,000 ק”מ אז שאלתי אותה למה לא מצלמים מלווין אחר, והיא אמרה שאין להם תמונה כזאת.
    אז אם לחלל תקשורת אין תמונה של לווין, מה אני אגיד.
    עוד משהו – לפני כחודש היה כנס חלל של משרד המדע בתל אביב וכמובן הלכתי לשם. היה ממש מעניין. היו מאות רבות של שקופיות, היו נניח בערך 100 של תמונות של לויין במצגות – רק אחת מתוך ה-100 הייתה כביכול צילום של לווין אמיתי. כל שאר ה-99 – הכל הדמיות!
    איך אני יודע אתה שואל? כי בשאלות הפתוחות שאלתי אותם על זה…….והם אמרו לי (היו 5 אנשים מ-5 סוכנויות חלל) שהנה באחד מהם היה תמונה אמיתית (אבל שאר ה-99 לוויינים במצגות היו הדמיות).
    לא בעייתי?

    (טיעון 14)
    קודם כל צריך להבין ש-99% מהתקשורת בעולם עוברת דרך…..כבלים תת-ימיים באוקיאנוסים ובימים. כלומר, כל התקשורת דרך הלווינים כבר לא כזאת חשובה
    (אנקדוטה – באותו כנס החלל שהייתי בו אתה יודע מה היו התכליות של 90% מהלווינים – אקלים וחקלאות. תקשורת בקושי הזכירו שם).
    GPS – אמרתי שוב שהמשטרה מאכנת טלפון אזי זה דרך 3 אנטנות.
    GPS מבוסס על אנטנות סלולריות, זירוי סטרטוספרי (מאות ק”מ דרך bounceמהרקיע).
    בחופי הים יש אנטנות וצלחות.
    לא פוסל שבים יש ספינות בעלות אנטנות,

    (טיעון 15)
    לא רוצה לענות. אתה יודע את תשובתי כבר

    (טיעון 16)
    תחשוב על אולם כדורסל יד אליהו חשוך. שים מנורה חזקה אבל בגובה 2 מטר בלבד ליד אחד הסלים (שים לב לא בתקרה אלא בגובה נמוך 2 מטר). חצי מהמגרש יהיה מואר, אבל חצי המגרש….יהיה חשוך.
    השמש והירח בעולם השטוח הם מקומיים– נמצאים במרחק רק של אלפי ק”מ מאיתנו. הוכחה לכך שמשגרים בלונים לשמיים, רואים כתמי שמש על המים. איך שמש שרחוקה מאיתנו 150 מיליון ק”מ עושה כתמי שמש?

    (טיעון 17)
    עניתי קודם

    (טיעון 18)
    אני מעלה את ההימור. איך אתה מסביר את תופעת ה star-trail ? רואים את כוכב הצפון קבוע וכל שאר הכוכבים מסתובבים סביבו. זה לא מסתדר למה. ומי שברדיוס גדול יותר מפולריס עובר היקף גדול יותר.
    https://www.youtube.com/watch?v=3V3rmDG5J8A
    מחזה מדהים.
    מה מיוחד בכוכב הצפון שכולם מסתובבים סביבו?
    כוכב הצפון ממוקם מעל הקוטב הצפוני – מרכז העולם
    שים לב לעוד משהו – כבר אלפי שנים הבבלים הקדמוניים רשמו את מיקומי הכוכבים (אני לא מדבר על הכוכבים במערכת השמש אלא שאר הכוכבים ומהמזלות). במשך אלפי שנים כל מיקומי הכוכבים לא זזו (הם זזים כמובן אבל בסוף חוזרים לאותה נקודה).
    איך אתה מסביר זאת?
    מהירות השמש סביב הגלקסיה היא 828,000 קמ”ש (כמובן שאנחנו גם נעים במהירות הזאת, כן….)
    בשנה היא נעה 7.2 מילארד ק”מ.!
    ב-4000 שנה היא נעה כמעט 29 אלף מיליארד ק”מ או 29 טריליון ק”מ (28,800,000,000,000) שזה שווה ערך ל-3 שנות אור!!!!!(שנת אור אחת היא 9.46 טריליון ק”מ).
    אז תגיד לי – ב-4000 שנה האחרונות זזנו 3 שנות אור, וגם שאר הכוכבים נעו כל אחד בכיוון אחר – אז איך השמיים נראים אותו דבר? איך מיקומי הכוכבים במזל שור לא השתנו? איך המיקומים במזל טלה לא השתנו?

    (טיעון 19-20)
    אני מכיר את הצילום של גדי – שוב, אתה רואה עצם מהיר שעובר. וגם בנוגע לשאר הלוויינים – אתה רואה עצמים שעוברים. אני יודע מה זה לווין – שוקל כמה ק”ג-טונות, יש בו ציוד, יש לו פנלים סולרים – אני לא רואה זאת בכל הצילומים. אני רואה נקודת אור שעוברת.
    ואחרי שהראיתי לך (טיעון 104 ) שלי – טילים לא הגיעו לשמיים אבל עדיין דיווחו שהם שיגרו לוויינים – אז למה להאמין לכך?
    תראה, בטוח משהו עובר, אני לא אומר שלא, הרי ראיתי זאת במו עיניי. אולי יש טכנולוגיה לשים טיל או מטוס בגובה 30-40 ק”מ , אני לא יודע, יש גם טכנולוגיה שנקראת “קרן כחולה” אני לא הבנתי מזה בדיוק, אבל אומרים שאפשר להקרין על השמים אז אולי זה.
    שוב, איני יודע מה בדיוק, אבל האור שעובר בשמיים אני לא חושב שזה לווין אלא משהו אחר, שאיני יודע בוודאות מה.

    לסיכום,
    האסטרונאוט דון פטיט – מה שאמר זה גם מפסק פתוח! זה לא משהו אלגורי. אתה רואה טכנולוגיה אחרת בכל תחום בעולם שהייתה לנו אותה שנות ה-60 ועכשיו אין לנו אותה? יש כזאת? בתחום המחשבים, תקשורת, מכוניות, מטוסים, צוללות, סורי, אין כזה דבר שפעם היה לנו טכנולוגיה, אבל איבדנו/השמדנו אותה והיום אין לנו.
    לכן זה מסריח מה שאמר.
    עוד משהו מעניין – אתה יודע כאשר נאס”א שידרו לכלל האומה את הנחיתה לירח, הם לא הפיצו את השידור האמיתי, אלא צילמו טלוויזיה שהקרינה את הצילום החי, ואת זה הפיצו. אתה מבין מה אומר? כלומר, מה שיש לנו היום זה צילומים מדרגה שנייה ולכן אי אפשר לנתח אותם כי הם צולמו מטלוויזיה לאחר שכל הצילום המקורי נעלם באופן מסתורי (רק שתראה, לא מדובר על גודל של דיסקון-קי או תקליט, מדובר על חדרים מלאים בסלילים ומידע אחר. חדרים שלמים, לא דיסקט קטן. איך אפשר להעלים חדרים שלמים? ולמה)

    תודה לך פלוני 321

    ניסים
    ובלי ציניות בבקשה
    אני מקווה שאתה מבין כי יש לך מגבלות ראייה למרחק של 15-20 ק”מ בגלל האטמוספרה לכן אתה לא יכול לראות מזיכרון יעקב את קפריסין. ברור שאם אתה עולה להר גבוה (כמו שאמרת) או למטוס אז נפתח-ת לך זווית ראייה שאתה יכול לראות למרחקים יותר גדולים. אבל שוב, בגלל האוויר והלחות הם מגבילים אותך. (זכור, גם פלוני 321 אמר זאת)
    ברור שרדיוס העולם הוא לא 180 ק”מ – שוב משהו שאתה לא מקבל זה הפרספקטיבה – גם בעולם שטוח אין סופי לא תוכל לראות את כולו גם אם אתה בגובה 50 ק”מ – מה לעשות בסוף יש מגבלת ראייה.
    בנוגע לצילום אגם מישיגן – זה לא נכון מה שאתה אומר. זהו צילום אמיתי ממרחק 96 ק”מ ותמונה מפורסמת מאוד שצילמו אותה מהצד של מדינת מישיגן לצד של מדינת אלינוי ששם שיקגו.
    אתה יכול להגיד שזה זיוף, אבל אף אחד אחר לא אמר זאת כי בדקו זאת. תירצו שבגלל האוויר הקר והחם “האור התעקם”. אשרי המאמין. התמונה הזאת היה הוכחה שאין עקמומיות.
    בנוגע ל ice-walls באנטרקטיקה – ראה תמונות הם יסבירו יותר טוב. (ברור לי שתגידו שזה הכל פייק כמו שאני אומר על התמונות של הלוויינים שזה פייק….)
    https://www.google.co.il/search?q=ice-wall+antarctica&rlz=1C1EJFA_enIL685IL685&tbm=isch&source=iu&ictx=1&fir=nbsCCGZcwOM4DM%253A%252C1WEJ8SEO2nEm7M%252C_&usg=__a_oqUa2Kg1-F8W8naP0ugDp8O80%3D&sa=X&ved=0ahUKEwiBqObAlLfZAhXDDewKHX0mDM8Q9QEIKjAA#imgrc=_

    בנוגע לגלים – אני אענה לכולם בסוף
    לסיכום איתך ניסים – אשמח לדעת על פי מה שאמרת שהמצלמה מחוברת למכונית, מה הדבר הפיזי שגרם למכונית לזוז ימינה, לזוז שמאלה ואז משהו איזן שהיא נשארה קבועה.
    וזאת היא עשתה פעמיים.

    ועוד דבר שוב – אם באמת טסת 20 ק”מ גובה כמו שאמרת בהתחלה – האם ראית עקמומיות או לא?

    ליריב
    1. הברית האנטרקטית היא אמנה מתחילת שנות ה-60 בו היא מסדירה מה מותר לעשות ומה אסור לעשות באנטרקטיקה. המעניין פה הוא הנושא על קו רוחב 60 ודרומה – כלומר בן אדם פרטי לא יכול לקחת מטוס או סירה על דעת עצמו וליסוע לאנטרקטיקה. אתה לא יכול לעבור קו רוחב 60 דרום (הקצה הדרומי של דרום אמריקה היא קו רוחב 56). כל האנשים שכן עוברים הם חוקרים בתיאום, או טיסות מתואמות. וגם אז לוקחים אותם למקומות ספציפיים עם ליווי.
    כל מי שסובב את העולם – כמו מגלן
    https://he.wikipedia.org/wiki/%D7%A4%D7%A8%D7%93%D7%99%D7%A0%D7%A0%D7%93_%D7%9E%D7%92%D7%9C%D7%9F
    – עשה זאת בקו מזרח-מערב. אין אף אחד, גם כיום במטוס שעשה סיבוב בעולם מצפון לדרום (הרי בעולם שטוח אתה לא יכול לעשות זאת).
    וגם מטוסים ששברו שיאים שאומרים שכן עשו, אז אתה רואה שהם הגיעו לאנטרקטיקה אבל חזרו חזרה באותה דרך לאוסטרליה. כלומר הם לא חצו לכיוון השני.
    עכשיו, אם בתיאוריה היו נותנים למישהו ספקן כמוני לשוט דרומה כל הזמן, אני מניח שהייתי מגיע לקירות של קרח, ואז הייתי צריך ללכת עוד כמה אלפי ק”מ על הקרח דרומה ואז הייתי מגיע לחופה/כיפה/רקיע שעוטפת את כדור הארץ.
    זה אותה כיפה שמונעת מטילים לחלל לחצות את קו קרמן קו 100 ק”מ לכן רק בגלל החופה אי אפשר לשגר לווינים.
    הזוי. אני יודע. גם אני חשבתי ככה לפני 11 חודשים.
    נתתי את הדוגמה שאמורה להיות שמש 24 שעות באנטרקטיקה בזמן דצמבר-פברואר, אבל הבסיס שנמצא שם במשך 4 שעות אין צל ואין שמש.

    2. אנשים פרטיים שיגרו בלונים לשמיים עד גובה 120 אלף רגל ורואים שהארץ שטוחה. זוהי רק דוגמה אחת. יש הרבה כאלה. ראה נניח 1:49:15
    https://www.youtube.com/watch?v=hceMXAQWWN8
    וזה הכי מפורסם
    https://www.youtube.com/watch?v=qAPXZgJjv7A

    באותה מידה, יש צילומים גם של בלונים שהראו שיש עקמומיות, נכון?
    לצערי אני לא מוצא את הסרטון, אבל היה לפני כמה זמן בכתב מדעי שאוניברסיטה שיגרה לשמיים בלון ורואים את העקמומיות. אבל שאתה הולך לסרטון המקורי שלה אתה רואה שבזמן שהם שיחררו את הבלון מהאדמה, ממש במטרים הראשונים אתה רואה את האדמה גם עקומה…..כלומר הם השתמשו בעדשת עין דג.

    3. איך שאר הכוכבים נראים:
    יש מצלמה מדהימה ניקון P900 שיש לה הגדלה של עד 83.
    https://www.youtube.com/watch?v=juJ2-f9T_lI
    סרטון שצולם ביפן. יש עוד הרבה כאלה.
    (רק לידיעה – ה”אינשטיין” של העולם השטוח הוא ניקולה טסלה. לא אפרט עכשיו)
    הכוכבים הם נקודות אור מבולגנות שמנצנצות. בדיוק כמו מה שלמדנו שהיינו ילדים על כוכבים מנצנצים.

    ולכולם
    “אני שם את הכסף שלי איפה שהפה” (המקור באנגלית)
    שתדעו שכבר הוצאתי 20 אלף שקל על 2 ניסויים שעשיתי עם טלסקופ ותיאודוליט– בדיקת עקמומיות ל-4 ק”מ בחוף אשקלון עם צינור שטיכמוס באורך 500 מטר (!) ובדיקת עקמומיות ל-3 ק”מ על סוללה שטוחה בבריכות ים המלח. סה”כ השתתפו ב-2 הניסויים 18 אנשים (!).
    לצערי, צינור השטיכמוס נמתח מדיי ל-650 מטר והקריאות שלו לא היו מהימנות שאפשר להשתמש בו, והניסוי השני בים המלח (השתמשתי בבריכת ים המלח לסמן 00) בגלל הלחות הרבה שיצאה מהבריכה לא הצלחתי לראות טוב ולקבל קריאות מהימנות.
    ניסוי שלישי עם הפקת לקחים מתוכנן בקרוב.

    ולכן, אני אעשה את זה קצר בנוגע לגלים –
    בעולם שטוח הגלים הרגילים בגובה 5 מטר (ולא גלים בגובה 5 ק”מ) הם קו האופק וקו ההעלמות והם המפגש עם השמיים. נקרא להם הרמפה.
    כל עצם (ספינות או הרים או ארובות חדרה) שמאחורי המפגש של הרמפה (גלים) עם השמיים – התח-תית מוסתרת בחלקה כי היא מעבר לקו האופק. זה בגלל הפרספקטיבה. ולכן רואים רק את החלק העליון.
    אבל בגלל שלא תקבלו זאת, בשבוע-שבועיים הקרובים אלך למשטח שטוח ארוך, אשים מצלמה נמוכה ואתן לילדה שלי ליסוע במכונית קטנה שתתרחק ממני – ואז אתן לכם תוצאות. גם אותי זה מעניין מה יהיה.

    בברכה (לקח לי 5 שעות לרשום זאת!)
    מתן

  134. חלק 1 מתוך 2

    (ניסים ויריב תשובות בסוף ההודעה)
    (תשובה על למה אוניות ותח-תית הרים מוסתרות בסוף ההודעה)
    ואני מבקש מכולם אף ציניות

    לפלוני 321

    (הקדמה 1)
    קודם כל על עצמי:
    אני בן 40
    בבגרות עשיתי 5 יחידות מתמטיקה 100 5 יחידות פיזיקה 94
    הייתי מ”פ וסמג”ד בשיריון ב-10 שנות קבע
    אני בעל תואר ראשון ושני (לא מדעים מדוייקים)
    (אני בטוח שאם לא הייתי נשאר בצבע קבע אזי הייתי לומד פיזיקה (מחשבים לא סובל))
    כיום אני איש עסקים
    אני חובב מדע ואסטרונומיה מגיל צעיר. אני קורא הרבה ספרי מדע ורואה הרבה סדרות תיעודיות על החלל

    מה שאני רוצה לומר, זה שאני לא ילד מתלהב בן 22 או פנסיונר שמחפש מה לעשות בזמן הפנוי. אני איש של מספרים ואני בוחן את כל הנושא הזה בעובדות ונתונים.
    עד לפני שנה הייתי בדיוק כמוכם.

    (הקדמה 2)
    בנוגע לדברי הפתיחה שלך על 2 דרכי גישה, פיתרון חידה וכו’.
    אני מתסכל כל כל דבר בחיים כמעגל חשמלי המחובר למנורה ולסוללה, ובאמצע יש 100 מפסקים בטור. המפסקים הם מוליכי על ומעבירים את הזרם בננו-שנייה. אממה, אם 99 מפסקים יעבדו בצורה מדהימה אבל מפסק אחד בודד יהיה פתוח – לעולם, אבל לעולם הנורה לא תידלק.

    ובגישה כזאת אני בא לבחון את העולם ההליוצנטרי. יכול להיות ש-99 מפסקים יעבדו מעולה – יש גרביטציה, יש לוויינים, מיליוני אנשים עובדים בזה, יש צילומים מהחלל, אבל מפסק אחד, רק אחד שאני פותח בחקירה שלי – כל העולם ההליוצנטרי נופל.

    העולם ההליוצנרי בנוי כקומות במגדל:
    קודם היוונים לפני 2500 שנה ראו שספינות נעלמות
    ואז לפני 2300 ארטוסתנס הסיק עם הניסוי שלו שהעולם עגול
    ואז קופרינקוס
    ואז קפלר עם 3 החוקים
    במקביל גלילאו
    ואז ניוטון שהסיק את הגרביטציה כדי להסביר למה מה מחזיק את הכוכבים לנוע אחד מסביב לשני
    ואז קוונדיש שמצא את ה G של הגרביטציה
    ואז ניסויים מדידים למהירות האור של פיזו פוקו (לפני זה חישבו עם פרלקסה ואברצית כוכבים)
    ואז איינשטיין
    ואז האבל שחישב את המרחק לכוכבים
    ואז מחשבים, טילים, לוויינים שהתבססו הכל על כל ההיסטוריה.

    מה שתיארתי לך למעלה זה מגדל של קומות:
    אין גרביצטיה – כל המגדל נופל
    אין קבוע G – כל המגדל נופל

    (הקדמה 3)(טיעון 17)
    אתה שואל איך אנשים רבים שעובדים בדברים הקשורים לחלל – אז הם עובדים על כל האנושות?
    אני מאמין שרוב האנשים שעובדים בתעשייה באמת מתכננים לוויינים, באמת שולחים לחלל טילים, ובאמת מקבלים אותות ממשהו, אבל לא יודעים שעובדים עליהם.
    רק אלפי אנשים יודעים את האמת האמיתית.
    אבל זה קשור לבונים החופשיים והאילומיניטי. הזרוע הביצועית זה רוקפלר, הזרוע המימונית זה רוטשילד.
    נכון, זה נשמע הזוי ומופרך. אני יודע. גם מופרך ש-2 מתוך 20 ממחלקת המילואים שלי קשורים לארגון הבונים החופשיים.
    היום דרך אגב משתמשים במושג חדש Deep-State – מאמר מעניין מלפני חודש:
    http://www.mako.co.il/news-world/international-q1_2018/Article-3af8607c9eb5161004.htm?sCh=31750a2610f26110&pId=948912327
    “בסקר שערכו השנה ה”וושינגטון פוסט” ורשת ABC נמצא שמחצית מהאמריקנים מאמינים בקיומה של זרוע מכתיבה מדיניות מתוך הממשל. הם מאמינים במושג “Deep State”.”ככל שאתה מבין את ה’דיפ סטייט’ טוב יותר, אתה מרגיש כאילו אתה יורד עמוק יותר למאורת הארנב ומבין את עומק הקונספירציה”, אומרת איב שטיגליץ, עיתונאית מניו יורק”.
    כמו שאמרתי,
    המהפכה הצרפתית נעשתה על ידי הבונים החופשיים.
    https://he.wikipedia.org/wiki/%D7%94%D7%91%D7%95%D7%A0%D7%99%D7%9D_%D7%94%D7%97%D7%95%D7%A4%D7%A9%D7%99%D7%99%D7%9D

    “במהלך עידן המהפכות, בשליש האחרון של המאה ה-18, מילאו הבונים החופשיים תפקיד מכריע בתהפוכות הפוליטיות שפקדו את אירופה ובעיקר במהפכה הצרפתית. בקרב הציבור אף רווחה הטענה שהבונים החופשיים הם שחוללו את המהפכות וכי הם קיימו גוף מחתרתי גדול נגד הממשל הישן והכנסייה. אחד החיבורים המקיפים שנכתבו בנושא הוא מחקרו של האב אוגוסטן דה בארואל: “זיכרונות להבהרת ההיסטוריה של היעקוביניזם”.”
    אז אולי זה לא הזוי כל כך…….

    לאחר ההקדמה בוא נדבר לעניין – קצת טיעונים שלי ואז אענה על שלך:
    (טיעון 101) טסלה בחלל
    בכוונה אני כל הזמן מזכיר את הטסלה של ספייס-איקס. ספייס איקס כיום היא הזרוע הביצועית של נאס”א בלספק דברים לחלל. רוב הדברים של נאס”א מסופקים לחלל על ידי ספייס איקס.
    עכשיו, בנוגע לכך שהארץ זזה מאחורי המכונית:
    https://www.youtube.com/watch?v=PrpGXfwyibI
    בסרטון רואים – כדור הארץ רועד, כדור הארץ עם פיקסלים, כדור הארץ רועד.
    יש 3 אופציות:
    אופציה 1 – הכל פייק וזה מצולם על הארץ.
    אופציה 2 – רק המצלמה זזה. אבל הרי זה לא הגיוני כי הארץ רעדה והמכונית גם הייתה צריכה לרעוד.
    אופציה 3 שניסים אמר – המצלמה מקובעת למכונית.

    בוא ננתח את אופציה 3 המצלמה מקובעת למכונית (כמו בסלפי – אתה לא זז ורק הרקע זז)
    בגלל שבחלל הכל וואקום, ואם אתה מתחיל תנועה אתה תמשיך אותה לעולמי עד כי אין התנגדות אזי:
    יש כוח/בוסט/דחף שמזיז את המכונית ימינה
    יש כוח/בוסט/דחף גדול יותר שעוצר את המכונית ומזיז את המכונית שמאלה
    יש כוח/בוסט/דחף גדול יותר שמזיז את המכונית ימינה ומאזן את המכונית והמכונית נשארת קבועה ללא תנועה.

    ושוב פעם זה קרה:
    יש כוח/בוסט/דחף שמזיז את המכונית ימינה
    יש כוח/בוסט/דחף גדול יותר שעוצר את המכונית ומזיז את המכונית שמאלה
    יש כוח/בוסט/דחף גדול יותר שמזיז את המכונית ימינה ומאזן את המכונית והמכונית נשארת קבועה ללא תנועה.

    כן, אז מה הכוח שגרם לתנועות של המכונית?
    אנו יודעים שאין אף מנועי רקטה למכונית שיכולים להזיז את המכונית.
    אם נניח שהמכונית פגעה בשברים או בשפוכת ((debris אז איך התנועות של המכונית היא ישרה וקבועה ולא מבולגנת
    אולי אוויר שיצא מהגלגלים ונתן את הבוסט? עדיין התנועה לא תהיה קבועה ומאוזנת

    אתה מבין פלוני 321, אין אף דבר פיזיקלי שיכול להסביר למה המכונית זזה בצורה שהיא זזה – ימינה, עצירה, שמאלה, איזון. ועשתה זאת פעמיים.

    לכן, אמר את זה שרלוק הולמס – ” שכאשר אתה פוסל את הבלתי אפשרי,
    מה שנשאר, בלתי סביר ככל שיהיה, חייב להיות האמת”.

    הטסלה בחלל היא המפסק הפתוח, ולמרות ש-99 המפסקים סגורים הנורה לא דולקת.
    אין אין טסלה, אז אין פלקון האווי, אין פלקון 9, אין כל שיגורי הלוויינים ואין את הזרוע הביצועית של נאס”א.
    אם תפסתי פעם מישהו על שקר, הוא משקר הרבה.

    (טיעון 102) פעלולי מחשב מתוך תחנת החלל
    המונח האמיתי כמו שרשמתי הוא fade-away. אבל מה זה משנה אם זה דוהה או מהבהב.
    זוהי הוכחה שעל גבי תחנת החלל יש פעלולי מחשב. זה שחור ולבן.
    עוד מפסק נפל.
    עכשיו, אי אפשר לבטל את נושא הסרטונים. גם את התקפת ארה”ב בעיראק בשנת 1991 ראינו דרך סרטונים בטלויזיה, במגדלי התאומים ב 2001 ראינו סרטונים של הבניינים קורסים, ולמעשה כל אירוע חדשותי אתה רואה דרך הטלוויזיה או המחשב. אז אם תבטל את הסרטונים של תחנת החלל, אז תבטל את כל שאר האירועים שקרו.
    עכשיו, נכון אפשר לזייף סרטונים, אבל היופי פה הוא שהם לרוב מקשרים בקישור אמיתי מאיפה לקחו זאת, ואתה יכול בעצמך לבדוק את הסרטון הרשמי שפורסם.
    נחזור לפעלולי המחשב:
    ואתה שואל את עצמך, למה? הרי הם משדרים מתוך התחנה, למה צריך פעלולי מחשב?
    והראיתי עוד סרטון (די חדש רק מלפני חודש) שבו רואים את הדמויות מתערבלות אבל הרקע נשאר קבוע! איך בעולם הפיזיקלי שלנו יכולים רק דמויות להתבלגן, אבל הרקע נשאר קבוע? אם הייתה בעיה בתמונה אז גם הרקע היה צריך להתבלגן
    ויש עוד עשרות פעמים שתפסו בשידורים בתחנת החלל בעיות (לא צירפתי סרטונים אבל אפשר למצוא בקלות)
    1. הרבה פעמים רואים את הידיים שלהם עם פיקסלים, חסר להם אצבעות בידיים במיוחד שהם מעבירים את המיקרופון, לפעמים יש להם אקסטרה אצבעות
    2. רואים בועות בחלל.
    3. רואים שסוגרים בתחנת החלפה את אחד הפתחים העגולים, והפתח מברזל מתעקם כאילו הוא עשוי מספוג.
    4. רואים מישהו אוכל אפונים אבל יש חיתוך של כל אקט. כלומר זה סרטון של 3-4 דקות אבל ערוך כל הזמן. ואחרי כל עריכה הוא נמצא בפוזיציה אחרת, ואז אתה חושד שעשו זאת במטוסי 0 G, ובגלל שיש לך חלון של 30-45 שניות של אפס משיכה, כל קטע שערוך הוא פעם אחרת שהמטוס עשה את הירידה שלו ל 0 משיכה.
    5. רואים מישהי בזווית של 45 מעלות, ואז לפתע היא זזה באותה תנוחה בפתאומיות 2 מטר הצידה ונתקעת בדופן התחנה כאילו משהו דחף אותה. בתחנת חלל בחלל לא אפשרי תנועה כזאת. ואז אתה שואל את עצמך שזה מתאים למטוס ב-0 משיכה ואז המטוס נכנס למערבולת והיא זזה הצידה ממש.
    6. כאשר מישהו נמצא על כדור הארץ והוא נמצא בזווית נטייה נניח 45 מעלות ומדבר (לא ישן), הראש שלו תמיד מנסה להתיישר להיות אנך, כי הראש שלנו שואף להיות ישר. אבל שרואים אסטרונאוטים בתחנת החלל בזווית של 45 מעלות, הראש שלהם גם סוטה למעלה ומנסה להתיישר. זה לא מסתדר אם אתה בחלל ואתה לא מרגיש משיכה אין לך את הדחף ליישר את הראש.
    7. תמיד אומרים שיש כמה שניות delay בין הרגע שמישהו מדבר מהארץ ועד שהם בתחנה עונים, וזה לגיטימי. אבל יש מספר סרטונים שהם מנגנים ביחד סימולטנית ללא delay אנשים בארץ ואסטרונאוט בתחנה, ואתה שואל את עצמך איך הם מנגנים סימולטנית אם תמיד יש דיליי של 2-3 שניות? זה לא ניתן.
    8. מציאות רבודה –אסטרונאוט עשה תנועות ידיים כאילו לקח חפץ ושם אותו בצד, אבל אתה כצופה לא רואה כלום. והתנועות הללו לא הגיוניות למה עשה זאת. אבל אם כמו שאמרתי יש להם מציאות רבודה אז יכול להיות שהוא לקח חפץ “משכבה אחרת” ובאמת שם אותו הצידה, אבל הייתה בעיה בשידור לנו לצופים ואנו לא ראינו את השכבה החיה של החפץ.
    ויש עוד לפחות 10 מקרים אחרים כאלה.
    וזה לא אמור להיות ככה.

    (טיעון 103) ההיסטוריה של המדע
    כבר רשמתי זאת באריכות ולא ארחיב יותר מהשורות הנ”ל:
    איך ארטוסתנס העריך בכלל שקרני השמש מקבילות, איך בכלל היה לו בתפיסה שלו את זה, הרי הם מקבילות רק בגלל שהשמש עצומה מאד. גם אז שהם חשבו שאנו מסתובבים סביב השמש, אין סיכוי שהם חשבו שהשמש כזאת עצומה.
    איך ניוטון המציא נוסחה שבמשך 200 שנה לא מדדו לא את הקבוע הקוסמלוגי ולא את הכוח
    איך פיזו מדד בעין האנושית שלו 1 שן או זמן של 0.000053 שניות?
    ועוד

    (טיעון 104) פייק טילים שמגיעים לחלל
    זה רק 3 מתוך כל הארסנל:
    1. באמת מדהים. תסתכל על זה ותגיד לי את האמת מה אתה חושב
    חיל הים האמריקאי שיגר טיל אטלס לחלל בשנת 2015. ב-3 הדקות הראשונות מדובר על שיגור מוצלח והלווין בשמיים, אבל ציינו שהייתה תופעת טבע מדהימה בגלל מזג האוויר.
    מה הבעייה? מישהו צילם את כל השיגור ותופעת הטבע במצלמת חובבים
    https://www.youtube.com/watch?v=YqEw1yz7aFs&sns=em
    זמן 03:15
    מה רואים – רואים שהטיל למעשה חזר למטה וממנו יצא האש, וכל תופעת הטבע המדהימה הזאת היא בגלל הטיל שהתפוצץ.
    אז תגיד לי, איך אפשר להאמין שבאמת לווין התקשורת של טיל אטלס בחלל?

    2. זה מתחבר לשיגור של ספייס איקס אירידיום 4 – מדצמבר 2017 שכל מערב ארה”ב ראה תופעה מדהימה בשמיים ואפילו מאסק התלוצץ ואמר שזה חייזרים עשו זאת.
    וכמובן שהשיגור שוגר בהצלחה והלוויינים בחלל.
    https://www.youtube.com/watch?v=EFrFe4MKfco
    צילום ממצלמת חובבים.
    סורי, אבל זה נראה בדיוק אותו מחזה בשמיים כמו שהטיל אטלס עשה. הטיל התפוצץ באוויר, רואים גם את הבוסטרים שלו עושים סללומים שמוציאים אש.
    סורי, הלויינים באירודיום 4 לא הגיעו לחלל. הטיל התפוצץ בדרך ועשה מחזה מרשים

    3. טיל אריאן האירופי. משימות VA 219 ועד VA 234 שוגרו בין השנים 2014-2016.
    פה אפשר להתרשם ממשימות הטיל
    https://en.m.wikipedia.org/wiki/List_of_Ariane_launches_(2010%E2%80%932019)
    שהטיל טס למעלה לחלל ורואים מלמטה את האדמה מתברר שהתמונה זהה בכולם….הם משתמשים באותו סרט. לאחר הסרטון שהבחור הזה פרסם, אני ווידאתי והסתכלתי בעצמי על השידורים הרשמיים של הטיל – זה אותו דבר.
    https://www.youtube.com/watch?v=edckAyDkJ40
    זמן 1:05:32 –חייבים לראות! כל התשעה שיגורים התמונה שרואים היא אותו דבר! תסתכלו בעצמיכם.
    אז אפשר להאמין שטיל אריאן שם לווינים בחלל?

    (טיעון 105) תיחזוק תחנת החלל
    זה לא טיעון מרכזי אבל מעניין
    כפי שאמרתי הייתי שריונר. בהתחלה על טנק מג”ח ולאחר מכן על מרכבה 3.
    לא צריך להגיד לכם כמה טיפולים עשינו.
    שעה בבוקר טל”ת, טיפולים תוך כדי תנועה, טיפולים בסוף תנועה סוף יום טא”ת, וטיפול של חצי יום כל חמישי טפ”ש.
    וכמה תקלות היו, כן גם במרכבה. תקלות במחשב, במערכת בקרת האש, במזקום, בתאים השונים, בקשר, ובכל מערכת שיש.
    והמצחיק הוא שהרבה פעמים שאתה לא נוגע בטנק שבוע – יש לך יותר תקלות.
    וכמה חלקי חילוף, כמה השתמשנו.
    ואת כל זה עושים 4 אנשי צוות רק לטנק אחד.

    בוא נלך לצוללת.
    נעלמו לפני מספר חודשים 44 אנשי צוללת בארגנטינה.
    אז בצוללת (שהיא יותר קטנה מתחנת החלל) יש נניח 30-50 אנשי צוות. ואני בטוח שיש שם הרבה תקלות, והרבה טיפולים.
    אז אתה שואל את עצמך – איך שרוב הזמן (לא בזמן חפיפה) יש רק 3 אסטרונאוטים איך הם יכולים לטפל במפלצת הזאת? יש מאות מערכות שונות, רובן הן קיום-חיים – שאם אחת מהן מתקלקלת אתה מת – אין אוויר, נפער חור, אין חימום וכו (לא כמו בטנק שגם אם משהו מתקלקל אתה עדיין חי).
    איך 3 יכולים לטפל בכל זה? במקביל הם עושים ניסויים ומחקרים.
    אתה יודע מה זה “לעשות טיפול לפני תנועה” (הכוונה פה היא טיפול ב-24 שעות) לתחנת חלל לעבור על כל המערכות ולבדוק אותן, ולהחליף חלקי חילוף? ייקח להם רק שבוע לעבור על הכל.
    מזכיר לך שרוב המערכות שם הן בנות 20 שנה, אז היה סמוך ובטוח שיש תקלות שמה.
    סורי, זה לא מסתדר.

    המשך בחלק 2

  135. אני חושב שמתן צודק בדבר אחד, הגלים אכן יכולים להסתיר חלק מהספינה/מגדל באופק,
    *אבל* ההסתרה תלויה מאוד בגובה שלי, גובה הגלים, והמרחק שלהם ממני, זה נכון שאם אני צופה מגובה מטר אחד מעל פני הים, ובמרחק ק”מ אחד ממני יש גל בגובה של שני מטר, אזי הוא יסתיר לי 10 מטר במרחק 10 ק”מ ו-50 מטר במרחק 50 ק”מ (למעשה יתכן שיותר בשל עקמומיות הארץ),

    קל מאוד להימנע מהבעיה הזו, פשוט לתצפת מגובה של יותר מ- 4 או 5 מטר מעל גובה פני הים, נדיר שיש גלים בגובה כזה ואפשר גם לבדוק את זה מראש ולעלות עוד יותר אם צריך, מה שמבטל לחלוטין את השפעת הגלים על האופק הנצפה ואם עדיין אנחנו רואים שאונייה נעלמת מלמטה קודם (רואים בבירור, וכעבור זמן מה גם הקצה העליון נעלם מעבר לאופק) זו ראייה ניצחת לכדוריות הארץ

    בשני הסרטונים שהבאתי בתגובתי, (וכן התצפית של ניסים מראש ההר) הבעיה הזו לא קיימת, באחד מדובר באגם רגוע עם גלים בגובה של עשרות ס”מ בלבד ובשני התצפית מבוצעת מגובה 5 מטר ובנוסף לא לעומק הים אלא על קו החוף, היכן שהגלים בשום אופן לא מתקרבים אפילו לגובה 5 מטר.

  136. מתן,

    יודע מה מצטער שקראתי לך אהבל (אני רואה שנעלבת) אבל תבין שאתה מדבר דברים פשוט הזויים.

    לא ענית על הנקודות שהעליתי ואני עדיין מחכה:

    1. איך זה שאף מטוס עד היום לא הגיע לקצה העולם? הרי אלפי מטוסים טסו לכל כיוון אפשרי, איך אף אחד מהם לא הגיע לקצה העולם? לאיזו תהום ענקית? כל הטייסים שקרנים?

    2. אם אף פעם לא יצאנו לחלל אז איך הסקת שהארץ שטוחה? לפי מה?

    3. מודל הגלים שלך לא יכול להסביר מדוע רואים רק חצי ספינה כשהיא נמצאת מעבר לאופק כפי שרואים בסרטון. אמרתי לך, אם הם לא הסתירו אותה כשהיא הייתה במרחק של 500 מטר ממך אז הם גם לא יסתירו אותה כשהיא תהייה במרחק של אלפי קילומטרים כי כמו שהאנייה הולכת וקטנה כך גם הגלים הולכים וקטנים באותו היחס, אז איך פתאום הם יסתירו אותה?

    4. מה לגבי הירח, וכוכבי הלכת שאנו רואים בטלסקופים שלנו? מאדים, צדק, שבתאי… מדוע הם לא שטוחים כמו כדור הארץ? איך אתה מסביר זאת?

    מחכה לתשובותיך.

  137. פלוני 321 –
    אענה לך הכל בהמשך בהרחבה

    ניסים ניסים ניסים
    אענה לך על הכל בהמשך

    וליריב השחצן
    רק שתראה איך אתה לא מבין –
    בוא אשאל אותך חידה: בעולם עגול – אתה עכשיו במטוס בניו-יורק שיוצא משדה התעופה. הטייס מסתכל במצפן איפה מזרחה אזימוט 90 ומתחיל לטוס בקו ישר כל הזמן. יותר הטייס לא משתמש במחשב ולא במצפן. מניו-יורק הוא טס אלפי ק”מ בקו ישר. מה המסלול שלו יהיה?
    א. ניו-יורק – בריטניה – סין – לוס אנגלס – ניו-יורק
    ב. ניו-יורק – דרום אפריקה! – דרומית לאוסטרליה! – מקסיקו – ניו-יורק

    99.9% מהאנשים, וגם אתה השחצן, יגידו שזה תשובה א’! זה הרי ההיגיון והורגלנו לחשוב ככה.
    אבל זה לא נכון!
    התשובה היא ב’- בגלל שאם אתה לא עושה התאמות ותיקוני כיוון לצפון כל הזמן, אתה עושה מסלול
    Great-circle של ניו-יורק.
    מהשאלות שאתה שואל פה רוצה להגיד שאתה לא יודע הכל, אז שוב קצת צניעות.

    אענה על הכל לכולם
    יום טוב
    מתן

  138. מתן
    ביום יפה רואים מאות ק”מ. אתה יכול לשקר כמה שאתה רק רוצה, אבל אתה חייב להין שחזרה על שקר לא הופך אותו לאמת. אתה מסוגל להבין את זה?

    תסביר לי איך ראיתי פסגה של הר ואל את הבסיס שלו. בלי סיפורים, בלי קישורים לאתרים של שקרנים, ובלי לחשוב שאנחנו מפגרים.

    בבקשה – איך אני יכול לראות את פסגתו של הר בים בלי לראות את הבסיס שלו.

    בלי שקרים!!!!!!!!!!!!!!!!!!!!!!!!!!!

  139. מתן
    צא מההנחה שאני לא משקר ואני לא מטומטם בסדר?

    הצילום שהפנית עליו לא מצולם מהצד השני של אגם מישיגן. זה הכל. אתה לא לוקח אידיוט והופך אותו לאלוהים. ואתה לוקח את אלוהים והופך אותו לאידיוט…

  140. יריב
    הנה עוד קישור נחמד -https://en.wikipedia.org/wiki/Bedford_Level_experiment

    יש להם הסבר לזה שלא נופלים: יש קירות קרח עצומים בקצה העולם.

    תחשוב על זה שיש לחצי מהאנשים בעולם IQ דו ספרתי (במקרה הטוב). למתן וחבריו זה כנראה אפילו חד-ספרתי.

  141. יריב, ניסים, מתן, חבר של מתן, פלוני, פולני

    יש סיכוי סביר שמתן מסתלבט עלינו. נחכה לראות אם ואיך הוא יענה על השאלה על ניסוי פיזו. אולי הוא יודה בטעות או שירד למחתרת ככתוב: מתן בסתר.

  142. ניסים,

    זה אחד הדברים הכי מטומטמים שיצא לי לראות, לא ייאמן שיש אנשים שבאמת מאמינים לזה.

    מעניין איך הם מסבירים את זה שאף מטוס או ספינה מעולם לא הגיעו לקצה של הארץ השטוחה הזו 😀 יש שם איזה מחסום או משהו שלא מאפשר להתקרב? 🙂 איך זה שאין אפילו צילום אחד של התהום בקצה?

  143. פלוני 321,

    בדיוק רציתי להגיד לך שהתגובה שלך השתחררה. היא הייתה חסומה מהסיבה שאמרתי לך, המילה ״ת-ח-ת״ הופיעה שם פעמיים 🙂

    נקודה יפה העלית לגבי החושך בארה״ב כאשר כאן אור יום, כנראה שגם ארה”ב זו קונספירציה ואין בעצם מקום כזה 😀

  144. אוקיי,עשיו אני רואה שתגובתי התפרסמה במלואה, יש ללכת אחורה עד שעה 5:33 ביום 19.02.18, שם היא נמצאת.

  145. המשך (חלק 2)

    לפני שנדון בכל הראיות שהבאת בא נסתכל על זה קצת מלמעלה:

    יש כל כך כל כך הרבה ראיות לכך שכדור הארץ הוא אכן כדור, ושאכן יש לווינים, הנה רשימה לא שלמה:

    (טיעון 10) המציאות היא שכאשר צופים למרחק בים, דברים שנמצאים רחוק מאיתנו יילכו ויתכסו מלמטה למעלה עד שייעלמו לגמרי, גם אם נצפה מגובה של עשרות מטרים – גבוה יותר מכל גל שיכול להסתיר את האוניה מאיתנו – וכן לא משנה עם כמה זום נצפה בה, זה יישאר אותו דבר, ולא, לא מדובר על שום תעתועי ראיה, רואים את זה חד וברור (מתן, אני מוכן להתווכח איתך על פרשנות אבל לא על עובדות, זו המציאות, כפי שכל ספן וטייס מכירים) ישנם המוני סרטונים שמראים את זה, באחד מהם (קישור: https://www.youtube.com/watch?v=QVa2UmgdTM4&t=2s ) אדם אחד עומד על שפת אגם עם טלסקופ, מסוק טס מהמקום שלו אל העבר השני של האגם ויורד אט אט לנחיתה, רואים בטלסקופ איך המסוק מנמיך ומנמיך ואז הוא פשוט יורד אל מעבר (ההפך מלמעלה… (אני מנסה להמנע ממילים שעלולות לחסום אותי כדברי יריב)) לקו המים ומוסתר על ידם, כאשר הצופה בטלסקופ פונה אל טייס המסוק בקשר מתברר שהוא נמצא בגובה של כ10 מטר מעל הקרקע !. באחר מישהו צופה בטלסקופ מחוף תל אביב אל ארובות חדרה ומראה בבירור איך למעלה מ-100 מטר מהחלק ה-היפך מעליון- של הארובות מכוסות על ידי המים, יתירה מזאת הוא עורך חישוב מדוייק לפי כל הנתונים (המרחק מהטלסקופ אל הארובות, הגובה שלו מעל פני הים -5 מטר-, גובה הארובות והחלק המוסתר) והפלא ופלא החישוב מראה שרדיוס כדור הארץ הוא כ- 6,371 ק”מ (כמובן שמידת הדיוק מוגבלת בשיטה זו) הנה הקישור: https://www.youtube.com/watch?v=GrihjP5tTTM&t=29s . אז נכון, סרטונים ביוטיוב אינם הוכחות מושלמות אבל הם מדגימים את מה שידוע היטב לכל מי שיש לו יד ורגל (ואפילו אצבע…) בנושא. ומתן, מספיק עם הקטע של “בהתחלה לא רואים כלום בעין ואז שמתקרבים עם זום הספינה מתגלה” נכון הספינה מתגלה בבירור, אבל רק הצ’ופצ’יק העליון שלה ואחרי עוד קצת זמן גם הוא נעלם לא משנה כמה תגדיל (וכמו שכתבתי קודם לא יעזרו לך לא גלים ולא שום תעתועי ראיה או תעתועי לשון)

    נ.ב. אני חושב שאתה צודק בדבר אחד, הגלים אכן יכולים להסתיר את החלק התח-תון של ספינה/מגדל באופק, *אבל* ההסתרה תלויה מאוד בגובה שלי, גובה הגלים, והמרחק שלהם ממני, נכון שאם אני צופה מגובה מטר אחד מעל פני הים, ובמרחק ק”מ אחד ממני יש גל בגובה של שני מטר, אזי הוא יסתיר לי 10 מטר במרחק 10 ק”מ ו-50 מטר במרחק 50 ק”מ (למעשה אני חושב שיותר בשל עקמומיות הארץ),
    קל מאוד להימנע מהבעיה הזו, פשוט לתצפת מגובה של יותר מ- 4, 5 מטר נדיר שיש גלים בגובה כזה ואפשר לבדוק זאת מראש לעלות עוד יותר, דבר נוסף: בשני הסרטונים הנ”ל הבעיה הזו לא קיימת, באחד מדובר באגם רגוע עם גלים בגובה של עשרות ס”מ בלבד ובשני התצפית מבוצעת מגובה 5 מטר ובנוסף לא לעומק הים אלא על קו החוף.

  146. טוב, אין לי סבלנות לחכות שהתגובה שלי תאושר ( זה כבר 16 שעות) אנ הולך לפרסם אותה בחלקים חלקים בתקווה שהכל יפורסם

    ובכן:

    היי מתן,

    בא נעשה כזה דבר, נמספר כל ראיה או טיעון משמעותי במספר בן 2 או 3 ספרות בתחילתו או בסופו, ונסכם שכל אחד חייב להתייחס לכל טענה ממוספרת כנ”ל שחברו מציג (להתייחס כולל כל דבר: אתה צודק, אתה טועה כי… וכל מה שבאמצע), כך נהיה מוכרחים להתייחס לכל הוכחה ולא לזו שמתחשק או קל לנו, וכך גם יהיה ברור בדיוק על מה אנחנו מדברים ולא יהיה צריך להסביר כל פעם הכל מההתחלה, (אם מדובר על מספר סרטונים שמראים את אותה נקודה או חלקים שונים של טיעון אחד ניתן להוסיף אותיות אחרי המספר למשל 45א, 45ב, 45ג, מה אתה אומר? אני מתחייב בכל אופן להתייחס לכל טענה ממוספרת שתציג.

    עכשיו לענינינו: תראה, בכל נושא בעולם שאתה מנסה לדון עליו אם הוא נכון או לא, ישנם 2 דרכים לגשת אליו, דרך אחת היא לחפש ראיות או הפרכות נקודתיות, ספציפיות, למשל הטיעון עם הספינה נעלמת באופק, או כל סרטוני האסטרונאוטים שאתה מביא, ויש דרך אחרת, להסתכל על הנושא – ממבט על, זאת אומרת למשל האם הדבר הגיוני מנקודת מבט כוללת.

    באחת התגובות הקודמות מישהו כתב לכך דוגמא יפה, אני מצטט:
    מכיר את זה שמישהו שואל אותך חידה קשה מאוד, אתה חושב וחושב ונראה לך שאין דרך בעולם לפתור אותה
    אבל חבר שלך אומר שיש גם יש תשובה לחידה, בסיטואציה כזו יש מעין דחף ללכת עם הראש בקיר ולהאמין שאין תשובה לחידה, הרי זה נראה כ”כ בלתי אפשרי!!!

    כאן צריך לקחת פסיעה אחורה ולחשוב בהיגיון, נכון שאני לא מצליח להעלות בדעתי שום פיתרון שהוא וזה נראה בלתי פתיר, אבל יש כאן מישהו (שאני סומך עליו) שאומר לי שיש, אז יש, יש הפתעות בחיים ולא הכל אני יודע. סוף ציטוט.

    אפשר לדון מנקודת מבט צרה רק על הסרטון/ההוכחה שמונחת לפניי ואם אני לא מצליח להפריך אותה או למצוא הסבר מספק אז זהו אשתכנע, דרך אחרת היא להסתכל על זה מנקודת מבט רחבה יותר שלוקחת בחשבון את שאר הראיות, את זה שאני לא יודע הכל בעולם, את זה שלא פעם ראיה נראית כמו הוכחה ניצחת אבל כשחושבים על זה פעם נוספת מגלים שלא כך וכו’ וכו’, אני מניח שגם לך קרה לא פעם שהיית בטוח במשהו או שחשבת שאתה זוכר משהו ב100% ולבסוף התברר לך שטעית, זה קורה, אנחנו בני אדם ובני אדם טועים לעיתים, לא חושבים על כל האפשרויות, לא מכירים את כל הידע והניסיון שיש בעלם וכדו’. זו הסיבה שלא נכון להתייחס רק להוכחות ספציפיות אלא גם להסתכל על הנושא במבט כולל.

  147. יריב
    כן, 30 אלף רגל. מלמדים טייס “כדי לשמור גובה – שים אף על האופק”. הכוונה היא שווקטור המהירות יהיה מכוון לאופק, כלומר – לא יהיה לא רכיב אנכי.
    במטוסי קרב יש תצוגה עילית שעליו מצוייר קו האופק. בגבהים נמוכים – הקו הזה באמת חופף לאופק בחוץ, אבל הוא יושב מעל לאופק כשטסים גבוה.

    אתמול היית בים והיה מזג אוויר נפלא: ראות לא מוגבלת (ו-10 ס”מ על החוף….). ראו יפה מאד קו חף חד וברור, וזה לא מה שהיית מצפה לראות לו העולם היה שטוח. מעבר לקו האופק – ראו פסגות מושלגות על הרים רחוקים, אבל רק את הפסגות.

    אתה מכיר את תסביך דנינג-קרוגר? קרא על זה ותזהה מיד את מתן …. באמת יש אנשים כאלה בעולם. מתן נתן כמה לינקים ליוטיוב – קרא את התגובות של המטומטמים שם. אלו אותם אנשים שמאמינים שלא נחתו על הירח, שחיסונים זה רע, שהעולם לא מתחמם ושהומיאופטיה עובדת.

  148. שני דברים לגמרי לא ברורים לי עם הטענות של מתן:

    1. כיצד הוא יודע שהארץ שטוחה אם לטענתו מעולם לא הסתכלנו עליה מהחלל? איך הוא יודע שהיא דווקא שטוחה ולא עגולה?!

    2. אם טענתו נכונה והארץ שטוחה אז כל אנייה שמפליגה הרחק לכיוון האופק אמורה להגיע לתהום ענק!! איך זה שאף ספינה או מטוס מעולם לא הגיעו לתהום כזו?? כולם משקרים???

  149. ניסים,

    הבנתי תודה על ההסבר! כלומר אתה אומר שבעולם כדורי (כמו שלנו) האופק אמור לרדת הרבה יותר מהר ככל שעולים לגובה, לעומת עולם שטוח ששם הגובה גם ירד, אבל הרבה יותר לאט.

    ואני מניח שהתכוונת להגיד 30 אלף רגל?

  150. אני אחכה לתגובה הארוכה של פלוני 321 ואז אגיב לכולם כולל ליריב
    ויריב – תודה על ההסבר למה ההודעות מתעכבות
    ניסים – בצהריים עניתי לך למה אתה לא רואה מטלסקופ, אם סוף סוף יפרסמו (תתייחס רק להודעה השנייה והשלישית – את ההודעה הראשונה רשמתי שלא הבנתי למה התכוונת ב”גובה רב”….ירד לי האסימון רק אחרי הפרסום…)

    מתן

  151. יריב
    פלוני 123 צודק.

    בוא נניח שהעולם שטוח ואינסופי. במקרה הזה קו האופק הוא המפגש של קו הראיה שלך “עם האינסוף” – שזה אחת ההגדרות של קווים מגבילים. הסתכל שוב על הדוגמה של השולחן. חשב את הזווית. עכשיו – תתחיל להגדיל את השולחן עוד ועוד. הזווית אל האופק יילך ויקטן.

    בוא נניח עולם שטוח סופי. במקרה הזה ניתן לחשב את הזווית לאופק כתלות בגובה.

    אני יודע שבגובה 30 הזווית לאופק הוא בערך 3 מעלות. גובה 30 זה 9 ק”מ. לפי חוק 1:60 המרחק לאופק יהיה 180 ק”מ (חישוב מדוייק נותן 171 ק”מ). מתן כנראה טוען שרדיוס העולם הוא 180 ק”מ….

    אם אתה מניח עולם כדורי – הזווית שראיתי נותן רדיוס של 6600 ק”מ.

    תבין מתן אומר שאם תסע 180 ק”מ תיפול מהעולם….

  152. פלוני 321,

    תודה הבנתי. גם אני נתתי למתן כמה נקודות שהייתי רוצה לראות כיצד הוא יתייחס אליהן.

    לגבי חסימת תגובות, מספיק שרשמת משהו כמו: ״האנייה שוקעת מתח-ת לאופק״ אבל בלי המקף (מילה גסה) והתגובה שלך תיחסם 🙂 צריך מאוד לשים לב שאין קומבינציות כאלו בתוך ההודעה.

  153. ליריב,

    אתה שוכח שהגובה שאתה מתרומם אליו מעל השולחן הוא אחוז משמעותי מאורך השולחן כולו, אם השולחן היה באורך 8,000 ק”מ לשם הדוגמא, היית צריך להתרומם כמעט 14 קילומטר רק כדי שתפתח לך זוית של עשירית המעלה בלבד ! (טריגונומטריה)

    אגב אתה מוזמן בתגובה הארוכה שכתבתי היום (19.02.18) בבוקר, היא עדיין לא אושרה לפירסום (הידען, מה הקטע?) אבל כשתאושר תוכל לגלול אחורה ולראות אותה.

  154. ניסים,

    ״מתן – קו האופק מנמיך כשטסים גבוה. תסביר את זה בבקשה בהנחה של עולם שטוח״

    לא הבנתי, אתה טוען שבעולם שטוח קו האופק לא אמור להנמיך? כשאני שם את הראש על שולחן שטוח אני רואה את הקצה שלו (את ״האופק״) בזווית של 20 מעלות בערך מת-חת לעיניי. כאשר אני מתרומם ועומד עכשיו אני רואה את קצה השולחן בזווית של 70 מעלות מת-חת לעיניי, כלומר קו האופק הנמיך. אז מה אני מפספס בהסבר שלך?

  155. ולאחר שיצאתי חומוס קודם……. רשמת “גובה רב” וחשבתי שגובה רב מבניינים ורק לאחר שליחת ההודעה הבנתי שהכוונה בטח ממטוס (מודה, לא הייתי מרוכז ועניתי לך תוך כדי משהו אחר)

    עכשיו תשובה לשאלתך:
    קודם, ראה מה רשם פלוני 321 לפני כמה ימים:
    “אבל אתה שוכח שכשצופים למרחקים גדולים על פני הארץ האטמוספירה העבה בולעת ומפזרת הרבה מהאור בדרך ולכן משהו שנמצא מעבר ל- 20-30 ק”מ ייראה כהה ומעורפל”
    יש מגבלות ראיה בגלל האוויר. לא משנה אם אתה בעולם שטוח או עגול, אתה לא יכול לראות למרחק רב. במיוחד מעל מים.
    גם אם תלך לאוורסט, ותמצא לך יום ללא עננים, ואתה בתיאוריה (בעולם שטוח ובעולם עגול) יכול לראות למאות רבות של ק”מ, בגלל מגבלות האטמוספרה והאוויר יהיה לך קשה לראות למרחקים של מאות ק”מ.

    אבל מה, בימים קפואים שהאוויר קר, אז כן אפשר לראות למרחקים גדולים יותר.
    הנה תמונה מפורסמת שמישהו צילם על גדות אגם מישיגן, הוא היה בצד של מישיגן וצילם את קו גורדי השחקים של שיקגו.
    https://www.youtube.com/watch?v=aLlNKy5j_O8

    המרחק הוא 60 מייל (96 ק”מ), והוא הצליח לצלם את שיקגו. בעולם עגול אין סיכוי שהיה מצליח לצלם כי יש עקמומיות ( ה drop הוא 700 מטר בערך שהוא צילם בגובה 2 מטר מהחוף – כל הבניינים נמוכים מהגובה 700 מטר).
    אז כמובן, כתב החדשות מתפתל (ביום שאחרי כי הם בטח ניסו סיבה טובה) ומסביר שהוא לא היה צריך לראות את זה, אבל הצליח לצלם בגלל….. שהאור התעקם. כן, תמשיך להאמין. האור התעקם….

    אענה בערב על הכל

  156. בהמשך להודעה הקודמת – אם התכוונת כמובן בגובה רב – “במטוס” – מעל חדרה זה משהו אחר כמובן.
    בנוגע לטלסקופ, ברור שלא תראה למרחק של 300 ק”מ כי יש מגבלות של אטמוספרה. במיוחד מעל המים.

  157. היום אני עסוק, אענה לכולם בערב על הכל.
    אבל ניסים –רשמת שאתה מחדרה בגובה רב רואה את קפריסין בעין???? 300 ק”מ? הבנתי נכון? אתה בטוח?

    אמרתי לך על נוסחאת עקמומיות:
    ה Hidden drop – כלומר החלק החבוי הוא ק”מ בריבוע כפול 8 ס”מ.
    יש אתר שאתה מכניס את הנתונים וזה מחשב לך:
    https://www.metabunk.org/curve/
    אם אני שם מרחק של 300 ק”מ, ונניח הגובה של חדרה 50 מטר – אתה יודע כמה קפריסין אמורה להיות חבויה מתחת לעקמומיות – 6 ק”מ גובה !!! (ו-5 ק”מ גובה עם מתחשבים ב- refraction). המשמעות היא שאם יש הר בקפריסין בגובה 4,999 מטר – אתה לעולם לא תראה אותו!
    לקחתי נניח את הר אולימפוס שבהרי טרואודוס הנישא לגובה של 1,952 מטר. אתה מבין שאם תוסיף להר הזה עוד 3 ק”מ עד לגובה 4,952 מטר, עדיין לא תראה אותו!!!!!!!

    מקרה שני, אני לארג’ ושמתי שהגובה שלי בחדרה הוא 1,000 מטר. עדיין קפריסין תהיה חבויה בגלל העקמומיות 2.75 ק”מ גובה !!! (ו-2.13 ק”מ גובה עם מתחשבים ב- refraction).
    אתה מבין מה אני אומר? – אפילו תעמוד בחדרה בגובה 1 ק”מ, לא תוכל לראות את אותו הר גבוה בקפריסין שנמצא 300 ק”מ ממך כי הוא עדיין נחבא בגלל העקמומיות.

    אז ככה – אם באמת כמו שרשמת ראית את קפריסין מחדרה, וזו לא הייתה טעות סופר – סורי – לפי המדע בעולם עגול לא תוכל לראות זאת בחיים.

    מתן

  158. מתן
    מחדרה בגובה רב אני רואה את קפריסין בעין. מהבית שלי בזכרון, עם טלסקופ חזק – אני לא רואה.

    הסבר סביר, או הודאה בזה שאתה מדבר שטויות.

  159. מתן
    כתבת “כן, כל אנשי העולם העגול – כמו שלא עניתם לי איך זה שכדור הארץ רעד והטסלה לא, איך אתם מסבירים את האסטרונאוט שנעלם והתמונה שלו דוהה ונעלמת? מי המשוגע? מי עדיין מאמין שהתחנה בחלל?”

    הסברתי לך בדיוק למה הטסלה לא רעדה. לא יפה. אסביר לך שוב, כי קשה לך (אני חושב שהמונח המנומס זה מאותגר)…. המצלמה ממוקמת על מוט שמחוברת לאוטו. זה הכל 🙂

    מתן – קו האופק מנמיך כשטסים גבוה. תסביר את זה בבקשה בהנחה של עולם שטוח. בלי שקרים ובלי תרוצים – תסביר או תודה שאתה שקרן. וגם תסביר למה אני יודע בדיוק לחשב את זווית ההנמכה (עשיתי את זה כחלק מהעבודה שלי – אסביר לך עם תרצה).

    סיפרתי לך על ההרים בהוואי. רואים בפרוש שהעולם לא שטוח!!!! אתה רואה את החלק העליון של הר הגעש הרחוק ולא את הבסיס שלו. הר הגעש הזה יוצא מתוך הים ואין שום דבר שיכול להסתיר את הבסיס

    שמע – עזוב אותי עם תמונות מזוייפות של חלליות ואסטרונאוטים. מה זה משנה לטענות שלי? אז זייפו את הנחיתה על הירח, כל 500 אלף האנשים שעבדו על הפרוייקט. גם לא שמו שם מחזירי לייזר שמשתמשים בהם עד היום למדידת הטווח לירח. עזוב את כל זה. תסביר רק את מה ששאלתי!!

    ובלי שקרים בסדר, מתן? מפקד הטייסת שלי נהרג בחללית הקולומביה – לא אתן ליצור כמוך לפגוע באומץ ליבו ויושרו.

  160. היי מתן,

    בא נעשה כזה דבר, נמספר כל ראיה או טיעון משמעותי במספר בן 2 או 3 ספרות בתחילתו או בסופו, ונסכם שכל אחד חייב להתייחס לכל טענה ממוספרת כנ”ל שחברו מציג (להתייחס כולל כל דבר: אתה צודק, אתה טועה כי… וכל מה שבאמצע), כך נהיה מוכרחים להתייחס לכל הוכחה ולא לזו שמתחשק או קל לנו, וכך גם יהיה ברור בדיוק על מה אנחנו מדברים ולא יהיה צריך להסביר כל פעם הכל מההתחלה, (אם מדובר על מספר סרטונים שמראים את אותה נקודה או חלקים שונים של טיעון אחד ניתן להוסיף אותיות אחרי המספר למשל 45א, 45ב, 45ג, מה אתה אומר? אני מתחייב בכל אופן להתייחס לכל טענה ממוספרת שתציג.

    עכשיו לענינינו: תראה, בכל נושא בעולם שאתה מנסה לדון עליו אם הוא נכון או לא, ישנם 2 דרכים לגשת אליו, דרך אחת היא לחפש ראיות או הפרכות נקודתיות, ספציפיות, למשל הטיעון עם הספינה נעלמת באופק, כל סרטוני האסטרונאוטים שאתה מביא, ויש דרך אחרת, להסתכל על הנושא – ממבט על, זאת אומרת למשל האם הדבר הגיוני מנקודת מבט כוללת.

    באחת התגובות הקודמות מישהו כתב לכך דוגמא יפה, אני מצטט:
    מכיר את זה שמישהו שואל אותך חידה קשה מאוד, אתה חושב וחושב ונראה לך שאין דרך בעולם לפתור אותה
    אבל חבר שלך אומר שיש גם יש תשובה לחידה, בסיטואציה כזו יש מין דחף ללכת עם הראש בקיר ולהאמין שאין תשובה לחידה, הרי זה נראה כ”כ בלתי אפשרי!!!

    כאן צריך לקחת פסיעה אחורה ולחשוב בהיגיון, נכון שאני לא מצליח להעלות בדעתי שום פיתרון שהוא וזה נראה בלתי פתיר, אבל יש כאן מישהו (שאני סומך עליו) שאומר לי שיש, אז יש, יש הפתעות בחיים ולא הכל אני יודע. סוף ציטוט.

    אפשר לדון מנקודת מבט צרה רק על הסרטון/ההוכחה שמונחת לפניי ואם אני לא מצליח להפריך אותה או למצוא הסבר מספק אז זהו אשתכנע, דרך אחרת היא להסתכל על זה מנקודת מבט רחבה יותר שלוקחת בחשבון את שאר הראיות, את זה שאני לא יודע הכל בעולם, את זה שלא פעם ראיה נראית כמו הוכחה ניצחת אבל כשחושבים על זה פעם נוספת מגלים שלא כך וכו’ וכו’, אני מניח שגם לך קרה לא פעם שהיית בטוח במשהו או שחשבת שאתה זוכר משהו ב100% ולבסוף התברר לך שטעית, זה קורה, אנחנו בני אדם ובני אדם טועים לעיתים, לא חושבים על כל האפשרויות, לא מכירים את כל הידע והניסיון שיש בעלם וכדו’. זו הסיבה שלא נכון להתייחס רק להוכחות ספציפיות אלא גם להסתכל על הנושא במבט כולל.

    לפני שנדון בכל הראיות שהבאת בא נסתכל על זה קצת מלמעלה:

    יש כל כך כל כך הרבה ראיות לכך שכדור הארץ הוא אכן כדור, ושאכן יש לווינים, הנה רשימה לא שלמה:

    (טיעון 10) המציאות היא שכאשר צופים למרחק בים, דברים שנמצאים רחוק מאיתנו יילכו ויתכסו מלמטה למעלה עד שייעלמו לגמרי, גם אם נצפה מגובה של עשרות מטרים – גבוה יותר מכל גל שיכול להסתיר את האוניה מאיתנו – וכן לא משנה עם איזו זום נצפה בה, זה יישאר אותו דבר, ולא, לא מדובר על שום תעתועי ראיה, רואים את זה חד וברור (מתן, אני מוכן להתווכח איתך על פרשנות אבל לא על עובדות, זו המציאות, כפי שכל ספן וטייס מכירים) ישנם המוני סרטונים שמראים את זה, באחד מהם (קישור: https://www.youtube.com/watch?v=QVa2UmgdTM4&t=2s ) אדם אחד עומד על שפת אגם עם טלסקופ, מסוק טס מהמקום שלו אל העבר השני של האגם ויורד אט אט לנחיתה, רואים בטלסקופ איך המסוק מנמיך ומנמיך ואז הוא פשוט יורד אל מתחת לקו המים ומוסתר על ידם, כאשר הצופה בטלסקופ פונה אל טייס המסוק בקשר מתברר שהוא נמצא בגובה של כ10 מטר מעל הקרקע !. באחר מישהו צופה בטלסקופ מחוף תל אביב אל ארובות חדרה ומראה בבירור איך למעלה מ-100 מטר מהחלק התחתון של הארובות מכוסות על ידי המים (מתן, אין גלים בגובה של 100 מטר, מה לעשות), יתירה מזאת הוא עורך חישוב מדוייק לפי כל הנתונים (המרחק מהטלסקופ אל הארובות, הגובה שלו מעל פני הים, גובה הארובות והחלק המוסתר) והפלא ופלא החישוב מראה שרדיוס כדור הארץ הוא כ- 6,371 ק”מ (כמובן שמידת הדיוק מוגבלת בשיטה זו) הנה הקישור: https://www.youtube.com/watch?v=GrihjP5tTTM&t=29s . אז נכון, סרטונים ביוטיוב אינם הוכחות מושלמות אבל הם מדגימים את מה שידוע היטב לכל מי שיש לו יד ורגל (ואפילו אצבע…) בנושא. ומתן, מספיק עם הקטע של “בהתחלה לא רואים כלום בעין ואז שמתקרבים עם זום הספינה מתגלה” נכון הספינה מתגלה בבירור, אבל רק הצ’ופצ’יק העליון שלה ואחרי עוד קצת זמן גם הוא נעלם לא משנה כמה תגדיל (וכמו שכתבתי קודם לא יעזרו לך לא גלים ולא שום תעתועי ראיה או תעתועי לשון)

    (טיעון 11) באופן דומה, רק לגבי השמש, ככל שמטפסים כך השקיעה מתאחרת (לדוגמא: מגובה של 100 מטר השקיעה מתאחרת ב-98 שניות לעומת גובה פני הים) , מתן, בעולם שטוח, כשבינך לבין השמש מפרידים אלפי קילומטרים של קרקע שטוחה, תהיה צריך לעלות הרבה הרבה יותר מ-100 מטר כדי לראות שוב את השמש באופק (אם בינך לבין “קצה העולם” מפרידים 8,000 ק”מ תהיה צריך לטפס 13.96 ק”מ כדי להרחיב את שדה הראיה/להעלות את השמש ולו בעשירית המעלה בלבד !!!)

    (טיעון 12) מטוטלת פוקו

    (טיעון 13) טוב, האמת שזה היה צריך להיות הדבר הראשון, ישנם עשרות אלפי צילומים של כדור הארץ מלווינים וחלליות, (אני חושב שתסתדר למצוא אחת בלי קישור) כמובן שתטען שהכל מזוייף אבל שים לב, לא מדובר על לווין אחד או שתיים, אלא על מאות רבות מאוד, שמופעלים על ידי מדינות וגופים שונים מאוד ולא פעם יריבים האחד לשני, כמו כן כיום ניתן לבנות לווינים זעירים (קיובסאטים) בעלות של עשרות אלפי דולרים בודדים כך שכל בית ספר או סתם אנשים מספיק כסף יכולים לבנות להם אחד כזה, וחוץ מזה, אתה צריך להתבסס על משהו חיצוני כשאתה טוען שכ-ו-ל-ם הם שקר מוחלט, אתה לא יכול להכחיש עשרות אלפי צילומים רק כי זה לא מתאים לאג’נדה שלך. גם אם אתה טוען שכל מזוייפים, עדיין הטענה שלי היא מאוד כבידת משקל, אתה צריך להביא ראיות טובות מאוד לדחות אותה ולא סתם לזרוק, אה הכל זיוף. ואל תשכח שאלות זה לא הוכחות, זה אך טבעי שיהיו שאלות ואי הבנות למישהו שלא מתמצא בתחום.
    וסתם ככה תחשוב בהיגיון (טיעון 13א) ישנם בעולם כמה וכמה חברות לווינים וחברות טילים (למשל אימרסאט, SES, אירידיום, אינטלסאט, BTRC, Telkom Indonesia, ועוד ועוד) לכל חברה כזו יש מבנים ענקיים שהיא קנתה או שוכרת, אלפי עובדים, הוצאות והכנסות של מיליארדים בשנה, מה הם עושים שם לפי דעתך??? למה שמהנדס צעיר שמחפש עבודה יסכים לשבת ולא לעשות כלום ובדרך עוד לעבוד על כל העולם? ולא אחד אלא אלפים רבים! וגם אם תטען שהמהנדסים הזוטרים בטוחים שהם באמת משגרים לווינים ורק הבכירים יודעים את הסוד, עדיין, מישהו משלם על העבודה הזאת, מיליארדי דולרים!!! למה? רק כדי לעבוד על כולם?!? זה חסר היגיון לחלוטין.

    גם אם אכן עובדים על כולנו למה צריך לעשות מזה כזה סיפור, כל כך הרבה חברות, שיגורים, לווינים, תחנת חלל שמאויישת כבר שנים ברציפות, לווינים (ואסטרונאוטים) שמתפוצצים בדרך לחלל וכו’ וכו’, לא היה הרבה הרבה יותר פשוט א. פשוט להגיד את האמת. ב. לשקר לכולם אבל במינימום הדרוש בלי שום דרמות, נגיד למצוא איזו סיבה שבגללה אי אפשר כמעט לשגר לווינים אלא רק כמה בודדים? למה לבזבז מיליארדים על עשרות “כאילו” שיגורים לחלל בשנה? תחשוב רגע כמה זה מטורף, לשגר כל שנה עשרות טילים, לבנות לווינים במאות מיליוני דולרים ללווין (סיכמנו שהמהנדסים שבונים אותם עושים את עבודת נאמנה ורק הדרגים הגבוהים יותר משקרים, או שגם המהנדסים יושבים כל היום רגל על רגל…) שלא יגיעו לשום מקום, וכל זה למה כדי לעבוד על כולנו כאילו כדור הארץ עגול? שיהיה משולש, למה שזה יהיה משנה למישהו עד כדי להשקיע כל כך הרבה כסף ומאמץ??? רק לשם הדוגמא, תסתכל על אילון מאסק מנכ”ל ומייסד ספייס איקס, הוא בחור שעובד קשה מאוד מאוד, הוא עבד לגייס אליו את המהנדסים המוכשרים ביותר שיעבדו לצידו, למה??? אפילו אם נגיד שאתה צודק, למה הוא צריך לעבוד כל כך קשה?!?

    נ.ב. אל תתבלבל, נכון שכתבתי לך לפני רגע ששאלות הם לא הוכחות, אבל זה רק בתחומים מדעיים והנדסיים, אני מדבר איתך על היגיון פשוט ובנוסף נכון אלה לא ממש הוכחות מוחלטות אבל אלה בפירוש נקודות חזקות מאוד שמקשות מאוד על התיאוריה שלך.

    (טיעון 14) לוויני GPS, כל אחד בימינו יכול לנווט לפי לוויני הGPS שממוקמים בחלל, וכמו שניסים כתב לך GPS (וטלפון לוויני גם, אגב) עובד בכל נקודה על פני כדור הארץ, כולל בלב ים מקום שאין בו שום אנטנות סלולריות.

    (טיעון 15) איפה לדעתך נמצאים אילן רמון וששת חבריו? כלואים במרתפי נאס”א? (שוב, זו לא הוכחה מוחלטת, אבל באמת…)

    (טיעון 16) איך אתה יכול להסביר למה כשאצלנו יום בארה”ב לילה, הרי לדעתך אנחנו כולנו חיים על מעין שולחן ענקי? (או שלפי דעתך אין מקום כזה בשם ארצות הברית, פשוט עובדים על כולנו, זה באמת יסביר הרבה דברים…)

    (טיעון 17) יש לך מושג כמה אנשים עובדים בתעשיית החלל ומסביב? אסטרונומים, אנשי סוכנויות החלל, טכנאים, מהנדסים שבונים לווינים, ועוד ועוד, מאות אלפי אנשים ברחבי העולם אם לא יותר, אנשים מוכשרים, מומחים, רציניים, אתה חושב שהם לא מכירים את השאלות שלך? אתה באמת חושב שהאסטרונומים וכו’ הם טיפשים מוחלטים שלא חושבים על השאלות שהעלת? או שמאיימים על כולם שישתקו? זה לא רציני.
    יש לך מושג איזו סנסציה זו תהיה אם מדענים יגלו שכדור הארץ הוא שטוח/אין לווינים/המצאה אחרת, מי שיצא נגד “קשר השתיקה” יזכה לכבוד ולפרסום עולמי! איך זה שאין שום מדען רציני שהגיע ל”חקר האמת”? (ושוב זו לא הוכחה של ממש, אבל התאוריה שלך הולכת ומאבדת מהסבירות שלה)

    (טיעון 18) בדומה לטיעון 16 רק עם הכוכבים, איך זה שמכל מקום בכדור הארץ רואים כוכבים אחרים, ובדיוק באופן שבו הם היו נראים לו כדור הארץ היה כדור? (כפי שהוא באמת)

    (טיעון 19) בכל ערב ובוקר ניתן לראות בעין עשרות לווינים, כולם מתאימים בזמנים ובמסלול על פני כיפת השמים לתחזית שניתנה ימים ושבועות מראש, איזה הסבר אחר יכול להיות מלבד שאלה באמת לווינים, אני מצרף עוד כמה סרטונים שמראים מעבר של תחנת החלל על פני השמש והירח, וכן סתם לווינים (לבקשתך). שים לב, אמרת שאתה לא מאמין לסרטון הקודם שנתתי לך כי מי שפרסם אותו הוא מהנדס “מבפנים, אין בעיה הפעם יש כאן סתם אנשים מרחבי העולם גם מישראל, https://www.youtube.com/watch?v=cDckLwhb_7Q (אותו אני מכיר בעצמי). https://www.youtube.com/watch?v=DT1wSm_iK6o , מדובר בלווין גבוה מאוד (לפי המהירות), ולא, אין סיכוי שזו ציפור, לא המהירות ולא הצורה. https://www.youtube.com/watch?v=waxqSCEFkBo . https://www.youtube.com/watch?v=qROmsXf8-fg ויש עוד ועוד בלי סוף, תשאל כל חובב אסטרונומיה בעל טלסקופ והוא יספר לך איך הוא או חבר שלו הצליחו לצלם מעבר של תחנת החלל ושל לווינים על פני השמש והירח (כולל אני עצמי.)

    (טיעון 20) יש כאן עוד נקודה, היה לך ספק אם מה שראית בשמיים אכן היה לווין (עזוב תחנת חלל, בא נפשט את זה, לווין כלשהו), ישנה הוכחה ברורה שאכן מדובר בלווין, אנשים שנמצאים מאות ק”מ ממך רואים אותו גם יחד איתך! (כמובן הם רואים את הלווין נמוך יותר באופק ופחות חזק) (דרך אגב: אני אישית ראיתי כמה פעמים את תחנת החלל ממודיעין ודיברתי בטלפון עם אחי שגר בירושלים שראה אותה גם, יחד איתי, כאשר המרחק בינינו הוא 25 ק”מ, שנינו ראינו אותה ממש מעלינו באותו זמן) באמת מתן, מה זה עוד יכול להיות?

    מה שרציתי להגיד לך עם משל המטוסים הוא שכשאדם לא מתמצא בתחום מסויים, הוא יכול לחשוב בטעות על טענות מסויימות שהם ראיות ניצחות בעוד שבפועל הם נובעות מחוסר הההבנה שלו בנושא האמור.
    לגבי כל הסרטונים שהבאת על תחנת החלל, לא עברתי על כולם עדיין אבל כמו שאמרו לך לפני אני בטוח שמומחי וידאו יסבירו את רוב אם לא את כל הטענות, במיוחד שיש כל כך הרבה ראיות איכותיות לכך שתחנת החלל אכן שם, וגם אם לא, אני ואתה לא מומחים בוידאו וגרפיקה, אין לנו את הכלים לשפוט את זה. ובכלל, אתה מתעלם מסרטון שלם (מתוך אינספור קטעי וידאו אחרים)שמציג אפס כבידה וקופץ רק כשיש שם משהו שמחזק את העמדה שלך (בלי שיש לך הבנה מעמיקה בגרפיקה וכדו’), קצת מצחיק, אם יורשה לי, מזכיר לי עורך דין שלא יכול להתמודד עם הראיות של הצד השני אז הוא נטפל לניסוח, רגע, למה בחרת במילה הזאת ולא באחרת? (מדובר על קטעים ארוכים, במטוס ניתן לחוות 0 כבידה לפרקי זמן קצרים בהרבה, 20-30 שניות בלבד) נכון, צריך למצוא תשובות לכל השאלות ששאלת, אבל המשקל שלהם כראיות הוא קטן מאוד, וקטן עוד יותר כשאתה לא מתמצא בתחום.

    לגבי מה שכן הספקתי לראות: הקטע עם הפנים שמופיעות כלווין נפרס ממעבורת החלל צ’לאנג’ר, זה פשוט השתקפות של אחד האסטרונאוטים מתוך המעבורת… (ליתר דיוק, אני לא יכול להיות בטוח בזה ב100%, אבל ברגע שיש הסבר הגיוני אחר, אין כאן שום הוכחה.)

    לגבי הסרטון עם האסטרונאוט שדוהה ברגע שהוא יוצא מהצד: ובכן הוא לא דוהה אלא מעין מהבהב, אני לא יודע מה זה כן (זיוף של מציג הסרטון? עריכה רשמית מקורית של נאס”א, שעשו אפקט מעניין?) אבל ככה לא נראה עצם שיוצא ממסך ירוק, הוא אמור להעלם, לא להבהב.
    הקטע עם האסטרונאוט שעובר מאחורה עם מה שנראה כמו רתמות… נו באמת, זה אפילו לא ראוי לתגובה, לזה אתה קורא הוכחה???

    כתבת לי: היום אני במצב טוב אז גם זורם איתך:
    אנא תראה לי את הדברים הבאים: א. תמונה של לווין (לא תחנת חלל) שצולמה מטלסקופ מכדור הארץ.
    ב. תראה לי צילום שמזהים אור ממרחק של 400 ק”מ – אבל לא מלווין שאני טוען שהוא לא קיים. צילום קרקע-קרקע. מחכה.
    א. שמתי כבר אחד אבל הנה עוד אחד https://www.youtube.com/watch?v=HdZWq3eBONQ . והנה שלישי https://www.youtube.com/watch?v=q-VMN-d0D5s . נכון זה לא משהו, אבל ככה זה, לווינים הם קטנים יותר ולא מרשימים כמו תחנת החלל ב. מכיוון שכדור הארץ הוא עגול אי אפשר לראות משהו ממרחק כזה על פני הארץ, מה גם שהאטמוספירה הסמיכה תכהה ותערפל מאוד את התמונה, בשונה מצילום כלפי מעלה, שם האויר הולך ונהיה דליל ככל שעולים.
    ראיתי את “הסרטון שמסביר את הספינות שרואים אותם מעבר לאופק” אין שם כלום, להפך, רואים בבירור שספינה קרובה נראית בחציה העליון וספינה רחוקה יותר נראית אמנם אבל ממש בקושי, רק הקצה העליון מבצבץ.

    ואחרון חביב לגבי האסטרונאוט דון פטיט אומר: “הייתי מת להגיע לירח בננו שנייה. הבעיה שאין לנו את הטכנולוגיה יותר. היה לנו אבל השמדנו אותה”. שוב אתה לוקח משהו שהוא רסיס ראיה ומתייחס אליו כאילו הוא מלכת הראיות, הוא בסך הכל התכוון (או יכול היה, זה לא משנה (כי כבר אין כאן הוכחה), אפילו שברור לכל בר דעת שהוא אכן התכוון לזה) שנאס”א בחרה שלא להמשיך עם הטיסות לירח (מסיבות תקציביות בעיקר) וכך עם הזמן היכולת הזו נעלמה, ברור שאפשר לעשות זאת שוב, אבל לשם כך יהיה צריך לבנות מחדש את כל החומרה מאז, דבר לא פשוט בהתחשב בשנים הרבות שעברו ובשינויים הטכנולוגיים הגדולים שקרו מאז, או לפתח כלים דומים חדשים, דבר שייקח זמן ויעלה הרבה מאוד כסף, זה הכל. הוא השתמש רק בלשון מעט ציורית ומוגזמת, בדיוק כמו שמישהו יגיד שתאונה מסויימת קרתה בכביש אדום, אתה תקפוץ ותאמר, היי, אבל הכביש שחור, למה הוא אמר אדום?.

    ואחרון אחרון חביב לגבי היעלמות סלילי ההקלטה המקוריים של אפולו, א. מדובר רק על אפולו 11 ורק על הסלילים המקוריים שהיו על הירח, בודאי שיש העתקים שלהם. וכן התיעוד המקורי של כל שאר משימות אפולו, עדיין יושבות בכספות ללא פגע. נכון זה קצת מוזר, אבל מחדלים קורים לפעמים, זה ממש לא מספיק כדי לשכנע אותי שכדור הארץ הוא בעצם שטוח.

  161. מתן

    הנה, בצניעות:

    בשביל מה אתה צריך לצפות בשן בודדת בניסוי פיזו? אני אומר לך שערכתי וריאציה שלו בעצמי, ולא נזקקתי לשום שן אלא רק לרמת הבהירות.

    מה רע בלייזר? אפשר כמובן עם מנורה כמו בניסוי המקורי, אך בשביל מה?

    אז הסבר לי מדוע לא ניתן למדוד את מהירות האור במתקן של פיזו עיי מדידת שיאי אור (שליליים) בלבד. רק את זה.

    ואם כן ניתן – וזה ניתן, מתן – מדוע נזקק פיזו לצפות בשן בודדת ולא הסתפק בדרך הפשוטה של שיאי האור.

  162. ודרך אגב מתן,

    האם לפי התיאוריה שלך רק כדור הארץ שטוח? מה לגבי הירח, צדק, מאדים ושאר כוכבי הלכת שאנו רואים היטב גם בעין וגם באמצעות טלסקופים? האם גם הם שטוחים? אם לא, מדוע דווקא כדור הארץ שטוח? מה גרם לכך? ואיך בכלל הגעת למסקנה שהוא שטוח?

  163. חברה יש לנו כאן אשכרה בן אדם שטוען שכדור הארץ שטוח, מי היה מאמין (עד לא מזמן חשבתי שזו בדיחה 🙂 )

    מתן,

    הטענה שלך שגלי הים מסתירים את חלקה התח-תiן של הספינה ולכן היא נראית שוקעת באופק היא טענה מגוחכת, כי ככל שהספינה מתרחקת ונהיית קטנה יותר כך גם גלי הים הולכים וקטנים באותו היחס. אם הם לא הסתירו לך את חלקה התח-תiן של הספינה כשהיא הייתה במרחק של 500 מטר ממך אז הם לא יסתירו לך אותה גם כשהיא תהיה במרחק של אלפי ק״מ ממך, כי כאשר הספינה קטנה פי 50 גם גלי הים קטנים פי 50.

    לגבי הסרטונים שהבאת, אני מבטיח לך שאם תשאל אנשים שמתעסקים ומתמחים בצילומי וידאו הם יתנו לך תשובה מספקת, חבל שמתוך בורות אתה מסיק מסקנות מטופשות. לטעון שכדור הארץ שטוח זו פשוט בדיחה.

  164. שבוע טוב
    חבריה – אני מספיק משוגע שחושב שהעולם שטוח…..נראה לכם שמזיז לי כל ההערות המעליבות והציניות שלכם? ההפך, אנא המשיכו זה מדרבן אותי. אבל קצת צניעות לא תזיק לכם. הנה תשובותיי.

    קודם כל, נראה לכם איך מזייפים את תחנת החלל – בתחנת החלל משתמשים: במטוסי G 0, בריתמות, במסך ירוק, CGI , והטכנולוגיה האחרונה – מציאות רבודה (ויש עוד 50 סרטונים שאפשר למצוא בנוסף):
    1. ההוכחה הניצחת שתחנת החלל היא פייק – ראו את התמונה של האסטרונאוט דוהה ברגע שהוא יוצא מהצד – דקה 00:37 –הם משתמשים במסכים ירוקים
    https://www.youtube.com/watch?v=PIIjD3LU__c

    כן, כל אנשי העולם העגול – כמו שלא עניתם לי איך זה שכדור הארץ רעד והטסלה לא, איך אתם מסבירים את האסטרונאוט שנעלם והתמונה שלו דוהה ונעלמת? מי המשוגע? מי עדיין מאמין שהתחנה בחלל?

    הוכחות נוספות:

    2. רואים אסטרונאוט ברקע….עם רתמות
    https://www.youtube.com/watch?v=rHLrVHsOOYE

    3. שימוש נוסף ברתמות – אחרי גלגול רואים את הבחור מימין שולח את ידו מאחורי הבחור שעשה את הגלגול:
    https://www.youtube.com/watch?v=X-huF7fRlnA

    4. מציאות רבודה – בשידור חי יש מספר שכבות חיות – שכבת הרקע, שכבת האסטרונאוט, שכבת עצם אחר – הסבר מאלף על הטכנולוגיה https://www.youtube.com/watch?v=uJhL7y0ahUE
    5. הוכחה ניצחת למציאות הרבודה – הייתה תקלה בסוף השידור והתמונה של 3 האסטרונאוטים התמלאה בצבעים – אבל שימו לב שהרקע שהוא שכבה אחרת נשאר קבוע!
    דקה 02:53
    https://www.youtube.com/watch?v=-E5nQ_3j5MA

    כן, זוהי עוד הוכחה ניצחת שרק הדמויות שלהם מתערבלות, והרקע נשאר קבוע!!! תלמדו משהו – זהו מציאות רבודה ומשתמשים בה ב”תחנת החלל”.

    דברים מוזרים נוספים:
    1. חללית צ’אלדר 1983 – אז השתמשו במודלים ולא במחשבים, ורואים פרצוף בצד בזמן שפרסו את הלווין זמן 03:25 –
    https://www.youtube.com/watch?v=0pVm7p4nkwo&t=217s
    2. אסטרונאוט דון פטיט אומר: “הייתי מת להגיע לירח בננו שנייה. הבעיה שאין לנו את הטכנולוגיה יותר. היה לנו אבל השמדנו אותה”. סליחה????? השמדנו את הטכנולוגיה להגיע לירח? ואני המשוגע חחח…….
    https://www.youtube.com/watch?v=qxJbQpdYINg
    3. נאס”א מודה שכל החומר המקורי של הנחיתה של הירח נעלם – האירוע הכי חשוב לאנושות, והם איבדו זאת….תמשיכו להאמין לזה חברים, תמשיכו להאמין. מושכים בכם בחוטים ואתם מאמינים לארגון הזה.
    https://www.youtube.com/watch?v=7q1l-jf3KqA

    ישראל שפירא – אתחיל איתך
    את הניסוי של פיזו חקרתי את כל כל הפרטים עם פיזקאים מישראל ומחו”ל בשביל להבין את הניסוי ואת המספרים שלו.
    אתה טועה ומטעה וקצת צניעות לא תזיק, באמת.
    https://en.wikipedia.org/wiki/Hippolyte_Fizeau
    “In 1849, Fizeau calculated a value for the speed of light to a better precision than the previous value determined by Ole Rømer in 1676. He used a beam of light reflected from a mirror 8 kilometers away. The beam passed through the gaps between teeth of a rapidly rotating wheel. The speed of the wheel was increased until the returning light passed through the next gap and could be seen”.

    הוא שם גלגל שיניים של 720 שיניים בולטות ועוד 720 חורים בין השיניים (gaps) והוא סובב את גלגל השיניים עד למהירות של 1562 סיבובים לדקה כדי להבחין שהאור בהלוך עובר בחור הראשון, עוברת 1 שן, והאור חוזר לחור השני. כלומר עברה 1 שן שהוא הבחין שהאור הלך ובא. וחישבתי כבר שעברו בשנייה 18,744 (!) שיניים.
    והשאלה שאף פיזקאי לא יכל לענות לי – איך הוא יכל להבחין ב-1 שן מתוך 18,744 שיניים שעוברות בשנייה, או בזמן של 0.000053?
    אז ישראל, תבטל את התור שלך לרופא השיניים ולסטנדאפיסט, ותקבע עם פיזקאי – מה התשובה? (ואתה כל הזמן לוקח אותי לייזרים (גם בתשובה האחרונה שלך), זה לא רלוונטי!)
    אני עדיין מחכה לתשובות

    ניסים ניסים ניסים
    בנוגע למנורה של אדיסון, אתה צודק (הנה אני מפרגן), ובאמת השתמשו במנורה קודם. שים לב שההערה על המדורה רשמתי על טווח של 35 ק”מ של פוקו ולא של פיזו. זורם איתך שהם זיהו מעולה מטווח של 35 ק”מ (למרות שלא מאמין לזה).
    פיזו – איך זיהה אירוע שקרה בזמן 0.000053 שניות?
    ופוקו – איך זיהה אירוע שקרה בזמן של 0.00023 שניות או איך מדד זווית כזאת קטנה?

    בנוגע לקו האופק – אמרתי כבר את דעתי. אני תמיד שואל אותך האם שאמרת שהיית בגובה של 20 ק”מ האם ראית עקמומיות?
    אתה לא ענית לי – או שאתה לא זוכר, או שאתה משקר ולא טסת בגובה הזה, או שראית את האופק ישר, או שראית עקמומיות. מה נכון?
    לא סתם שאלתי אותך.
    אחד מ”מפצחי המיתוסים” טס במטוס U2 בגובה 70,000 רגל וכמובן שהראה שיש עקמומיות (דקה 04:11). איפה הבעיה???? 3 שניות אחרי זה שהוא נמצא בקוקפיט – תסתכל על האופק – הוא…… ישר!!!! (דקה 04:14) – איך אתה מסביר שבתוך המטוס רואים את קו האופק שהוא ישר?
    אני יודע מה התשובה -מבחוץ השתמשו בעדשת עין-הדג. ואיך אתה מסביר זאת?
    “מפצחי המיתוסים” גם משתפים פעולה עם השקר.
    https://www.youtube.com/watch?v=Liom-xUzvK0

    פלוני 321
    אתה מדבר איתי על מדע, מדע ומדע. איך אתה מסביר את הסרטונים בהתחלה, בדגש את האסטרונאוט שתמונתו נהפכה ל fade-away?
    1. קודם כלל לא הגבת לי על המציאות בנוגע למכונית שמתרחקת על מגרש כדורגל גדול. האם אתה מבין שגם במשטח ישר המכונית תעלם לך? ואם תהיה יותר גבוה היא תופיע מחדש ואז שוב תיעלם שתתרחק? האם אתה מבין זאת – זה הבסיס להכל. הוא שאתה מבטל זאת ואתה אומר שתמשיך לראות שאתה 10 ס”מ מעל הרצפה את המכונית לעולם ועד? מחכה לתשובתך בנוגע לזה.
    2. אל תגיד זה מדע וזה נכון. יש פה גם הגיון בדברים. אני בכוונה חוזר לפיזו כי זה הבסיס להכל – אם אתה והמדע יגיד לי כן: “פיזו הצליח לזהות 1 שן (בזמן של 0.000053)” וככה המדע קבע שזה נכון – אני לא קונה זאת. זה שקר! יש מגבלות אנושיות ויש הגיון בדברים.
    3. בהמשך לזה, אמרתי לך שבתחילת ה-15 שניות עד לשיא שמעלי, וחישבת שזה קוסינוס 85 כפול המרחק לתחנה יוצא 35 ק”מ, עדיין זה לא יוצא 112 ק”מ. אתה לא יכול להגיד לי שאור שנמצא בגובה 400 ק”מ ממני ממש מעל הכתף שלי – הוא נמצא אופקית 112 ק”מ ממני – 90 ק”מ בתוך הים שאיני יכול לזהות בכלל.
    ואחרי שאני רואה את הפברוקים שלהם בתוך תחנת החלל – איך אאמין שהם שמה, איך?
    4. כל נושא המטוסים – מה הייתה המטרה? האם זה להראות שאתה נע ב-500,000 קמ”ש עם השמש מסביב לשביל החלב אז בגלל שזה מהירות קבועה אתה לא מרגיש כלום. אשרי המאמין
    5. בנוגע לסרטון ששלחת שזיהו את תחנת החלל חולפת לפני השמש.
    בוא נבדוק מי עשה את זה – smarter every day
    בוא נבדוק מי זה?
    הוא מהנדס בעל אינטרסים– איך אני יודע?
    הבחור הזה הוא בין הבודדים בעולם קיבל אישור ועשה סרטון במתחם השיגור של פלקון האווי ששיגר את הטסלה שכולכם עדיין מאמינים שהיא בחלל.
    https://www.youtube.com/watch?v=ImoQqNyRL8Y&t=11s
    לא מאמין לו. הוא שותף לזה.

    (אנקדוטה – ראה את הפדיחה של הסרטון שעשה מתחיל בזמן 04:53 –
    מי שלא יודע – בשיגור החי של הפלקון האווי ספייס-איקס בטעות שידרה את אותו פיד של טיל אחד ב-2 המסכים שלשני הטילים (כלומר פיד אחד של טיל לא שודר בכלל).
    אבל הבחור הזה, התלהב בשידור החי – למרות שזיהה ש-2 המסכים נראים אותו דבר – התלהב מהמקצועיות של חדר הבקרה ולמרות שזה אותו דבר עדיין חשב שזה 2 טילים. הוא כמובן לא הודה שטעה)

    היום אני במצב טוב אז גם זורם איתך:
    אנא תראה לי את הדברים הבאים:
    א. תמונה של לווין (לא תחנת חלל) שצולמה מטלסקופ מכדור הארץ.
    ב. תראה לי צילום שמזהים אור ממרחק של 400 ק”מ – אבל לא מלווין שאני טוען שהוא לא קיים. צילום קרקע-קרקע. מחכה.

    מישהו,
    זהו הסרטון שמסביר את הספינות שרואים אותם מעבר לאופק.
    https://www.youtube.com/watch?v=pPK-cNfz2Eo
    האופק הוא המפגש בין השמיים, העיניים שלך, ובין הרמפה של המים (הגלים). שספינה רחוקה שמעבר לקו הזה, אז הגלים מסתירים קודם את התחתית שלה. ככל שהגלים (הרמפה) יותר גבוהים, כך תפסיק הספינה תעלם לך יותר מהר. זה הכל פרספקטיבה ואין קשר לעקמומיות.
    אני חוזר שוב בשביל ניסים – הראיתי שאתה בחוף הים עם העיניים שלך אתה לא מזהה ספינות, ולכל אורך ההיסטוריה הבינו שהם נעלמו בגלל העקמומיות.
    כל המדענים בעבר גם בגלל העובדה הזאת הגיעו למסקנה שהעולם עגול!!!!
    אבל עם אמצעי הגדלה פתאום מזהים את הספינות, למרות שבעיניים חשופות חשבת שהם נעלמו מהאופק.
    אמור פרספקטיבה, לא עקמומיות.

    גם לא עניתם איך אתם מסבירים את ליקוי ירח חלקי שהוא הפוך – איך כדור הארץ עושה צל מלמעלה ולא מלמטה
    https://www.facebook.com/ronhagberg/posts/10155891202491166

    והדבר האחרון שאני לא מפסיק לצחוק – איך אתם מסבירים ש”השמדנו את הטכנולוגיה להגיע לירח שוב”?
    חחח
    ואני המשקר, אהה ניסים????? אסטרונאוט אומר לכם כזה דבר הזוי, ואני הלא מבין????

    אתם יכולים עדיין לקחת את הכדור האדום של המטריקס – אחרי כל מייל שלי המחיר שלו עולה…..קנו בהקדם.
    אהה סליחה טעיתי, ניסים אומר לי שהמחיר של הכדור יורד כי השמדנו את הטכנולגיה להגיע לחלל.

    בברכה

    מתן

  165. מתן,

    אגיב בהמשך בארוכה אבל בינתיים,

    זה מה שכתבת לי : ” בנוגע לתחנת החלל – אמרתי שהיא הייתה ממש מעלי (זווית של 85 מעלות) במשך 30 שניות. כלומר עברו 15 שניות מהרגע שהיא כמעט מעלי (אם הייתי מעביר אנך אז זה פוגע בערך ביישוב לידי 5 ק”מ ממני) ועד הרגע שהיא ממש מעלי (אם אני מעביר אנך זה פוגע בי).
    ב-15 שניות הללו היא עברה 112 ק”מ. ושוב השאלה שלי. היא הייתה אמורה להיות בתחילת ה-15 שניות 90 ק”מ בתוך הים (מרחק של 112 ממני) – בזווית חדה ממני, אבל הנה היא ממש כמעט מעלי ביישוב לידי ממוקם 5 ק”מ בלבד. איך אתה יכול להסביר זאת?
    כן – תמשיך להאמין שאתה יכול לראות אורות באורך 100 מטר ממרחק של 400 ק”מ. כמו שפיזו לא יכל למדוד 0.000053 שניות גם אם הוא יישבע לי, ככה לא תוכל לראות אורות או השתקפות של אורות או מה שאתה רוצה שקשור לאורות ממרחק של 400 ק”מ. ” סוף ציטוט

    יש לך כאן טעויות בסיסיות בטירגונומטריה, א. כשהתחנה נראתה לך בזוית של 85 מעלות האנך שלה היה פוגע 35.4 ק”מ ממך ולא 5 כמו שכתבת. ב. 15 שניות מהרגע שהתחנה היתה ממש מעליך היא התקדמה 112. ק”מ, נכון אבל בסיום 15 השניות האלה הזווית מהאופק שהיא נראתה לך היה 74.5.

    ובכלל הרי לא מדדת את הזוית זו רק הערכה, איך אתה יכול בכלל להוכיח משהו מזה? זויות בשמים זה משהו שניתן לטעות בו בקלות, לך תמדוד ואז נדבר. ולגבי ראיית האורות אתה שב חוזר על הטעות שלך: מה הקשר להאמין או לא להאמין??? אני מדבר איתך על מדע! פשוט תעשה חישוב מה אמורה להיות עוצמת האור שמוחזרת מתחנת החלל, אם זה לא יתאים למה שנצפה בשמיים בפועל אז יהיה לנו על מה לדבר. ואם אתה לא יודע איך מחשבים דבר כזה תתייעץ עם מי שכן יודע (פיזיקאים, אסטרונומים וכו’, שים לב לא ביקשתי ממך להאמין לכל מילה שיוצאת להם מהפה, פשוט תשאל, אתה תופתע לגלות תשובות מפורטות ומבוססות ופשוטות לכל ה”קושיות” שלך), אבל אל תחשוב שאם משהו “לא נראה לך” זו הוכחה שהוא לא נכון .

    ואגב לא ענית על על שלל ההוכחות שהבאתי לך בהודעה הקודמת שלי.

  166. ישראל
    הפטנט של אדיסון היה על השימוש בחוט להט מפחם. הנקודה היא שלפיזו היה מקור אור מאד חזק שהומצא עשרות שנים לפני הניסוי שלו.
    חבל שמתן כזה שקרן. אבל – צחוקים איתו, לא?

    אתה בטח מכיר שיש הרבה אמריקאים אטומים. עולם שטוח, לא נחתו על הירח, כמטריילז, אנטי-חיסונים, אין התחממות וכן הלאה.
    אבל – שיא הטמטום זה עולם שטוח….

  167. אנא ערף, אולי הוא לא המציא אבל פטנט על הנורה הוא רשם..

    תמיד תמצא איזה מישהו שחשב על הרעיון שלך קודם. גם E=mc^2 זה לא בדיוק של איינשטיין אלא של מהנדס איטלקי שהגיע אליה מתוך תאוריית …. לסאז׳!

  168. מתן (וגם ישראל)
    אדיסון לא המציא את המנורה, אז תנו כבוד במקום המתאים. סר המפרי דייוי המציא את המנורה ב-1806!! והמנורה שלו הייתה מאד חזקה. מתן – לעולם אל תיתן לעובדות לבלבל אותך!!! אדיסון אפילו לא המציא את מנורת הלהט – פרדריק דמולינז המציא אותו, ב-1841. אדיסון עדיין לא נולד אז….
    אבל מתן, עזוב עובדות.

    מתן – גם מתמטיקה של תיכון אתה לא יודע! בעולם שטוח קו האופק תמיד בגובה שלך!!!!! כלומר הזווית אליו תמיד 0. ואגיד לך שוב – קו האופק יורד עם הגובה. כשטסים נמוך – שמים “אף על האופק” כדי לשמור גובה. בגובה רב – האף חייב להיות מעל לאופק, והיום המחשב נותן לטייס את הזווית הזו.

    בוא אספר לך עוד משהו. לפני חודש טיפסתי על הר געש בהוואי’י שנקרא מאונה קיה – הר בגובה 4100 מטר. יש הר נוסף, בגובה 4000 מטר, במרחק 120 ק”מ. הבסיס של ההר השני יותר קרוב כמובן – כ-90 ק”מ.
    עכשיו, כשטיפסתי על ההר – קודם התגלתה הפסגה של של ההר השני (הלאה-קאלה) ולאחר מכן מעט יותר. בפסגת ההר – עדיין לא ראו את הבסיס, אבל ראו את החלק העליון של ההר בצורה יפה.

    עכשיו – על ההר הזה יש מלא טלסקופים ונתנו לי להסתכל באחד הקטנים (זה היה ביום). ומה ראיתי בטלסקופ – את אותו חלק של ההר רק גדול יותר!!!

    מתן, בבקשה, די עם השקרים. בעולם שטוח המרחק לאופק תלוי רק בראות. ביום בהיר יותר רואים רחוק יותר. בעולם עגול, המרחק לאופק מוגבל ע”י הגובה והעקמומיות. בכל המיקרים שסיפרתי, הראות הייתה מצויינת.

    ניזכרתי בעוד משהו בשבילך. בעבר, העבודה שלי הייתה לתכנן ולבנות מאמני טיסה. במאמני טיסה, יש מחשב שמצייר את העולם (האמת – לפעמים עשרות מחשבים מפלצתיים). בהתחלה, היינו מציירים את העולם כשטוח, ומשחקים עם הראות כדי לשקו האופק יהיה במרחק הנכון. ומה קרה? טייסים התלוננו שכשטסים בגובה ביום בהיר זה נראה אחרת. למשל, כשאתה טס בים ומתקרב לישראל, אז אתה רואה את הרי ירושלים לפני שאתה רואה את תל-אביב (הדגש על יום יפה!).

    כשהיכטה נעלמת אחרי קו האופק לא רואים אותה גם בטלסקופ. מתן, בבקשה אל תשקר בקשר לזה. לא כולם מטומטמים!

    נתתי לך מספר דוגמאות שמראות שהעולם עגול. אני אבקש שוב – איך אתה מסביר שקו האופק יורד ב-3 מעלות בגובה 30 אלף רגל. תראה לי חישוב פשוט שבעולם שטוח, בראות טובה, זה קורה. אתה יודע מה זה סינוס, נכון?

    הבונים החופשיים זה ארגון אמיתי. בן פרנקלין היה חבר בארגון הזה. וגם מכר שלי חבר שם. הם לא שולטים על העולם 🙂

    מתן – קדימה – הסבר לקו האופק וגם להר שראיתי. אחרת – תודה שאתה מדבר שטויות. זה לא בושה (טוב, זה כן בושה, אבל עדיין עדיף להודות).

    לא הייתי בוטה אלייך, אם לא היית משקר ללא הרף. זה מראה שמה שאתה מתאר אינו מה שאתה באמת מאמין בו. או שאתה טיפש גמור…..

  169. מתן

    בנושא אי הדיוקים.

    מה שאדיסון המציא זו נורה, לא מנורה, שכמוה היה גם בבית המקדש והיא אחד מסמלי העם היהודי למעלה מ2000 שנים, ושמשה גם בניסוי פיזו להפקת אור.

    פיזו בכלל לא ספר או התייחס אפילו לשיניים או שן בודדת, אלא רק לרמת בהירות האור שהוחזר מהמראה המרוחקת.

    אין בעיה לעשות זאת, עשיתי את הניסוי לפני 4 שנים עם קרן לייזר ולא ספרתי או התייחסתי לשיניים כלל. ראה גם טבלה בעמוד 7 ב:

    https://hal.archives-ouvertes.fr/hal-00526667v1/document

    המדידה היא של רמת האור, לא של שיניים.

    זה לא 35 ק״מ אלא פחות מחצי, 8.7 ק״מ כפול 2.

    שבת שלום.

  170. לכולם ולפני שאגיד שבת שלום

    1. הסרטון שמישהו ניתח את תמונת הטסלה בחלל, שהאדמה רעדה והמכונית לא, הוא הוכחה ניצחת שהשידור הזה הוא פייק ושהיא לא בחלל. לא משנה מה אתם חושבים עלי.

    2. אני עושה הרבה מילואים ולפני כחודשיים עשיתי מילואים וכמובן שסיפרתי להם על העולם השטוח. התברר ש-2 מהמחלקה שלי (מתוך 20) הם בארגון הבונים חופשיים!!! (נשבע בחיי שזה אמיתי!) אחד ממש חבר באגודה והשני הציעו לו כמה פעמים להצטרף רשמית והוא אמר להם שהוא ממתין ובינתיים לא רוצה. ייאמר לזכות הארגון ששניהם ממש ממש תותחים שכבר הגיעו רחוק בחיים שלהם. אז לפחות יודעים שם איך לבחור את האנשים שלהם……

    אז תמשיכו לחשוב שזה רק בסרטים של דן בראון. זה א מ י ת י !

    לישראל שפירא
    תראה את כל האי-דיוקים שלך:
    קודם כל לפיזו לא הייתה מנורה בשנת 1849. לצערו אדיסון המציא זאת 30 שנה אחרי…. היה לו רק נר או עששית.
    שנית הוא מדד על פי שינויי אור (עבר/לא עבר אור) והבחין ב-1 שן שעברה (מתוך 18,744 שיניים שזה 0.000053 שניות).
    שלישית – ואפילו נלך לשיטתך שהוא מדד את שיאי הבהירות, המשוואה S=V*Tצריך תמיד 2 משתנים כדי לדעת את השלישי. את הדרך הוא ידע 16 ק”מ, אז איך לפי מדידה של שיאי בהירות הוא הגיע לזמן שזה, שוב, 0.000053 שניות? מה יעזור שיאי הבהירות שלך אם אין זמן? (ואחזור ואומר שכל החישובים המתמטיים של הניסוי הזה מסתדרים מעולה – 1562 סיבובים לדקה, 720 שיניים בגלגל, 1 שן שזוהתה – מהירות של 313,000 ק”מ בשנייה. יש רק בעיה פיזית – איך הבחור הבחין בזה בעין האנושית שלו).
    אחריו הגיע פוקו בשנת 1862 (החבר של ניסים שעשה את מטוטלת פוקו) וגם מדד את מהירות האור 298,000 ק”מ לשנייה, והבעיות הן אותן בעיות: עבר 0.00023 שניות שאף עין אנושית לא יכולה לזהות והוא מדד זווית (לטווח קרוב מספר מטרים ולא לטווח רחוק) של 0.00023 מעלות, שזה פחות משניית קשת ! (פחות מ1/3600 המעלה). ועוד בעיה שהוא הסתכל לטווח של 35 ק”מ, וגם לצערו אדיסון עדיין לא המציא את הנורה, אז איך “ירה” אלומת אור של נר או עששית לטווח של 35 ק”מ הלוך, 35 ק”מ חזור והסיט אותה במראה המסתובבת שלו? אתה עושה מדורה ענקית – האם תשים מראה במרחק 35 ק”מ ותראה השתקפות? זה כל כך מגוחך.
    (ושוב – בכוונה אני הולך למדענים הללו לפני המצאת הלייזר, המחשב וכו).
    הדבר הזה הוא קריטי כי לאחריהם איינשטיין על בסיס המדידות הללו ביסס את תורת היחסות.

    לפלוני 321
    אגיד לך מה מפריע לי – המדע אומר שזה נכון, ועד לפני שנה האמנתי באמת להכל, ואז אתה בא לבדוק בעיניים ביקורתיות ואתה רואה שאין סיכוי שבן אדם אנושי מדד כאלה דברים. ואל תגיד לי הם גאונים ומדענים ואני לא. אני לא מקבל את זה. לכל אחד יש את הזכות לחקור בעצמו ולבדוק, ומה לעשות, גם ההיגיון מדבר, ופיזו לא יכל לזהות בעינו 1 שן או זמן של 0.000053 שניות, פוקו לא יכל למדוד זמן כזה קטן, זווית כזאת קטנה, ועם נר/עששית לטווח של 35 ק”מ.
    עכשיו, דיברת על השקיעה בין 2 אנשים שונים בגובה להוכחה לעקמומיות. מה הקשר? כל אחד שעולה יותר למעלה…..נפתחת לו זווית ראייה יותר גדולה!. מי שעל שפת הים רואה את האופק למרחק נניח 5-6 ק”מ, ועם ניסים שנמצא בזכרון יעקב בגובה רב, רואה את האופק שלו במרחק של 30 ק”מ. זה לא הוכחה לכלום. גם בעולם שטוח זה ככה.
    לך למגרש כדורגל גדול. בשביל הסיפור המגרש אורכו 100 ק”מ, אין דשא והכל הוא אספלט שטוח. שים את העין שלך על קו השער שלך במרחק של 10 ס”מ מהקרקע ותסתכל לכיוון השער השני. ממך אני מסיע מכונית לכיוון השער השני שכל הזמן מתרחקת ממך. מתישהו אתה תראה אותה נעלמת! האם בגלל שהמגרש עגול? הרי אמרתי שהמגרש שטוח. היא נעלמת כי היא נעלמה לך מהקו שאתה יכול לראות בעיניים שלך, מהפרספקטיבה שלך.
    בוא אראה לך נס! תעלה את זווית הראייה שלך בעוד 20 ס”מ בגובה ולפתע תראה את המכונית! נס!. היא ממשיכה להתרחק ממך ואחרי מספר דקות היא שוב נעלמת ממך! המגרש עגול? לא, היא נעלמת לך מקו הראייה בגלל הפרספקטיבה.
    שים דגל על האוטו – אתה תשים לב שקודם האוטו נעלם לך אבל עדיין תראה את הדגל! כי ככה עוברת הפרספקטיבה.
    וחוזר חלילה.

    על השאלה השלישית שלי בנוגע לתחנת החלל – אמרתי שהיא הייתה ממש מעלי (זווית של 85 מעלות) במשך 30 שניות. כלומר עברו 15 שניות מהרגע שהיא כמעט מעלי (אם הייתי מעביר אנך אז זה פוגע בערך ביישוב לידי 5 ק”מ ממני) ועד הרגע שהיא ממש מעלי (אם אני מעביר אנך זה פוגע בי).
    ב-15 שניות הללו היא עברה 112 ק”מ. ושוב השאלה שלי. היא הייתה אמורה להיות בתחילת ה-15 שניות 90 ק”מ בתוך הים (מרחק של 112 ממני) – בזווית חדה ממני, אבל הנה היא ממש כמעט מעלי ביישוב לידי ממוקם 5 ק”מ בלבד. איך אתה יכול להסביר זאת?
    כן – תמשיך להאמין שאתה יכול לראות אורות באורך 100 מטר ממרחק של 400 ק”מ. כמו שפיזו לא יכל למדוד 0.000053 שניות גם אם הוא יישבע לי, ככה לא תוכל לראות אורות או השתקפות של אורות או מה שאתה רוצה שקשור לאורות ממרחק של 400 ק”מ.

    אני ארשום בתחילת השבוע על כל הבעיות שיש עם התחנת החלל. יהיה מעניין. תבואו…….

    לקורא בידען ולמישהו
    אני רושם שוב על נושא הספינות הפעם ב-2 היבטים:
    1. כפי שניתן לראות ב-2 הסרטונים, בזמן תחילת הצילום רואים את האופק והעיניים שלך לא מזהות אף ספינה. וכפי שסיפרתי, נניח שאם הספינה עזבה אותך במים הרדודים (זמן של מינוס 10 דקות) היא נעלמה לך (כמו שרואים בסרטון בזמן 00:00) ואז אם אתה מגדיל במצלמה היא נראית לך במלוא הדרה (זמן של 00:01). כלומר, היא לא נעלמה מעבר לעקמומיות, כי הרי אתה רואה אותה עכשיו.
    2. נושא שני זה הפטה מורגנה או המיראז’ מעל המים. שמתסכלים רחוק מעל מים יש הרבה שבירות של האור בגלל האטמוספרה והלחות ומה שאתה רואה זה מין פטה מורגנה. תעתועי ראייה.

    יש קו שנקרא “קו מראה” – זה קו שמחלק את הספינה ל-2: אתה רואה חלק מהספינה ואת המראה שלה כלפי מטה. לפעמים הספינה….מרחפת בשמיים!
    סרטון ראו דקה 10:02 – ראו את הספינה עם קו המראה – רואים אותה כפול ושהיא מעל המים.
    https://www.youtube.com/watch?v=awCx5ob04ZY&t=543s
    הכל זה תעתועים בגלל האטמוספרה שמעל המים.
    סרטון שמסביר את קו המראה. שימו לב שעוברת ספינה קרובה יותר בזמן של 07:27 ורואים את הספינה הרחוקה עם קו המראה למרות שהספינה הקרובה נראית מלא.
    https://www.youtube.com/watch?v=0xWsuFLdgBs

    אין לכך שום קשר לעקוממיות.

    לניסים הידיד הראשון
    עניתי כבר לשאלתך ואענה שוב – כמו בעולם שטוח, ככל שאתה עולה למעלה אתה מתרחק מהאדמה ומהקו של המפגש של השמיים והאדמה. ולכן אתה באמת רואה את קו האופק יותר נמוך. ושוב, אין לזה קשר לעקמומיות.

    עכשיו אשים את הציניות בצד – האם שהיית בגובה 20 ק”מ ראית עקמומיות? בגובה 10-12 ק”מ לא רואים עקמומיות. אשמח אם באמת תוכל לענות על השאלה כי היא מאד חשובה. זה חשוב לי לדעת האם רואים ב-20 ק”מ עקמומיות.

    בברכת שבת שלום לכולם

    מתן

  171. למה אתה לא מתייחס לסרטון הראשון שרואים בו בבירור את הספינה שוקעת מתחת לאופק?

  172. פוגי גורס: גרביים – לא חסר.

    אז בואו נסכם: אין דבר כזה ארץ שטוחה, אין! זוהי קונספירציה ילדותית ומגוחכת שכל תכליתה לשרת אינטרסים דתיים.

    ועכשיו אני מבקש מכולם לרדת מהסיפון ולהכנס לתאים. הרעש שאתם עושים מבלבל את מערכת הניווט, והאניה תיפול מעבר לאופק ותטבע.

  173. ישראל
    תפסתי פעם גרב שניסתה לברוח ממכונת הכביסה. היא נתקעה מתחת לאטם של הדלת הקידמית. כמובן שהיא הכחישה את כל הסיפור.

  174. טוב, מה הבעיה עכשיו עם ניסוי פיזו – הוא לא ספר או מדד שיניים. מה הוא, רופא שיניים?

    מה שהוא מדד זה את שיאי הבהירות של האור שהוחזר מהמראה במרחק 8.5 ק״מ מהמנורה שלו. ניסוי נחמד שגם סחבק ביצע בשינויים מסויימים.

    וגם את הבעיה עם G לא ממש הצלחתי להבין. אבל אם בקונספירציות עסקינן – אז מה עם ההוא שקבר את פול בסגר׳נט פפר? שלא לדבר על הגרביים שנעלמות בכביסה?

    ?
    ??
    ??!??

  175. היי מתן
    לגבי התצפית בתחנת החלל (שראית בעצמך) העלית 3 שאלות:

    ” 1. איך אני מזהה אור בגודל מגרש כדורגל 350 ק”מ ממני? ניקוסיה בקפריסין במרחק של כ-350 ק”מ מתל אביב. נניח שאין עקמומיות, אין אטמוספרה ואין הרים שמסתירים. אתה באמת מאמין שתראה בתל אביב אור של אצטדיון כדורגל מקפריסין?

    2. האור היה קבוע כל הזמן. הרי האור הוא מנצנוץ של הפנלים. הייתי מצפה שהאור ינצנץ כל הזמן, ובעוצמה שונה. הרי פעם הפנלים לא ממש לכיווני, ולכן הם צריכים לפעמים להאיר פחות, ולפעמים להאיר יותר. מה שראיתי זה נקודה חזקה קבועה שלא מתאימה לנצנוץ של פנל.

    3. והדבר שהכי מפריע לי – האור הזה נע ממש לאט יחסית ל-27,000 קמ”ש. הוא עובר מעלי כל שנייה 7.5 ק”מ. ממש מעליי בשיא (זווית בערך 85 מעלות) הוא היה לפחות חצי דקה – לפיכך ממש מעלי הוא נע מרחק של 225 ק”מ. חצי זה 112.5 ק”מ. אם אמרתי שאני במרחק 23 ק”מ מהים התיכון, אזי שהוא התחיל להיות ממש מעלי (בתחילת ה-30 שניות), הוא היה גם כ-90 ק”מ בתוך הים התיכון, שאין סיכוי שאראה זאת בפרספקטיבה שלי. רק את הנקודה הזאת – איך אתה מסביר זאת? זה ממש לא מסתדר.” סוף ציטוט
    אני אענה לך עליהם אבל קודם, שים לב לסגנון השאלות שלך, כולם מבטאים חוסר ידע, למשל בשאלה 2. כתבת “הייתי מצפה שהאור ינצנץ כל הזמן” מי אמר לך בכלל שלווין אמור לנצנץ? מאיפה הביטחון שלך? אתה שולט כ”כ טוב בנושאים כמו אופטיקה ולווינות כדי לדעת את זה? לא, פשוט ככה נראה לך, ואותו דבר שאלה 1. כתבת “אתה באמת מאמין שתראה בתל אביב אור של אצטדיון כדורגל מקפריסין?” אני לא, מאמין או לא מאמין, אני פשוט יודע שחוקי הפיזיקה מאפשרים את זה, אתה לעומת זאת לא מתבסס על ידע אלא זה לא נשמע לך. וכנ”ל לגבי שאלה 3. כל מה שאמרת זה רק רושם אישי, נראה לך שהתחנה זזה לאט מדי, אם היית עושה חישוב היית מגלה שהכל מסתדר מצוין… במדע המון פעמים האינטואיציה מכזבת, תראה איך 2,000 שנה האינטואיציה (והאמונה באריסטו) הטעתה את כולם לחשוב שגופים כבדים יותר נופלים מהר יותר מגופים קלים עד שבא ניסוי והפריך את זה לעין כל…

    עם “הוכחות” כמו שלך אפשר “להוכיח” כל דבר, הנה לדוגמא כמה “הוכחות” מסוג זה לכך שאין בעולם מטוסים שבאמת טסים, הכל שקר, ברגע שסוגרים את דלתות המטוס מגיע מנוף שלוקח את כל החלק המרכזי ומסיעים אותו בכבישים תת קרקעיים, החלונות במטוס הם באמת מסכים שמציגים כאילו נוף בגובה 10 ק”מ בעוד שהאמת היא שהמטוס מעולם לא עזב את הקרקע, עובדים על כולנו!!! :

    – כ”כ ברור שזה פייק… אתה רוצה להגיד לי שמטוס ששוקל מאות טונות מרחף באויר כאילו היה צמר גפן? כל ילד רואה שזה זיוף אחדגדול

    -איך יכול להיות שאנשים טסים ב-900 קמ”ש בלי להרגיש כלום??? הם מספרים לנו שלא צריך לחגור חגורות רק בהמראה ובנחיתה, נו באמת, אם המטוס היה טס במהירות מטורפת כזאת האנשים בפנים היו הופכים תוך שניות לרסק, ברור שהסיבה האמיתית היא כדי שלא ישימו לב לטלטולים של ההעלאה וההורדה ממשאיות הענק שסוחבים אותם, זה מסביר גם למה חברות ה”תעופה” מקרינים סרטים תוך כדי הטיסה הכל כדי לאנשים לא יהיה פנאי להתבונן יותר מדי ב”נוף מהחלון” ויגלו שזה בעצם מסך…

    -אם מטוסים באמת טסים, איך יכול להיות שיש מדי פעם התרסקויות מטוסים? הרי למטוס יש כנפיים ענקיות, לא? אז למה מטוס שנגמר לו הדלק או שקוראת לו תקלה במנוע לא ממשיך לגלוש כמו דאון כך שהגובה והמהירות יורדים אט אט? למה??? ברור! בטיסות שהתרסקו אנשים התחילו לחשוד שמשהו לא בסדר אז מהר פיצצו אותם כדי שלא יגלו את הסוד…

    -הייתי כמה פעמים בשדה תעופה לפני טיסה, אף פעם לא הרשו לי להסתובב סביב המטוס ולהסתכל עליו מקרוב, איך שמתקרבים אליו מהר מהר מכניסים את כולם פנימה וסוגרים את הדלת, מה יש להם להסתיר? וכו’ וכו’

    אתה מבין את הרעיון, שאלות כאלה (אבל למה? אבל איך? לא הגיוני ש…) הם אולי מענינות, אבל הם בשום אופן לא הוכחה לכלום, לכל היותר הם משהו להמשיך לברר ולשאול מומחים לנושא, (אלא אם כן אתה באמת מכיר את הנושא ויודע שהמצב הוא לא כמו שאמור היה להיות (למשל נצנוץ לווינים) זה משהו אחר, אבל גם אז צריך לראות את השאלה בהקשר כללי, כמות ועוצמת הראיות בעד מול כמות ועוצמת הראיות נגד).
    גם אני לא מבין כל מיני דברים בנושאי טיסה ומטוסים (וגם בנושאי חלל), אז מה? זה אומר שהם לא באמת טסים ושכולם עובדים עלינו?

    אצלך כל חוסר הבנה הופך מיד להוכחה שהכל שקר ושזה לא באמת קיים, אפשר להוכיח ככה רק אם אתה מכיר את התחום ויודע שזה לא אמור לעבוד, עד אז, 98% שזו רק חוסר הבנה שלך.

    הטענות שלך הם במקרה הטוב חוסר ידע ו/הבנה בנושא ובמקרה הגרוע טעויות/שקרים מוחלטים (אני לא חושד בך שאתה לא דובר אמת, אני מדבר על מקורות המידע שלך) כך למשל הטענה שאונייה לא נעלמת מתחת לאופק אלא רק קטנה ושבאמצעות זום מספיק גדול היא תתגלה שוב, זה פשוט טעות מוחלטת, יש בלי סוף סרטונים שמראים את זה (https://www.youtube.com/watch?v=7nUFLLUahSI&t=22s , https://www.youtube.com/watch?v=QVa2UmgdTM4 , https://www.youtube.com/watch?v=GrihjP5tTTM&t=29s , https://www.youtube.com/watch?v=0xWsuFLdgBs ),

    דרך אגב יש עוד ראיה פשוטה לכדוריות הארץ, קח איתך חבר למגדל כלשהו קרוב לים, תעמוד בקומה הכי גבוהה ותשאיר את חברך לצפות בשקיעה כמה קומות מתחתיך, אתה תראה שכשהוא ידווח לך שאצלו השמש נעלמה לחלוטין באופק, אתה עדיין תראה אותה, ובדיוק למשך הזמן שניתן לחשב שתראה אותה לפי קוטר כדור הארץ והגובה שלך מעל פני הים, אם אתה לא סומך על חבר שלך (אולי בזמן שעזבת אותו לשנייה הבונים החופשיים גייסו אותו…), אתה יכול לעשות אותו דבר בעצמך עם רחפן, הנה סרטון יפה שמדגים את זה: https://www.youtube.com/watch?v=xqAOsI2Ekf0

    יש לי גם תשובות קצת יותר ספציפיות אבל אל תשכח, גם אם לא היו לי זה לא היה משנה את העיקרון, אז ככה:
    1.נכון, 350 ק”מ זה המון (התחנה נמצאת כיום בגובה של כ- 405 ק”מ אבל לא משנה) אבל אתה שוכח שכשצופים למרחקים גדולים על פני הארץ האטמוספירה העבה בולעת ומפזרת הרבה מהאור בדרך ולכן משהו שנמצא מעבר ל- 20-30 ק”מ ייראה כהה ומעורפל, לעומת זאת כשמסתכלים לשמיים יש הרבה פחות אוויר בינינו לעצם הנצפה, אז נכון שאי אפשר לראות בעין את הצורה של התחנה מהמרחק הזה אבל אין שום סיבה שלא יהיה אפשר לראות אותה כנקודת אור.

    אמרת שאמנם ראית נקודת אור אבל מי אמר שזו אכן תחנת החלל הבינלאומית, א. יש אינספור אתרים שבהם
    מוצגים זמני מעברי הלווינים מעל כל מקום על פני כדור הארץ, כולל זמנים מדוייקים ומסלולם הנראה בשמיים מכל מקום שהוא, (הנה אתר לדוגמא שכולל גם עברית http://heavens-above.com/ ). בכל ערב אחרי השקיעה ובבוקר לפני הזריחה אפשר לראות בעין עשרות רבות של לווינים !, כולם מתאימים לתחזית שניתנה באתר זה ובאחרים, לכל עצם כזה יש שם, מספר קטלוגי, גודל פיזי (אם ידוע), מסלול מדוייק, ותאריך שיגור.
    אנשים רבים סביב העולם צילמו בטלסקופ את “מה שאמור להיות תחנת החלל הבינלאומית” כדבריך, בהגדלה רבה, במיוחד בעת שהיא עוברת מול השמש או הירח, שאז ניתן לראות את הצורה שלה באופן ברור ותנחש מה רואים בצילומים, הנה: https://www.youtube.com/watch?v=lepQoU4oek4

    בא תגיד אתה מה זה עוד יכול להיות, רחפן? בלון? הרי אנשים שנמצאים עשרות ומאות ק”מ ממך גם ראו אותה (כמובן ככל שהם היו רחוקים יותר ממנה היא נראתה להם נמוכה יותר בשמיים ומעט פחות בוהקת), כמו הכוכב הכי חזק ככה שאי אפשר לפספס אותה, אתה יכול לבדוק בעצמך באתר הנ”ל ולראות שאפילו באילת הרחוקה התחנה נראתה היטב והגיעה לרום 60° מעל האופק, באמצעות אתרים כאלה אנשים מצליחים לצלם את תחנת החלל ולווינים אחרים חולפים על פני השמש, זה דורש ידיעה של מסלול התחנה בדיוק עצום, אחרת פשוט לא היו מתקבלים סרטונים כאלה: https://www.youtube.com/watch?v=pDIPZFqfGGo , והנה אחד שצולם מישראל https://www.youtube.com/watch?v=cDckLwhb_7Q

    זאת ועוד, ישנם לא מעט חובבים סביב העולם שעוקבים להנאתם אחרי מסלולי לווינים (בטלסקופים, במכ”ם וכן הם מנסים לקלוט את שידורי הרדיו שלהם), אי אפשר להחביא שום דבר בחלל, הם מצליחים למצוא גם את המסלולים של לווינים מסווגים דוגמת מטוס החלל X37B או לווינים של צפון קוריאה וכדו’ לא מזמן למשל SPACEX שיגרה לחלל לווין סודי ביותר בשם זומה, אולי שמעת עליו, זמן לא רב אחרי השיגור החלו להתרוצץ שמועות שמשהו קרה לו ושהוא לא הצליח להגיע למסלול ואכן אותם חובבים אישרו זאת. זה חוץ מהמעקב הרשמי של נאס”א וגופים אחרים אחרי כלל העצמים במסלול סביב כדור הארץ כולל חלקי טילים וזבל חלל, הרשימה המלאה כוללת עשרות אלפי עצמים כאלה שעוקבים אחריהם באמצעות טלסקופים ומכ”מים ייעודיים, שזה כל מה שהם עושים.

    2. וואלה לא יודע בדיוק, שאלה מעניינת, אבל אני מניח שאור השמש שפוגע בתחנת החלל לא חוזר ישר לעינינו כמו ממראה אלא פשוט סתם מאיר אותה.

    3. זה נורא פשוט, הגובה של מסלול תחנת החלל הוא 405 ק”מ, אתה יודע לאיזה גובה מגיעה תחנת החלל בשמי דרום סיני (במעבר של ה- 14.02.18), 405 ק”מ דרומה מכאן? בדיוק 45 מעלות מהאופק…
    בקיצור, לסיום, יש כל כך הרבה ראיות איכותיות לכדוריות הארץ ששוחזרו כל כך הרבה פעמים, נגעתי רק בחלק, (אולי אפרסם בהמשך תגובה נוספת עם עוד הוכחות) אל תהיה מאותם אנשים שבמקום להסתמך על עשרות ראיות מבוססות ואיכותיות הם מתעלמים מהם ובמקום זאת מביאים לך רסיסי הוכחות שלא שוות דבר ובטוחים שהנה הם הגישו לך את ראיית הזהב…

  176. לקורא בידען

    הוכחת בסרטון השני את מה שאני מדבר עליו!

    שים לב- בסרטון השני תסתכל ב 00:00 – האם אתה רואה אונייה? אתה לא רואה.

    עכשיו בוא נגיד, באותו סרטון שאתה בחוף, והספינה יוצאת ממש ממך ממש מהמים הרדודים. נגיד שזה בסרטון 10:00- (מינוס 10 דקות).
    אם תעקוב אחרי הספינה בעיניים, אתה תראה אותה מתרחקת, בהתחלה הגוף ייעלם (נניח אנו בסרטון במינוס 5 דקות 05:00-), ולאחר מכן יישאר רק התורן ואז כל הספינה נעלמת (נניח במינוס אחד דקה 01:00-). הספינה נעלמה לך מהראייה. אתה מניח שהיא נעלמה בעקמומיות כדור הארץ. ככה הניחו כל המדענים מהעבר.
    והנה אתה מגיע בסרטון ל 00:00 – הספינה אתה לא רואה אותה!. אבל אם תשים מצלמה ותתקרב בזום אין– הפלא ופלא היא חזרה. כמו שרואים בסרטון בזמן 00:10.

    זה הכל עניין של פרספקטיבה. דברים מתרחקים ונעלמים לא בגלל העקמומיות.

    ניסים, ניסים,ניסים
    אני שואל אותך משהו אחד, אתה עונה לי משהו אחר. כרגיל. מה הקשר?
    איך אתה מסביר את זה שרואים את הארץ מאחורה רועדת, ואת המכונית לא? אם נניח המצלמה רעדה, אז גם המכונית וגם הארץ היו צריכות לרעוד ביחד!
    כן, איך?
    מתן

  177. ניסים – בקשר לפולריס, אל תסתכל במלל באנגלית אלא במסגרת מצד ימין.
    בכותרת צהובה השלישית רשום “אסטרונומיה”
    בשורה ה-4 רשום “מרחק”
    רשום 323-433 שנות אור

  178. מתן
    וכדי בכל זאת לסתור את השקר של הסרט הזה (השקר שאתה מפיץ, לא הסרט עצמו):

    המצלמה ממוקמת על זרוע שמחוברת למכונית. שים לב (אם אתה מסוגל…) שרואים צל שנע על המכונית כשכדור הארץ נע.

    אני כיסיתי את ה”חוב” שלי. תורך. ובלי לשקר שוב, בסדר? צילומים שרואים פתאום אוניה אחרי האופק? אתה חושב שכולנו סתומים ? 🙂

  179. מתן
    אענה על הכל – אחרי שתודה שאתה משקר בהקשר של אוניה בים – בתמונות בהגדלה רואים בדיוק מה שרואים בעין. האוניה אכן נעלמת אחרי האופק.

    אחרי זה תסביר לי למה האופק שאני רואה יורד כשאני טס גבוה.

  180. פרסמתי כבר 2 תגובות היום שלא פרסמו. מבקש שיפרסמו אותם.
    ניסים, אתה ממש פוגע בביטחון העצמי שלי…..ממש…….
    אבל הנה בשבילך – ההוכחה האולטימטיבית שהטסלה לא בחלל!
    רק 2 דקות. תראו.
    https://www.youtube.com/watch?v=PrpGXfwyibI
    בזמן שהטסלה חגה סביב הארץ, התמונה של הארץ רעדה מספר פעמים, אבל המכונית לא! אם היא בחלל והמצלמה רועדת, אזי גם האוטו יזוז ביחד עם הארץ. אבל לא. רק הארץ רעדה, והמכונית לא.
    כן, אנא מצא הסבר פיזקלי בבקשה איך הארץ רועדת מאחור אבל האוטו נשאר במקומו. שניהם אמורים לרעוד ביחד.
    אלא אם כן…..מה שרואים מאחורה זה הקרנה. סורי, אבל המכונית עדיין פה איתנו על הקרקע.

    תמשיכו להאמין לשקרים שמוכרים לכם שספייס איקס שיגרה לחלל את הטיל ההיסטורי.
    תמשיכו להאמין לשקרים שמוכרים לכם כבר 500 שנה.
    וניסים – אני עדיין מחכה לתשובה ממך איך פיזו הבחין בעינו ב-1 שן מתוך 18,744 שיניים שעברו בשנייה אחת, או בזמן 0.000053 שניות.
    מתן

  181. אלבנצו
    לצערי, אני מכיר אישית אנשים שמאמינים בשטויות האלה. לחשוב שכדור הארץ שטוח זה סתם מעיד על פיגור שיכלי ולא מזיק, אבל – זה מגיע להכחשת ההתחממות, טיפול הומיאפטי בילדים או מניעת חיסונים – וזה כבר רשעות מסוכנת.

    מתן הוא באמת טרול, אי אפשר להיות כזה אהבל, אבל יש אנשים שקוראים כאן שהם תמימים, וכל עוד לא ניתן למחוק תגובות מהסוג של מתן, לא כדאי להשאיר את התגובות האלה בראש הרשימה 🙂

  182. למה תגובות מתעכבות כאן יום שלם מה קורה פה?

    מתן חפש ביוטיוב:

    Ship Horizon

    ותראה הרבה דוגמאות יפות שסותרות את דבריך בקשר לאניות.

  183. ואתה צודק על זווית הנטייה של כדור הארץ – התכוונתי לרשום 23.4 ולא 24.4.
    אבל התוצאה שרשמתי כן הייתה נכונה “66.6” ובאמת 90 פחות 23.4 שווה 66.6…….

  184. ניסים

    הלכתי בלילה לרופא והוא אבחן שיש לי תסמונת אחרת – תסמונת פרדי קרוגר……שאני מופיע בחלומות של אנשים ומפחיד אותם על עובדות אמיתיות והאמת מפחידה אותם……

    קרא בעיון כל דבר שרשמתי. שים לב איך לא הבנת/פיספת את הבנת הנקרא שלי:
    1. ספינה נעלמת שאתה בחוף בגובה פני הים ולא בזכרון יעקב שאתה כבר בגובה רב!. מי אמר זאת ראשון – אריסטו! וכל שאר המדענים של העולם ההליוצנטרי כל הזמן השתמשו במודל הזה. אז לך לחוף הים – תעקוב אחרי יאכטה עד שנעלמת לך מהעיניים לאחר 5-6 ק”מ, הסתכל עליה בטלסקופ והיא תיראה לך מחדש.

    2. נוסחת עקמומיות – טסת הרבה, אתה מומחה בלווינים, ולא יודע מה זה? על בסיס זה המדע קבע העולם שלנו עגול. הנוסחה היא מייל בריבוע * 8 אינצ או ק”מ בריבוע * 8 ס”מ. (סליחה 7.8 ס”מ כי אמרת שאני לא מדוייק…..). ככל שאתה מתרחק, העקמומיות גדולה והיא נקראת drop.

    3. פולאריס – לא נכון מה שרשמת. יש לו 2 מרחקים כמו שרשמתי! תבדוק בקישור
    https://en.wikipedia.org/wiki/Polaris

    4. רשמתי:
    “פאי הוא מספר חשוב בטבע. פי הוא חיתוך הזהב”. שים לב רשמתי בשני “פי”. זוהי האות פי Φ. קח תקרא
    https://he.wikipedia.org/wiki/%D7%A4%D7%99

    5. רשמתי בפירוש שמש באנטרקטיקה!. ורשמתי שבקוטב הצפוני יש 24 שעות שמש ראה את ההערה שרשמתי בסוגריים:
    “(כמו שבחודשי יוני-אוגוסט יש שמש 24 שעות בקוטב הצפוני)”

    6. משקל סגולי של 1.2 ק”ג למ”ק – מה הקשר הניסוי שלך בוואקום? המטבע נפל כי…….המשקל סגולי שלו גדול יותר מ 1.2 ק”ג למ”ק!
    קח בלון יום הולדת מלא בהליום ושחרר אותו באוויר – הוא יעלה למעלה לשמיים כי המשקל הסגולי שלו קטן. אז איך הכוח הגרביטציה לא מצליח למשוך אותו למטה? כי מה שמשחק תפקיד זה המשקל הסגולי.

    7. מטוטלת פוקו – זה שהיא נעה כל הזמן זה לא אומר כלום – זה קשור הרבה רק לדבר אחד – האלסטיות של הכבל שכל הזמן צובר אנרגיה ולכן היא ממשיכה. הבעייה במטוטלת פוקו היא….שצריך לתת לה התנעה! אתה לא יכול להעמיד אותה במקום ושהיא תתחיל פתאום לנוע. למה צריך לתת לה את הדחיפה הראשונה? הרי כדור הארץ מסתובב ואמור לסובב אותה, לא? הדחיפה הראשונה נותנת לה את האנרגיה לנוע וביחד עם האלסטיות של הכבל.

    8. תמונה שחור לבן – דה, ברור שיש גם אפור. אבל איך מתמונה שחור לבן הוא קבע שהלבן הזה או האפור הזה הוא פי 7000 לבן או אפור מהנקודה השנייה – זוהי הבעייה! ועל בסיס זה קבעו את המרחק שלנו לגלקסיה. כן, תמשיך להאמין שמצילום שחור לבן (ועם גווני אפור כמובן) אתה יכול לקבוע בהירות.

    9. קוונדיש – לא נכון מה שרשמת. הייתה אוניברסיטה בארה”ב שעשתה זאת והיא הייתה הכי קרובה – אבל המשרעת של התטה (הזווית שנוצרת מהתזוזה) הייתה גדולה מאד יחסית לקוונדיש. וכל שאר הניסויים שעושים זה עם כדורי מתכת שיש בהם חשמל סטטי או קרינה אלקטרומגנטית. קח 2 בתים גדולים בעץ, תתלה אותם בחבל, שהמרחק ביניהם הוא רק 10 ס”מ, ללא רוחות, אתה באמת חושב שהבתים מעץ ינועו אחד אל השני?

    לשאלה ששאלת אותי בנוגע שאתה עולה למעלה – גם בעולם שטוח האדמה מתרחקת וככל שאתה עולה למעלה קו האופק – המפגש של השמיים עם האדמה – יורד.

    שאלתי אותך אבל באמת תגיד את האמת כי אף פעם לא פגשתי מישהו שטס בגבהים כאלה: שטסת 20 ק”מ בשמיים, האם ראית את קו האופק ישר (גם אם הוא בזווית נמוכה יותר אבל ישר) או הבחנת בעקמומיות כלשהי?

    מתן

  185. מתן
    לא ענית לשאלה שלי.

    כתבת “שהספינה נעלמת לך מעבר לאופק בגלל עקמומיות כדור הארץ, אם אתה מסתכל בטלסקופ או במצלמה עם הגדלה – אתה רואה את כל הספינה שוב! מלא”

    גרתי שנים בזכרון יעקב ויש לי עשרות תמונות של אוניות בים, כולל בזום מאד גדול. אתה לא מתבייש מתן? אתה אולי סתם טרול?

    כתבת “חומר שהמשקל הסגולי שלו קטן מ 1.2 ק”ג למ”ק (משקל סגולי של אוויר בגובה פני הים) – עולה למעלה באוויר (כמו בלון הליום), חומר שהמשקל הסגולי שלו גדול מ 1.2 ק”ג למ”ק יורד למטה. ))”
    אז …. אםםםם …. איך גופים נופלים בוואקום? כשהייתי סטודנט הפלנו מטבע ונוצהר במיכל מרוקן – נפלו למטה יפה מאד….

    טיילתי בנורווגיה ובצפון קנדה, גם בקיץ וגם בחורף. השמש מסתובבת סביבך במשך 24 שעות, לפעמים מעל לאופק ולפעמים מתחת, בהתאם לעונה ולשעה.

    ניסוי קוונדיש בוצע מאז אלפי פעמים. למה אתה משקרת מתן?

    באחת הכנסיות של בולוניה יש מטולטלת פוקו. הסתכלתי עליו במשך שעות (זה שיגע אותי – ניסוי מדעי בכנסיה??). ראיתי גם מטולטלת דומה במוזיאון בוושינטון. שניהם נעים בקצב של סיבוב כל 24 שעות.

    ודרך אגב – כדור הארץ מסתובב סביב צירו ב-23 שעות ו-56 דקות. 🙂

    וואו – אתה ממש טיפש. לוח צילום שחור לבן לא מציג תמונה בשחור לבן! הוא מציג גווני אפור, החל משחור ועד ללבן. אף פעם לא ראית תמונה בשחור לבן? אם אתה תגיד שלא – אדע שאתה שקרן. אם תגיד שכן – אדע שאתה טיפש. מה תשובתך?

    עכשיו למתמטיקה המביכה שלך.
    פאי הוא לא יחס הזהב ואין לו כל קשר ליחס הזהב.
    מהירות כדור הארץ הממוצעת היא 65,526.5 מייל לשעה
    כדור הארץ נוטה על צירו 23.44 מעלות
    נוסחת העקמומיות? מה זה בכלל?
    המרחק לפולאריס הוא קבוע – אנחנו לא יודעים את המרחק הזה (מסיבות שללא בגרות לא תבין). לקחת סתם שני מספרים ועשית חישוב מטופש.
    דן בראון כתב ספרים בדיוניים – טוב שאתה לא מאמין שיש הוביטים (אולי אתה כן).

    מתן – בבקשה – קרא קצת על תסביך דאנינג-קרוגר ולך לקבל עזרה!!!

    האם אתה טוען שאני שקרן? שאבין … 🙂

  186. לכולם

    אני חוקר רק עובדות או עובדות היסטוריות ולא נכנס בכלל לאנקדוטות וקונספירציות. שום דבר מדעי ממה שאומרים לנו לא מסתדר. נקודה.
    לפני שאתחיל, רק בשביל לעורר את המוח של כולם, בואו ארשום לכם 2 קונספירציות המבוססות על עובדות אמיתיות (לאחר מכן נשים זאת בצד את זה ונדבר בהמשך רק על עובדות):
    1. ורנר פון בראון, אבי הטילים, הנאצי שערק לאמריקה, כתב בשנת 1952 את הספר המדעי “פרוייקט מאדים”. ואיך קראו למנהיג ממשלת מאדים – Elon. זה שם משנת 1952. שם זה הוא גם השם של היזם של ספייס איקס Elon Musk שמוביל את המסע לירח. עכשיו – זוהי עובדה. הקונספירציה אומרת שאלון מאסק הוא למעשה בובה על חוטים שתוכננה כבר לפני 16 שנה שספייס איקס נולדה.
    2. קפלר שהגה את 3 החוקים המפורסמים ביסס את עבודתו על העבודה של טיכו ברהה שחלה בפתאומיות ומת בשנת 1601. הסיבה הייתה סיבוך בשלפוחית השתן. כ-400 שנה אחרי בשנת 1991 ו-1996 בדקו את השפם שלו שנשאר בקבר, וגילו שהבן אדם מת מהרעלת כספית. כלומר גילו לאחר 400 שנה שמישהו הרעיל אותו. עד כאן עובדות.
    מי שלא ידע, טיכו ברהה האמין בעולם הגיאוצנטרי שהארץ במרכז. הקונספירציה אומרת שמי שהרוויח הכי הרבה מהמוות שלו הוא……קפלר, כי קפלר לקח את עבודתו ושינה אותה 180 מעלות לעולם הליוצנטרי.
    עד כאן קונספירציות.

    עכשיו נדבר על נתונים ועובדות שלא מסתדרים (יש לי עוד 50 כאלו):
    1. בתאריך 31 בינואר 2018 (לפני שבועיים) היה ליקוי ירח חלקי. המוזר הוא שהצל שעל הירח היה על החלק העליון של הירח (!) ולא על החלק התחתון. הרי למדנו במדע שבליקוי ירח חלקי כדור הארץ עושה צל על הירח בחלק התחתון. אז איך המציאות מראה שהחלק העליון מוצל? זה לא מסתדר פיזית. לא יכול להיות מצב שכדור הארץ עושה צל וזה בחלק העליון. מצורפים תמונות:
    https://www.facebook.com/ronhagberg/posts/10155891202491166
    יתר על כן, הרי למדנו כי אריסטו אמר לפני אלפי שנים שאיך יודעים שהעולם עגול – כי רואים צל מעוגל על הירח בזמן ליקוי ירח.
    איפה הבעייה? מתברר לי היום שגוף א’ שעושה צל על גוף ב’ – הצל מקבל את הצורה של גוף ב’ ללא קשר לצורה של גוף א’! כלומר, אם מקל ישר עושה צל על כדור – הצל על הכדור מתעגל. ושאתה רואה את הצל המעוגל הזה האם המקל הוא מעוגל? ממש לא, הוא ישר. מה רוצה להגיד – שמה שאריסטו לימד אותנו, ומה שכולנו יודעים שזוהי הוכחה לכדור הארץ עגול – פשוט לא נכון.
    2. מי לא מכיר שאיך יודעים שהעולם עגול – כי שספינה מתרחקת מאיתנו היא מתחילה להיעלם בגלל עקמומיות כדור הארץ, קודם נעלם גוף הספינה ולבסוף התורן. זה מה שלמדנו כל חיינו. הוכחה ניצחת לעולם עגול.
    איפה הבעייה?
    שהספינה נעלמת לך מעבר לאופק בגלל עקמומיות כדור הארץ, אם אתה מסתכל בטלסקופ או במצלמה עם הגדלה – אתה רואה את כל הספינה שוב! מלא! הרי למדנו שהיא נעלמת מעבר לאופק. אז איך רואים את הספינה? ואז בטלסקופ אתה רואה שהספינה נעלמת שוב מעבר לאופק. אתה עושה הגדלה יותר גדולה ואתה רואה את הספינה שוב! איפה העקמומיות?
    ואז אתה מבין כי כל העניין הוא פרספקטיבה – הדברים נעלמים כי הם מתרחקים מגבול הראייה שלך ולא בגלל עקמומיות. ואם תשים מכשיר ראייה עם הגדלה – תראה הכל שוב.

    3. גוגל ארת’ – לכו בתוכנה לחלק הדרומי הדרומי של כדור הארץ (סמנו בתוכנה רשת קורדינטות) ושתגיעו לקודקוד תראו משהו מוזר – יוצאים סטריפים משולשים מהקודקוד הדרומי בצורה מלאכותית. אתה שואל את עצמך למה? הרי יש לפחות מאות לוויינים שעוברים מעל אנטרקטיקה מצפון לדרום למה הם לא צילמו זאת? למה זה מלאכותי?
    (בעולם שטוח אין קוטב דרומי, אנטרקטיקה היא מסביב לכל הדיסק)
    4. בחודשים דצמבר-פברואר כל שנה מספרים לנו שיש שמש של 24 שעות באנטרקטיקה (כמו שבחודשי יוני-אוגוסט יש שמש 24 שעות בקוטב הצפוני). וכאשר אתה בא לחקור את הנושא אתה מסתכל במצלמות של time laps (פריים אחד מייצג קבוע זמן גדול) אשר נמצאות בבסיסים באנטרקטיקה, לדוגמה כאן במחנה דיוויס. שאתה מסתכל בסרטונים אתה שם לב שהצל מסתובב במשך 20 שעות ואז התמונה נעצרת, ומשעה 00:00 ועד שעה 04:00 אין תנועה של צל אלא הכל קבוע.
    https://www.youtube.com/watch?v=Q3XfKL0W844
    שתבינו – הנקודה הזאת היא קריטית כי זה שחור ולבן – בעולם עגול יש 24 שעות שמש בקוטב הדרומי נקודה. אם חסרים 4 שעות, כמו במקרה הזה, יש פה בעיה במודל.
    (יש עוד סרטון שמציג לכאורה 24 שעות של שמש אבל רואים ששמו בו שכבות של כותרות, ועדיין לא באמת רואים את השמש 24 שעות).

    להלן דברים נוספים שקשורים לדברים מדעיים שפשוט לא מסתדרים:
    1. הכל התחיל עם ארטוסתנס שבמאה ה-3 לפני הספירה עשה את הניסוי המפורסם שלו ומדד את היקף כדור הארץ. בגלל שקרני השמש הם קווים מקבילים, הוא מדד ביום מיוחד את הזווית שבין 2 מקומות באסואן ובאלכסנדריה וככה מדד את היקף כדור הארץ בסטייה של 15% בלבד. איפה הבעיה? יש פה פרדוקס. איך הניח שקרני השמש מקבילים? אנחנו “יודעים” שקרני השמש מקבילים כי השמש פשוט ענקית בקוטר של 1.4 מיליון ק”מ לעומת כ-12.8 אלף בלבד של הארץ. אבל בעולם המושגים של אז, איך יכל להסיק בכלל שזה מקביל? זה לא היה בכלל במערכת התפיסה וההבנה שלו (זה כמו שתסביר למישהו מהמאה ה-15 על אייפון). ואפילו ש-300 שנה לפניו שפיתגורס ואפלטון שהסיקו שאולי העולם כדור, גם הם לא הבינו את הגודל העצום של השמש ושקרני השמש מקבילים. לכן איך בכלל ארטוסתנס הניח שקרני השמש מקבילים? ללא הנחה זאת כל הבדיקה שלו לא רלוונטית.

    2. ניוטון פרסם את ספרו בשנת 1684 ובו הציג את עולם הגרביטציה ואת המשוואה F= G*m1*m2 / r^2 . איפה הבעיה? הוא לא ידע את G הקבוע הקוסמולוגי והוא אף פעם לא מדד את כוח הגרביטציה. למעשה, עד שנת 1874 בו פורסם הניסוי של קוונדיש (לאחר 76 שנה שהניסוי נעשה בפועל), במשך כ- 200 שנה אף אחד לא ידע למדוד כלום כי לא היה את G. אף אחד גם לא מדד כוח בינינו ובין הכוכבים ואף אחד לא מדד כוח בין 2 עצמים בארץ. הכל היו השערות ללא הוכחות. גם היום, את הכוח הזה לא ניתן למדוד באף מכשיר. המדענים אומרים שהוא קיים, אף אחד לא יכול להוכיח זאת.
    ((הדבר היחידי הידוע שהיה ניתן למדוד פיזית הוא ה g התאוצה לכדור הארץ של 9.8 מטר בשנייה. את זה כמובן ניוטון ייחס לגרביטציה, אבל יש משתנה עיקרי למה דברים נמשכים לארץ או עולים למעלה – רק בגלל צפיפות החומר (!) ואין לזה קשר לגרביטציה – חומר שהמשקל הסגולי שלו קטן מ 1.2 ק”ג למ”ק (משקל סגולי של אוויר בגובה פני הים) – עולה למעלה באוויר (כמו בלון הליום), חומר שהמשקל הסגולי שלו גדול מ 1.2 ק”ג למ”ק יורד למטה. ))

    3. ניסוי קוונדיש בשנת 1798 למציאת מסת כדור הארץ ו-G הקבוע הקוסמולוגי – תיארתי לך בהערות הקודמות על אופן הניסוי, ואגיד לך מה הבעיות:
    א. קוונדיש עשה את הניסוי לבד. אף אחד אחר לא היה נוכח בניסוי, אף אחד לא ידע שהוא עושה את הניסוי, אף משרד מדעי רשמי לא נכח, כלום. אם הייתי בא אליך עם ניסוי שלי ונתונים ואף גורם רשמי או אדם אחר לא היה נוכח – היית מקבל את הנתונים שלי?
    ב. קוונדיש הסתכל על הניסוי עם הטלסקופ ממרחק מאות מטרים כי הוא לא רצה שמשיכת גופו תשפיע על התוצאות. ומה עם המוטות, ומה עם תשתית העץ שהחזיקה את הניסוי, זה לא השפיע? מספיק השפעה קטנה שכל המדידות לא יהיו רלוונטיות. מזכיר לך את בעיית שלושת הגופים (תיאור תנועתם ההדדית של שלושה גופים תחת השפעתו של כוח המשיכה) שאין לה פיתרון אנליטי אלא רק במקרים מיוחדים.
    ג. הוא לא הציג את הניסוי שלו בצורה רשמית או מדעית עם הסברים מעמיקים. רק 76 שנה אחרי הניסוי שלו מצאו את הרשימות שלהם ופורסמו ב- 1874.
    ד. במהלך ה-150 שנה אחרונות ניסו פעמים רבות לבצע את ניסוי קוונדיש ללא הצלחה. חלק מהאוניברסיטאות כן ניסו לעשות זאת אבל זווית θ Δ שבדקה את הזווית של הזזת המוט הייתה גדולה מאד יחסית לניסוי המקורי. וככל שהזווית יותר גדולה ככה פחות מדויקת.
    ה. אף פעם לא הצליח ניסוי של 2 גופים גדולים קרובים מעץ או פלסטיק (ללא חשמל סטטי או אלקטרומגנטיות) שהראה שיש תזוזה ביניהם או שנמדד כוח ביניהם.

    4. מדידת מהירות האור על ידי פיזו בשנת 1849 – כבר רשמתי על זה באריכות – אין סיכוי שפיזו זיהה בעינו שעל הטלסקופ שעברה 1 שן מתוך 18,744 בשנייה, או שעברה 0.000053 שניות. אין סיכוי שהוא מדד זאת. אין. אף עין אנושית לא יכולה לזהות זאת. יש לזה חשיבות רבה שהמדידה הראשונית האמיתית שלא באמצעות כוכבים או פרלקסה לא הגיונית לזיהוי. ויש לזה חשיבות יותר גדולה כי איינשטיין ביסס את תורת היחסות על בסיס המדידות של האור שהיו ידועות לו ב-1900.

    5. מטוטלת פיקו 1851 – על פי המדע המטוטלת של פיקו נעה בצורה מסויימת בגלל ההשפעה של סיבוב כדור הארץ. להלן הבעיות:

    ו. אם היא נעה בגלל סיבוב כדור הארץ, למה היה חובה לתת לה דחיפה ראשונית? למה אי אפשר היה לשים אותה במצב נייח ואז היא הייתה מתחילה לנוע בגלל סיבוב כדור הארץ?
    ז. לא היה אף עקביות לתנועה – פעם איטית, פעם מהירה, פעם בכיוון השעון, פעם נגד כיוון השעון.
    ח. והדבר הכי חשוב – המטוטלת הייתה תלויה על ידי חוט שהיה לו מתיחות. אולי המתיחות של החוט השפיעה על התנועה? ככל שהחוט יותר אלסטי, אזי התנועה יותר רחבה, מהירה וגדולה. מתברר שפיקו לא דיבר על זה בכלל. (חשוב להשוות זאת לקוונדיש שהשווה את “הגרביטציה” לפיתול של המוט עם הכדורים הקטנים, ולמעשה התנועה של המוט (θ Δ) זזה וחזרה זזה וחזרה. אז איך פיקו ואף אחד אחר לא התייחס למתיחות של החבל?)

    6. מדידת מרחקים לכוכבים על ידי האבל 1923 –האבל מדד את המרחקים לכוכבים על בסיס היחס בין בהירות נראית ובהירות מוחלטת. הבעייה היא שהוא עשה זאת באמצעות לוח צילום שחור לבן. איך הוא ידע לזהות מלוח שחור לבן הבדלים בבהירות, איך? מזכיר כי הקפאיד שבחר היה פי 7,000 מבהירות מוחלטת מהשמש שלנו.

    7. אין אף מדידה במכשיר מדידה שנעשתה על גבי הארץ שאנו נעים 1,600 קמ”ש בתנועה סיבובית.
    8. אין אף מדידה במכשיר מדידה שנעשתה על גבי הארץ שאנו נעים 107,000 קמ”ש מסביב לשמש.
    9. אין אף מדידה במכשיר מדידה שנעשתה על גבי הארץ שאנו נעים 500,000 קמ”ש עם השמש מסביב לשביל החלב.
    10. אין אף מדידה במכשיר מדידה שמדדה את הגרביטציה בינינו ובין השמש, בינינו ובין הירח.
    11. אין אף מדידה במכשיר מדידה שמדדה את הגרביטציה בין 2 גופים שנמצאים בעולם.
    12. אין אף מדידה במכשיר מדידה שמדדה את המרחק של הארץ והירח שלנו (רק 380 אלף ק”מ)(ואלה שאומרים שמדדו עם לייזר זה פשוט לא נכון כי גם הלייזר הכי משוכלל הקרן שלו תתבזר בזמן מוקדם מאד).

    ולקינוח :
    13. פאי הוא מספר חשוב בטבע. פי הוא חיתוך הזהב. אבל יש עוד מספר חשוב בטבע של ההליו-צנטרי והוא….666. צא ולמד:
    א. המהירות של כדור הארץ סביב השמש היא…..66,600 מייל לשעה.
    ב. כדור הארץ נוטה על צידו 24.4 מעלות. אם תחסיר זאת מזווית ישרה 90 מעלות תקבל…..66.6 מעלות.
    ג. נוסחת העקמומיות היא על כל 1 מייל drop של…. 0.666 רגל
    ד. המרחק של כוכב הצפון פולאריס מאיתנו הוא בין 333 לבין 434 שנות אור (פעם הכי קרוב ופעם הכי רחוק) או 1,938,000,000,000,000 לבין 2,604,000,000,000,000 מיילים. ההבדל בין הנקודה הרחוקה לנקודה הקרובה….. 666,000,000,000,000 . (אמיתי – גם אני לא האמנתי ובדקתי בעצמי את המספרים).
    ה. המטוס הראשון של נאס”א 15-X היא המטוס הראשון שהגיע לגבול החלל בתחילת שנות ה-60. היו רק 3 מטוסים כאלו. מה היה מספר הסידורי שלהם?
    66670
    66671
    66672……..
    ו. וה-G הקבוע הקוסמולוגי הוא רק 6.67 * 10^-11 – לא היה כבר נעים להם אז קבוע קרוב ל 6.66……..
    לא מוזר שהמספר 666 מופיע בעולם ההליוצנטרי של שיטת המדידה האימפריאלית?

    ולשאלות איך יכולים אלפי אנשים לדעת את הסוד הזה? התשובה היא אחת – יש תנועה שמונה מיליוני אנשים, הרבה מהמנהיגים הכי גדולים בעולם נמצאים בה, והם שולטים באמת בעולם. עד לפני שנה הכרתי את התנועה הזאת רק מהסרטים של דן בראון, אבל מתברר שהתנועה הזאת קיימת 500 שנה, יש לה סמלים רבים שכל מדעני המודל ההליוצנטרי היו חברים בה (קופרניקוס, גלילאו, קפלר, ניוטון, האבל ועוד רבים), היא הייתה אחראית למהפכה הצרפתית והיא הקימה את הציונות (!) במאה ה-19 (אני עדיין בשוק מזה).
    אני יודע שזה מטורף, אבל לכו לבית המשפט העליון שבירושלים ותראו את הפירמידה וכל שאר הסמלים שלהם שם.
    כמובן שאני לא צריך לציין כי כל כל האסטרונאוטים חברים בה, יש להם טבעות של האחווה הזאת, והם הצטלמו עם הסמלים של האחווה הזאת.

    העובדות לא מסתדרות.

    מתן

  187. מתן
    בבקשה – בוא תסביר לי משהו שאני לי מבין.

    כשאני טס בגובה נמוך – אני רואה את קו האופק “בגובה שלי” כלומר – בזווית 0.
    כשאני טס ב-30 אלף רגל, קו האופק יורד ב-3 מעלות. זה קורה בכל מקום שאני טס, ובכל כיוון שאני מסתכל.
    כשאני טס ב-50 אלף רגל, קו האופק יורד ל-5 מעלות (בערך).

    איך זה יכול להיות?

  188. לפלוני 321

    הלכתי לגבעה חשוכה במקום מגוריי 250 מטר מעל פני הים – משם אני רואה את הבניינים של תל אביב. 23 ק”מ ממני זה הים מאחורי הבניינים. בימים יפים אני רואה גם את הים.

    מה ראיתי – ראיתי אור קבוע עובר מעליי במשך 6 דקות. בערך מגובה 30 מעלות מעל האופק שמעל הים ועד 30 מעלות ממעל האופק מהצד השני.

    מחזה יפה מאד.

    אבל,

    התחנה מרחפת מעלי בגובה 350 ק”מ. הפנלים הם 100 מטר על 50 מטר (גודל של מגרש כדורגל). מהירות תחנת החלל היא 27,000 קמ”ש או 7.5 ק”מ בשנייה. היא עפה במרחב שלי 6 דקות שזה 360 שניות, בזמן הזה היא עשתה 2,700 ק”מ מעלי.

    הנתונים לא מסתדרים בגלל כמה סיבות:
    1. איך אני מזהה אור בגודל מגרש כדורגל 350 ק”מ ממני? ניקוסיה בקפריסין במרחק של כ-350 ק”מ מתל אביב. נניח שאין עקמומיות, אין אטמוספרה ואין הרים שמסתירים. אתה באמת מאמין שתראה בתל אביב אור של אצטדיון כדורגל מקפריסין?

    2. האור היה קבוע כל הזמן. הרי האור הוא מנצנוץ של הפנלים. הייתי מצפה שהאור ינצנץ כל הזמן, ובעוצמה שונה. הרי פעם הפנלים לא ממש לכיווני, ולכן הם צריכים לפעמים להאיר פחות, ולפעמים להאיר יותר. מה שראיתי זה נקודה חזקה קבועה שלא מתאימה לנצנוץ של פנל.

    3. והדבר שהכי מפריע לי – האור הזה נע ממש לאט יחסית ל-27,000 קמ”ש. הוא עובר מעלי כל שנייה 7.5 ק”מ. ממש מעליי בשיא (זווית בערך 85 מעלות) הוא היה לפחות חצי דקה – לפיכך ממש מעלי הוא נע מרחק של 225 ק”מ. חצי זה 112.5 ק”מ. אם אמרתי שאני במרחק 23 ק”מ מהים התיכון, אזי שהוא התחיל להיות ממש מעלי (בתחילת ה-30 שניות), הוא היה גם כ-90 ק”מ בתוך הים התיכון, שאין סיכוי שאראה זאת בפרספקטיבה שלי. רק את הנקודה הזאת – איך אתה מסביר זאת? זה ממש לא מסתדר.

    זה רק על קצה המזלג. ארשום בהמשך על שאר הדברים.

    ניסים – אם אתה עדיין איתנו – האם שטסת בגובה 20 ק”מ זיהית כבר את עקמומיות כדור הארץ?

    מתן

  189. למתן ולכל השאר, דיברתי עם נאס”א והם ארגנו לנו מעבר מושלם של תחנת החלל הבינלאומית ? היישר מעלינו מחר – רביעי 14.01.18 היא תראה בשמיים כמו כוכב חזק מאוד יותר מכל כוכב אחר בשמיים, והיא תנוע באיטיות מאופק לאופק במשך כ-10 דקות, אפשר לראות אותה בקלות בעין, לא צריך משקפת או משהו. ניתן יהיה לראות אותה מכל מקום בארץ, (מהצפון והדרום הנתונים שונים אבל ממש בקצת, אולי דקה או שתיים).

    להלן הפרטים: תחנת החלל תעלה ב 17:55:00 (עיגלתי את השניות) באופק בכיוון צפון מערב – אזימוט 315°
    כעבור 5 דקות ב 18:00:17 התחנה תהיה ממש מעלינו, מעלות בודדות ממרכז השמיים ממש, אז היא גם תאיר בעוצמה הרבה ביותר מגניטודה 3.9- (האור הוא לא עצמי אלא היא פשוט מוארת מהשמש)

    ומשם בקו ישר היא תרד אל האופק הנגדי תוך שהיא חולפת דרך קבוצת אוריון -קרוב מאוד לביטלג’וז- דרך אגב תחנת החלל מאויישת ברציפות כבר 17 שנה וכעת שוהים בה 6 אסטרונאוטים.

    בהצלחה.

  190. למתן הנכבד, (בקשר לשתי התגובות הראשונות שלך) שמת לב שאתה סותר את עצמך? אתה מנסה להביא ראיה שאין לווינים מכך שבצילום של הטסלה אין נצנוצי לווינים ולא כוכבים אלא הכל שחור, רגע אבל אם ככה אז גם שיגור הטסלה הוא זיוף אז אי אפשר להוכיח ממנו כלום. ודרך אגב עשרות אלפי אנשים ראו בעיניהם את שיגור הפאלקון הוי, ובכלל מה אתה חושב שעושים בדיוק בספייס איקס וכל חברות הטילים והלווינים האחרות??? יושבים רגל על רגל ומנסים לעבוד על אנשים כאילו הם משגרים לווינים? למה שהם יעשו את זה בכלל? מי משלם להם לשבת ולא לעשות כלום (חוץ מלעבוד על כולנו, כן?)…

  191. י. גרין (אח של בריאן)?

    התשובה שלך מאלפת. עכשיו תקרא אותה שוב אבל הפעם תחשוב שהיא באה כתשובה למי שמעלה שאלות ותמיהות לגבי היהדות. מה אתה אומר?

  192. היי מתן, עצה קטנה: מכיר את זה שמישהו שואל אותך חידה קשה מאוד, אתה חושב וחושב ונראה לך שאין דרך בעולם לפתור אותה

    (דוגמא לחידה כזו: נגיד שמכניסים אותך -מתן- ואת חברך לחדר ואומרים לך כזה דבר, שים לב, בחדר הבא ישנם 100 קופסאות קרטון קטנות ממוספרות מ-1 עד 100, בתוך כל קופסא יש פיסת נייר עם מספר כלשהו מ1 עד 100 אך פיסות הנייר מפוזרות באקראי בקופסאות בלי התאמה בין המספר שעל הקופסא למספר שעל פיסת הנייר, כעת אומרים לך, עליך להכנס לחדר הבא, שבו נמצאים הקופסאות, לעשות שם משהו (שכבר נסביר מהו) ולצאת, ורק אז, אחרי שתצא החוצה מהבניין, תערך הגרלה על מספר מ1 עד 100 וחברך יצטרך להכנס לחדר ולמצוא את פיסת הנייר עם המספר שהוגרל תוך פתיחה של לא יותר מ50 קופסאות, אתה יכול לסכם איתו לפני ההגרלה מה שאתה רוצה וכן אתה יכול להשאר כמה זמן שבא לך בחדר אבל כל מה שמותר לך לעשות שם זה לפתוח קופסאות ולראות מה המספר על פיסת הנייר שבתוכו, וכן להחליף בין 2 פיסות נייר (רק 2 ובלי לכתוב או לסמן כלום , זה נשמע כאילו אין ולא יכול להיות פיתרון, הרי מה זה משנה מה אני יעשה אם רק אחרי שאצא יוחלט איזה מספר חברי יצטרך למצוא!!!)

    אבל חבר שלך אומר שיש גם יש תשובה לחידה, בסיטואציה כזו יש מין דחף ללכת עם הראש בקיר ולהאמין שאין תשובה לחידה, הרי זה נראה כ”כ בלתי אפשרי!
    כאן צריך לקחת פסיעה אחורה ולחשוב בהיגיון, נכון שאני לא מצליח להעלות בדעתי שום פיתרון שהוא וזה נראה בלתי פתיר, אבל יש כאן מישהו (אמין, שאני סומך עליו) שאומר לי שיש אז יש, יש הפתעות בחיים ולא הכל אני יודע.

    ועוד משהו, יש לך מושג כמה אנשים עובדים בתעשיית החלל ומסביב? אסטרונומים, אסטרונאוטים, טכנאים, מהנדסים שבונים לווינים, ועוד ועוד, מאות אלפי אנשים ברחבי העולם אם לא יותר, אנשים מוכשרים, מומחים, רציניים, אתה חושב שהם לא מכירים את השאלות שלך? אתה באמת חושב שהאסטרונומים וכו’ הם טיפשים מוחלטים שלא חושבים על השאלות שהעלת? או שמאיימים על כולם שישתקו? זה לא רציני.

    יש לך מושג איזו סנסציה זו תהיה אם מדענים יגלו שכדור הארץ הוא שטוח/אין לווינים/המצאה אחרת, מי שיצא נגד “קשר השתיקה” יזכה לכבוד ולפרסום עולמי!

    חשבת פעם למי יכול להיות אינטרס לעבוד על כולנו כבר מאות שנים???

    בקיצור אני באמת ממליץ לך לשאול ולהתייעץ את המומחים לנושא ולא עם איזה מומחים מטעם עצמם שמעלים טענות הזויות. השאלות שלך טובות, אבל אם היית טורח לשאול אנשים שלמדו את התחום בצורה רצינית ומתעסקים בו יום יום היית מקבל תשובות מפורטות ומספקות (או לחילופין הם היו משתכנעים…), במקום זה בחרת לחשוב, אה, אני לא מוצא תשובה לשאלות שלי אז זהו אני צודק, אין לווינים והכל שקר.

  193. לאתר הידען
    שלחתי 2 תגובות בצהריים ואתם ממדרים אותי. זה לא יפה.
    זה אתר מדעי ואנחנו מדברים פה על מדע ועל הבעיות במדע
    אני מבקש אם תוכלו להעלות את 2 ההודעות האחרונות שלי לניסים
    כמו כן, יש לי עוד דברים לכתוב
    אנא תנו לי להגיב
    תודה
    מתן

  194. ניסים -שאלה קטנה – שטסת בגובה 20 ק”מ האם ראית את עקמומיות כדור הארץ? או שבגובה כזה עדיין לא רואים עקמומיות? (בטיסה של 10-12 ק”מ לא רואים גם לא מהקוקפיט (חבר שלי טייס).

  195. שלום ניסים
    זה הגרסה הקצרה. בערב ארחיב ואעיר על הערותיך.
    לפני שנה פחות 4 ימים קיירי אירווניג שחקן כדורסל מה NBA אמר “עובדים עלינו, העולם שטוח”. לא הפסקתי לצחוק ולגחך. אני חובב אסטרונומיה ופיזיקה ופשוט צחקתי כמו שבטח 99.9% מהאנשים ששמעו את ההערה שלו צחקו.
    אבל, כמה שזה היה נראה לי מוזר, התחלתי לחקור את הנושא. אני חוקר את הנושא לא מהצד של “האם העולם הוא שטוח” אלא מהצד השני “האם העולם הוא עגול באמת”.
    ותאמין לי, אחרי כ-500 שעות של חקירה בספרים ישנים, בניסויים ישנים, בהיסטוריה של המדע, וניתוח אירועי חלל live שקורים, אני סמוך ובטוח שאנו חיים בעולם שטוח ולא בגלובס.
    כן, תצחק, תגחך, תבטל. אני רגיל לזה אבל עדיין ממשיך עם האמת שלי.
    התחלתי לחקור את ההיסטוריה של המדע ההליוצנטרי (השמש במרכז) בין השנים 1500 (קופרינקוס) ועד 1923 (האבל) כי אז לא היו מחשבים, לייזרים, GPS, מכ”מ (היה רק בראשיתו) וכו.

    וחשכו עיניי! כל, אבל כל המדע שקשור למודל ההליוצנטרי היה רק השערות. שום הוכחה. אני נתתי לך טעימה על פיזו משנת 1849, ואתה דיברת על מכ”מ ופולסים, אבל מה לעשות בשנת 1849 לא היה מכ”מים ופיזו התבסס על העין שלו שהייתה בטלסקופ והוא הבחין ב-1 שן שעברה (מתוך 18,744 שיניים שעברו בשנייה אחת) וככה הוא חישב את מהירות האור. לא על ידי מכ”מ. אלא על ידי עין אנושית בשנת 1849. ואז אחרי שאתה מחשב שעבר זמן 0.000053 שניות אתה מבין שיש פה בעיה בניסוי שלו. אין סיכוי שהוא מדד זאת.
    ואז אתה מוצא שניוטון קבע בשנת 1684 את המשוואה האלמותית של הגרביטציה (f=G*m1*m2/r^2), אבל ה- G הקבוע האוניברסלי “נמצא” רק בשנת 1874 לאחר 200 שנה !!! ואף אחד לא הצליח למדוד את ה-F הכוח. אז איך אפשר לקבוע תיאוריה שהמשתנה המרכזי שלו לא נמצא בכלל?
    ואז אתה בודק את הניסוי של קוונדיש שמצא את ה-G הקבוע הקוסמולוגי ומתברר שהוא עשה את הניסוי לבד לבד ללא מישהו אחר שהיה נוכח, ללא מדען אחר שהיה נוכח וללא אקדמיה שהייתה נוכחת. ואז 80 שנה אחרי זה מצאו את הרשימות שלו ו”וואלה”, נמצא ה-G שחיפשו 200 שנה.

    שום דבר לא מסתדר.

    יש לי עוד הרבה דברים לכתוב ארשום זאת בערב.
    מתן

  196. מתן
    אם אתה הוא זה שחוקר את האמת – אז המדענים שחושבים אחרת ממך הם שקרנים? בוא נעבור על ה”עובדות” שלך ונראה.

    כן – אני מאמין באמת שמכונית אכן מרחפת בחלל. בפרוייקט הזה התעסקו מאות ואלפי אנשים. האם כל אלה משקרים שהתקינו את הטסלה בטיל? שילמו להם? איימו עליהם? לא יותר קל וזול פשוט לשים שם מכונית? עכשיו – הטיל שוגר והגיע לחלל, והוא לא התפוצץ. לזה יש מספיק עדים.
    אז איך יכול להיות שהמכונית לא בחלל? איפה הוא? אלון מאסק הוא באמת איש נחמד וגאון, אבל הוא לא קוסם 🙂
    האם המכונית בדרך למאדים? אין לי כל סיבה לחשוב אחרת. חלליות הגיעו לשם לפני 45 שנה. אז – למה לא עכשיו?

    נאס”א אינה, בשום דרך, הבעלים של ספייס-אקס. נאס”א היא לקוח של ספייס-אקס, ובמקרה הנוכחי, השאילה ציוד וכוח אדם לטובת הפרוייקט.
    אם לא הבנת – נאס”א משלמת לספייס-אקס.

    הטמפרטורה בחלל נעה בטווח רחב, בין 300 מעלות למינוס 250 מעלות (תלוי בחשיפה לשמש). לחלקי מתכת של הרכב זה לא משנה, וגם הצבע עמיד בחום גבוה. בקשר לצמיגים – גם גומי עמיד בחום גבוה: ראיתי גלגלים שהיו כל כך חמים שהם היו אדומים! ולגומי לא קרה כלום. גומי של צמיגים מיוצר בחום גבוה, ואני לא רואה סיבה לחשוש במקרה של הטסלה שלנו.
    בפועל – החום גם לא גבוה, אסטרונאוטים מסתובבים בחלל בחליפות ללא שום בעיה.

    אני טסתי הרבה בחיים, גם בגבהים של 20 ק”מ. שום דבר לא קופא במטוס – מאיפה המצאת את השטות הזאת? מה שכן קופא זה מים שנוזלים מהמטוס (מה שנקרא קרח כחול). למטוסים יש מערכות מיזוג שעומדות בקלות בתנאים קיצוניים – טיסה בגובה נמוך ב-1200 קמ”ש, ועד לטיסה בגובה 20 ק”מ – בלי שום מאמצים. אולי זה לא ברור לך – אבל אף אחד לא טוען שמאסק נוהג את כל הדרך למאדים…

    האתר שהפנית אליו זה אחד הדברים המטומטמים שראיתי בחיים שלי 🙂 ומי שקורא באתר הזה הוא עוד יותר מטומטם. להגיד שג’ודי רזניק חיה? הם לא מתביישים!?!?!?!? איזה זבל אתה קורא?

    חגורות ואן אלן לא עושות נזק לרכיבים מוקשחים. הטסלה לא מוקשחת, אבל אין כל כוונה שהרדיו שלו יעבוד במאדים. החשיפה של האסטרונאוטים לקרינה הזו הייתה קצרה – הם קיבלו קרינה בכמות של ראד אחד (הרבה פחות מסריקת CT אחד), כשמותרת חשיפה שנתית של 5 ראדים לשנה (התקן האמריקאי).

    נכון – האסטרונאים אמרו שהם לא הרגישו כלום. הסכנה היא לא מיידית. גם ב-CT אתה לא מרגיש כלום.

    בקשר לצילום לווינים – בוא נעשה חשבון. הלווין נע 7.5 ק”מ בשניה, נכון (אני שמח שיש לך מחשב כיס!) בוא נניח גובה של 300 ק”מ. כלומר יחס של 1 ל-40. בוא ניקח מטוס צילום. טס בגובה 20 ק”מ, במהירות 0.5 ק”מ לשניה – כלומר גם יחס של 1 ל-40. היחס חשוב כי מה שמעניין זה מה הזווית שעצם נע על הקרקע בזמן צילום. – בלווין ובמטוס צילום העצם נע באותו זווית.
    עכשיו, 1 ל-40 נותן זווית של בערך מעלה וחצי (אלמד אותך טריק של טייסים: עצם בגודל מטר במרחק 60 מטר מכסה זווית של מעלה אחת).
    כלומר – המצלמה צריכה להסתובב לאחור במהירות של מעלה וחצי בשנייה כדי לפצות על המהירות. זה נראה לך הרבה?
    למה בכלל הזכרתי מטוסי צילום? כי – בעבר ביצעתי הרבה מאד צילומים בגובה 20 ק”מ, במהירות של חצי ק”מ לשנייה – ולמרבה הפלא: המצלמה שלנו הייתה מצלמה שנועדה ללוויין!!! 🙂

    אמרת פיקסל בגודל מטר. נניח שזה נכון, ונניח תמונה של 4000 פיקסל. זה אומר תמונה של 4 ק”מ צלע. 2 תמונות לשנייה – ואנחנו מסודרים. במטוסי הצילום שמרנו על חפיפה של 56% בין תמונות, ולכן צילמנו מספר גדול יותר של תמונות לשנייה. אז גם את העניין הזה סגרנו.

    הלוויין צורך יחסית מעט חשמל (במיוחד לוויני צילום. לוויני תקשורת צורכים יותר, וגם זה בערך עד 100 וואט בסה”כ). הפנלים שלו הרבה יותר יעילים מהפנלים שעל הבית שלך, הם מקבלים קרינה הרבה יותר חזקה (אין אוויר בחלל … גם זה חדש לך?) – והם מכוונים במדוייק לשמש!!!

    לא הבנתי מה אמרת על צלחות יס. תקשורת ע”י החזרה מהיונוספירה מתבצעת בתדרים נמוכים של בין 3 ל-30 מגה-הרץ ולכן צריך אנטנות גדולות. גם בישראל היו אנטנות כאלה (חלקם היו מעל מכתש רמון).

    שוב אתה מערבב את ההיגיון שלך במציאות … מה יהיה איתך, מתן?? למה נראה לך 45-60 מעלות? כמו שהסבירו לך הלווין של יס נמצא מעל לקו המשווה. התדר של יס הוא בסביבות 10 גיגה-הרץ, תדר שבו אין החזרה מהיונוספירה, אבל מצד שני – האנטנה הרבה יותר קטנה. עשיתי חישוב מהיר – יצא לי שהאנטנה של יס צריכה להיות 8-9 מעלות (בהנחה שהלווין שלהם נמצא באותו קו רוחב כמו ישראל. אחרת הזווית תהיה קצת גדולה יותר).

    כן, מתן – יש הסבר פיסיקלי להחזרה של גלי רקיע מהיונוספירה. אם הייתה לך השכלה פיסיקלית הייתי מסוגל להסביר לך בדיוק.
    יש משהו מעניין בנושא הזה – בגלל שזה החזרה, אז יש שטחים מתים בקליטה. כלומר אתה קולט בטווח מסויים, מתרחק ולא קולט, ואז מתרחק עוד יותר ושוב קולט. ולפעמים יש גם החזרה מהקרקע – ואז שוב יש איזור מת ולאחריו איזור קליטה. מעניין, נכון?

    כן – אני רואה לווינים בעיניים. נדייק – אני רואה החזרים של השמש מלווינים.יש לי אפליקציה שאומרת מתי לווינים עוברים מעלי, ואני רואה אותם הרבה, כשאין עננים כמובן.
    עין האדם קולט פוטון בודד. הוא רואה נר ממרחק 20 ק”מ. ההבזק מלוויני אירידיום למשל, הוא הרבה יותר חזק מזה. לעיתים – הלווינים האלה מקלקלים צילומים שאסטרונומים מבצעים דרך טלסקופים.

    ודרך אגב הנה צילום יפה בשבילך – https://qz.com/1058842/solar-eclipse-2017-photo-the-iss-just-pulled-off-the-photobomb-of-a-lifetime/

    חבר שלי עשה צילומים דומים, כך שאתה יכול להיות בטוח שזה לא פייק.

    עולם הליוצנטרי שלי? מה אני קשור לעניין?

    אתה לא יכול לרשום שורה אחת “פייק” בקשר לתחנת החלל. אתה כן יכול לשקר לאורך 5 עמודים. מה יוצא לך מזה?

    קולטים GPS בלב ים, במרחק אלפי ק”מ מכל אנטנה סלולורית, אז די עם זה. לא קולטים GPS במבנה, למרות שיש קליטה סלולרית. עובדות אתה קורא לזה 🙂 🙂

    מתן – אל תייחס את חוסר ההבנה שלך במדע למציאות. אתה סובל מתסמונת שנקראת דאנינג-קרוגר.

    הניסו של פיזו הוא פשוט. הוא ידע בערך מה מהירות האור, מהחישובים של אול רוימה. בנוסף, הוא שינה את המהירות של המכשיר בטווח גדול. אספר לך סיפור שיכול להבהיר:
    המכ”מ של מטוס הפנטום משתמש במהירות האור כדי לחשב טווח למטרה. שולחים פולס קצר ואז המכ”מ מתחיל למדוד זמן. אני חושב שאפילו אתה מסוגל להבין את זה. נניח שהמכ”מ שלי משדר פולס כל אלפית שניה. זה אומר שהטווח שאני יכול לראות מטרה זה מהירות האור חלקי 1000, ואז חלקי 2. כלומר – 150 ק”מ. עכשיו – תניח שיש מטוס גדול בטווח 200 ק”מ. המכ”מ יקלוט את ההד אחרי 1.333 מילי-שניות. אבל – המכ”מ יחשוב שזה הד מפולס מאוחר יותר, ולכן יחשוב שעבר רק 1/3 מילישניה. ואז – המכ”ם יציג לי מטרה במרחק 50 ק”מ.
    למה סיפרתי את זה? כי יש דרך להתגבר על הבעייה: משנים מעט את הזמן בין הפולסים, ואז אפשר לקשר בין פולס יוצא לפולס חוזר. וזה מה שעשה פיזו!!!

    עכשיו – מה קשור כאן תורת היחסות?

    מתן – מה אתה מנסה להגיד? בלי סיפורים, ובלי שקרים – מה אתה רוצה להגיד?

  197. שלום ניסים
    אני לא מתעסק בקונספירציות. בשנה האחרונה אני מתעסק בחקר האמת על המודל ההליוצנטרי.
    כל מה שאני רושם לך זה עובדות. לא ניחושים או גחמות, כמו שכל המדענים עד שנת 1950 ניחשו.
    בנוגע לדברים שרשמת יש התייחסות בסוף.

    ההיגיון שלי לא משהו? לפי הניסיון שרשמת שיש לך, האם אתה מאמין באמת, אבל באמת, שכרגע מכונית מרחפת מסביב לכדור הארץ ולאחר מכן בדרכה למאדים?
    אני אגיד לך למה אני מתעכב דווקא על המכונית המגוחכת. אנשים חושבים שספייס איקס היא חברה פרטית של אלון מאסק, אבל האמת היא שנאס”א היא גם הבעלים של ספייס איקס! זוהי חברה בבעלות נאס”א. היא מביאה לה מיליארדי דולרים כל שנה – לכן המבצע הזה הוא לא מבצע גחמה של יזם מוצלח, אלא יוזמה ושיתוף פעולה של נאסא.

    לעניינו, שצופים בסרטון של המכונית, קשה להאמין שזה אמיתי. זה נראה כל כך פייק. וציינתי זאת קודם שגם מאסק במסיבת העיתונאים אמר “זה אמיתי כי זה נראה כל כך פייק”. ועוד אמר במסיבת העיתונאים כי ” לא עשו במכונית אף שינויים בחומר, ולא עשו ניסויים לבדוק אם החומר יעמוד בתנאי החלל”.
    עכשיו, בוא אשאל אותך שאלות ותענה לי (כי רשמת שלי אין הגיון):
    ואם יש נתון שלא דייקתי – אנא ציין זאת:
    1. הטמפרטורה בחלל מסביב לכדור הארץ היא 200-250 מעלות צלזיוס. איך, אבל באמת באמת איך, המכונית, שמאסק אומר שלא עשו לה התאמות בחומרים, יכולה לעמוד בטמפרטורה הזאת? אתה באמת מאמין לזה? תכניס את המכונית הפרטית שלך לתוך תנור גדול של 200-250 מעלות – איך הריפוד לא נשרף? איך הפח לא מתעקם? (העיקר שרואים את השתקפות כדור הארץ דרך הווקס של הדלתות…….) נוסיף את העובדה שאם אתה לא בצד של השמש, אזי הטמפרטורה הרבה הרבה מתחת ל-0 באזור מינוס 273 צלסיוס. אבל נפרגן ונגיד רק מינוס 100. אז איך גם האוטו לא קופא??? (על אף שהוא נע מהר? לידיעה, מטוסים שטסים בגובה 10-12 ק”מ ברגע שהטמפרטורה בחוץ מגיעה פחות ממינוס 60-70 אזי חלק מהמערכות במטוס מתחילות לקפוא, לכן גם שנעים מהר עדיין בטמפרטורה נמוכה הדברים מתחילים לקפוא )

    (דיאגרמה שמראה את הטמפרטורה לפי גובה)
    https://aplanetruth.info/2015/04/15/30-how-do-satellites-survive-4000f-degree-heat-in-space/

    2. חגורת ואן הלן – המכונית אמורה להמשיך לחלל העמוק. איך היא יכולה לעבור את חגורת ואן הלן שנמצאת מסביב לכדור הארץ במרחק של 700 ק”מ עד ל -10,000 ק”מ? החגורה כולה קרינה אלקטרומגנטית שיכולה ליינן כל מעגל אלקטרוני.
    את האמת, אתה באמת מאמין שהמכונית של מאסק יכולה לעבור את החגורה הזאת? תראה בפרוייקט הדגל של נאסא “האוריון” כמה מחשבות ומשאבים הם משקיעים כדי לעבור את הקרינה הזאת.
    זה פשוט מגוחך. ואני לא ההגיוני……
    (אנקדוטה – אז איך בשנת 1969 חללית אפולו עברה את חגורת ואן אלן? לא היה להם מיגון בכלל כמו שנאסא מתכננת היום. כלום. הם בכלל לא חשבו על זה. יתר על כן, ששאלו כמה שנים לאחר מכן את האסטרונאוטיים “איך הם עברו את החגורה?” הם הגיבו שלא הרגישו כלום ואולי זו לא הייתה בעייה).

    שאלות נוספות על לווינים:
    3. לווין צילום נמצא בגובה נמוך של 200-300 ק”מ ונע במהירות של 27,000 קמ”ש או 7,500 מטר בשנייה!!! שוב – כל שנייה הוא נע 7,500 מטרים. עכשיו, איך הוא יכול לצלם משהו על פני כדור הארץ? אם הוא שם מצלמה על נקודה כלשהי – אחרי שנייה הנקודה התרחקה קצת פחות מ-7.5 ק”מ! (רשמתי קצת פחות כי הרדיוס של הלווין גדול ב-200-300 ק”מ יותר מרדיוס כדור הארץ).
    מספרים לנו כל חיינו על צילומי לווין, ושלווין יכול לצלם את הקרקע עד דיוק של 1 מטר. בוא נעשה חשבון:
    אם הוא מצלם את צומת גלילות, לאחר 1 שנייה הוא כבר יהיה במחלף השלום, לאחר עוד 1 שנייה יהיה במרכז בת ים ולאחר עוד 1 שנייה יהיה באיקאה ראשון ולאחר עוד 1 שנייה יהיה בא.ת. יבנה. מה רוצה להגיד? ב-4 שניות בלבד הוא עבר מרחק של צומת גלילות עד יבנה. איך איך הוא יכול לצלם משהו במהירות כזאת?
    4. נוסיף על זה את העובדה, שבשביל לראות מרחק של 200-300 ק”מ צריך עדשות כל כך ארוכות. איך כל זה נכנס בלווין של כמה מטרים בלבד? ושוב, מספרים לנו שלוויינים מצלמים את הבתים והרחובות (אני לא מדבר על צילום גבוה שתופס מפתח של ק”מ).
    (יש חברת Vexcel ובשיתוף של מיקרוסופט היא שמה מצלמות מיוחדות על מטוסים קלים ומצלמת סטריפים, והתוכנה של מיקרוסופט הופכת זאת ל……מבט מלמעלה של רחובות ובתים. בדיוק כמו……גוגל ארת’. אז איך אומרים שגוגל ארת’ מצלמת מלווין את כל הרחובות והבתים מלמעלה, זה לא מסתדר במהירות של 7,500 מטר בשנייה, אבל עם מטוס בגובה נמוך צילומים אלה עושים שכל
    http://www.vexcel-imaging.com/

    5. הלווין צורך חשמל רב, ולפיכך יש לו פנלים סולריים שמטעינים את הסוללה. לי יש בגג הבית פנלים סולריים. הפנלים הסולריים שלי מכסים גג של 100 מ”ר. אני מאמין שללווין ממוצע יש 10 מ”ר של פנלים.
    מספר מאפיינים לפנלים:
    א. צריך הרבה מטרז’ כדי להפיק קצת חשמל.
    ב. הזווית מאד מאד חשובה – אם השמש קצת בזווית, היעילות יורדת פלאים.
    ג. טמפרטורה של 20-25 מעלות היא נצילות אופטימלית. דווקא ב-40 מעלות היעילות יורדת בעשרות אחוזים!
    עכשיו, לאור הנ”ל, לא מבין איך הקצת הפנלים שנמצאים על הלווין, לא בזווית, בטמפרטורה של 200-250 מעלות, או במינוס שהפנל קופא יכולים לספק חשמל במשך 5-10 שנים לכל מערכות הלווין?
    ומה בנושא הננו-לווינים? יפן שיגרה לפני שבוע ננו לווין לחלל במסה של 3 ק”ג בגודל של 37 ס”מ. איפה יש לה מקום לסוללה? איפה יש מקום לפנלים להטעין? והכי חשוב – איזה עדשת צילום אתה יכול להכניס ללויין כזה קטן? ואני הלא הגיוני…….

    אלה עיקרי הקושיות שלי,

    להלן תשובות ל-4 הערותיך:
    1. צלחת ייס –
    שמעת על White Alice Communications System ?
    https://en.wikipedia.org/wiki/White_Alice_Communications_System
    בשנות ה-50 חיל האוויר האמריקאי (אותו חיל האוויר ששולט ב (GPS עשה ניסויים באלסקה אודות אורך קליטת גלים. לאחר שפרס צלחות “לווין” גדולות (כן, כמו הצלחות הגדולות בעמק האלה) הוא שיגר גלי רדיו לזווית בשמיים, והגלים פגעו ברקיע/שמים/אטמוספרה וניתרו חזרה bounce” ” לצלחת אחרת. ככה הם הצליחו לשדר אותות רדיו למרחקים של מאות ק”מ. הדבר הצריך אותם לכוון בצורה מדויקת את הצלחת. אם תזיז את הצלחת בכמה מעלות, זווית הקפיצה מהרקיע לא תגיע לצלחת הרחוקה ולא תהיה קליטה.
    היום קוראים לשיטה הזאת זירוי טרופוספירי:
    https://he.wikipedia.org/wiki/%D7%96%D7%99%D7%A8%D7%95%D7%99_%D7%98%D7%A8%D7%95%D7%A4%D7%95%D7%A1%D7%A4%D7%99%D7%A8%D7%99
    רגע, מה זה מזכיר לנו? רגע, מה רואים בתמונות? רואים….צלחות דמוי צלחות ייס! בשנות ה-50? לא הגיוני.
    מתברר כי הצבא האמריקאי הבין שכיוון צלחות לכיוון מסויים לשמיים גורם להעביר תקשורת ל-500 ק”מ.
    שים לב לעוד משהו מיוחד – תמיד לא הבנתי מה ההיגיון של צלחות ייס שהזווית שלהן היא רק 10 מעלות. זה לא הגיוני. הרי הם צריכים לפנות לשמיים בזווית של לפחות 45-60 מעלות. ואז מסבירים לנו שהצלחות מכוונות בזווית ללוויין תקשורת מעל קו המשווה ולכן הזווית היא חדה מאד 10 מעלות. אנא תסתכל בתמונות של “זירוי טרופוספירי” ושל המבצע באלסקה בשנות ה-50. כל הצלחות הזווית שלהן היא…..חדה מאד כ-10 מעלות.
    דבר אחרון – אין שום הסבר הגיוני בעולם שלך ההליוצנטרי למה האותות מנתרים חזרה. אין. האם תוכל להסביר לי למה?
    2. א. האם אתה באמת רואה לווינים בעיניים?
    בוא נעשה חשבון קטן – נניח עולם אוטופי ללא הרים ועמקים, ללא עקמומיות וללא אטמוספרה. אתה נמצא בתל אביב ואני נמצא באילת 300 ק”מ ממך. אצלך חושך ואצלי אור. אני מחזיק ומזיז פנל סולרי של אורך 10 מטר (אני מפרגן לך גודל יותר גדול מהפנלים של הלווינים…..). השמש פוגעת ומנצנצת על הפנל שלי.
    אתה רוצה להגיד לי שאתה תזהה את הנצנוץ שלי? ואני הלא הגיוני…..
    אין סיכוי ידידי. אין סיכוי שאתה תראה נצנוץ מ-300 ק”מ!!!
    כל אלה שרואים לווינים חולפים מהר באור לבן בשמיים זה דבר לא הגיוני. לפיכך אתה לא רואה לווינים בשמיים. אתה כן רואה מטוסים שיכולים לנצנצץ, אתה כן יכול לראות בלונים שיכולים להאיר או לנצנץ. אין סיכוי שתראה נצנוץ קטן ממרחק של 300 ק”מ.

    ב. בנוגע לתחנת החלל – אני יכול לרשום לך 5 עמודים על כל האנומליות של תחנת החלל. יש כל כך פייק בדבר הזה. אולי בהמשך הדו-שיח ארשום על זה.

    3. GPS:
    הוכחתי לך שמשטרה מאכנת סלולר רק על בסיס 3 אנטנות סלולריות ולא לווינים. אם כך, ה GPS עובד גם על אנטנות סלולריות.
    הראיתי לך על זירוי טרופוספירי שאנטנות הסלולריות (שים לב שיש אנטנות סלולריות עם קופסאות מלבניות עומדות ויש גם עם צלחות קטנות) גם יכולות לעבוד ככה

    4. החזרים מלווינים – עד שלא תפרט לאיזה מרחק זה איני יכול להתייחס לכך.

    שורה תחתונה – אני בכוונה מדגיש את המכונית של מאסק כי נאס”א מעורבים בזה, ולמדתי בחיים שאם מישהו משקר לך פעם אחת, אז הוא משקר לך הרבה.

    אם נמשיך את הדיונים, בשנה האחרונה חקרתי לעומק את ההיסטוריה של העולם הליוצנטרי בין השנים 1500 ועד 1950 ולשום דבר אין הוכחה או מדידה. הכל השערות, ניחושים ושקרים. אתן לך טעימה קטנה – אראה לך קושייה קטנה ובו נראה אם תמצא הסבר שמניח את הדעת – מדידת מהירות האור.
    מהירות האור כידוע היא 300,000 ק”מ בשנייה – הראשון ש”מדד באמת” הוא המדען הצרפתי ארמן פיזו – מתוך ויקיפדיה:
    “בשנת 1849 הוא היה האדם הראשון לחשב את מהירות האור על כדור הארץ. הוא העביר קרן אור דרך גלגל שיניים, הקרן עברה 8 ק”מ עד שהגיעה למראה שהחזירה אותה, אז הקרן חזרה אותם 8 ק”מ עד לגלגל השיניים שבמשך זמן התנועה הסתובב מהר מספיק כדי שהקרן תחלוף בפער הבא של השיניים ותגיע לעינו של הצופה. באמצעות חלוקת המרחק שעברה הקרן בזמן שלוקח לגלגל השינויים “לעבור” שן אחת, חושבה מהירות האור להיות 315,000 ק”מ בשנייה, סטייה של כ-5% מהערך הנכון (כ-300,000 ק”מ בשנייה). לאחר מכן הוא פרסם תוצאות ניסיוניות ראשונות שחושבו בשיטה שפיתח למדידת מהירות האור ”

    איפה הבעיה?????? קרן האור עוברת 16 ק”מ בזמן של 0.000053 שניות!!! שוב 0.000053 שניות!!! לפני הניסוי, הוא שם 720 שיניים בגלגל וסיבב אותו במהירות של 1562 סיבובים לדקה (1562 RPM ).עכשיו בוא ותגיד לי איך הוא מדד 0.000053 שניות? אפילו היום קשה למדוד זאת. ואז מסבירים שהוא שם לב ש-1 גלגל שיניים הסתובב וזה היה בדיוק הזמן של מהירות האור. עכשיו, בוא נחשב כמה גלגלי שיניים עוברים בשנייה – 720 * 1562 / 60 שניות = 18,744 שיניים עברו בשנייה אחת! אז איך הוא שם לב שעבר גלגל שיניים אחד? או איך מדד 0.000053 שניות? זכור – בשביל למצוא מהירות חייבים לדעת דרך וזמן. לא ניתן לקבוע מהירות ללא מדידת הזמן. אם היה רואה 2 שיניים אז המהירות הייתה רק 150 אלף ק”מ לשנייה. ואם היה רואה 4 שיניים (הזמן בשניות הוא 0.0002…….) אז המהירות הפלא ופלא רק 75 אלף ק”מ לשנייה.
    לכן אשאל אותך שוב – איך הוא הבחין בעינו 1 גלגל שיניים או זמן של 0.000053?
    החשיבות לניסוי הזה גדולה כי 50 שנה אחרי זה איינשטיין על בסיס המדידות הללו ומדידות אחרות ביסס את תורת היחסות. הוא לא נעזר במחשבים או לייזרים שלא היו קיימים.

    בברכה

    מתן

  198. מתן
    אני יודע בוודאות שיש לווינים בגלל הרבה סיבות.

    1. אני מכוון את האנטנה של יס ללויין וקולט, אני מזיז בכמה מעלות ומאבד קליטה.

    2. אני רואה לווינים רבים בעין.
    א. המסלול של תחנת החלל הבינלאומית ידוע, וניתן לראות אותו אם הוא עובר מעלינו זמו קצר לאחר שגיאה
    ב. יש סוג לווין שנקר מגלן שיש לו קולטים גדולים והוא במסלול נמוך. קל מאד לראות אותם בשעות מסויימות – אם אתה יודע איפה ומתי. יש אפליקציה לטלפון שאומר לך מתי עובר לווין שניתן לראות בעין. יצא לי לראות לווינים כאלה גם ביום!

    3. יש לי GPS. אני יודע בדיוק איך המכשיר עובד כי התעסקתי בתחום הזה בעבר.

    4. ראיתי החזרים מלווינים במכ”מ מספר פעמים. לא אפרט…

    שים לב שתנאי לראות לווין זה שאתה תהיה בחושך והלווין יהיה באור. יש טווח זמנים קצר יחסית שבו זה קורה.

    יש משהו כמו 1100-1200 לווינים פעילים – ומחציתם בהקפה נמוכה. ה”שטח” בה נמצאים לווינים כאלה הוא כ-200 מילון קמ”ר. כלומר, בכל ריבוע של 300 אלף קמ”ר יש לווין אחד בממוצע. זה שטח פי 50 מישראל. ובנוסף – לא לכל לווין יש קולטי שמש גדולים.

    מתן – סלח לי, אבל ההיגיון שלך לא משהו 🙂 אתה רוצה להאמין בקונספריציות טיפשיות, אז שיהיה לך בכיף. רק אל תקשר בין אמונות כאלה להיגיון. או לאינטליגנציה….

  199. שלום ניסים ושלום גם גם לכותב המאמר אליסף

    אני רשמתי את ההערה הקודמת למה לא רואים לוונים.

    אני לא מבין – אשמח שתענו לי:
    -יש כרגע מכונית שחגה סביב הארץ במרחק של 250-300 ק”מ והיא אמורה לחוג יותר גבוה עד שתצא מהאורביט של הארץ
    -אלפי לווינים (רוב הלווינים הקיימים) חגים בגובה הזה
    -תחנת החלל חגה בגובה 350 ק”מ
    – לכל לווין יש כנפיים סולריות שאמורות לנצנץ למרחק רב

    אז למה אתה לא רואה אף נצנוץ של לויין?? לא רואה אף לווין? לא רואה את תחנת החלל?

    אתן לך דוגמה: אתה עכשיו בצהריים בשדה פתוח של 10-20 קמ על 10-20 קמ. בשדה אני פורש מאות מראות בגודל 10 ס”מ כל אחת ומזיז אותן. אתה רוצה להגיד לי שלא תראה אף נצנוץ??? אתה לא תראה אף מראה אבל תראה עשרות/מאות נצנוצים.

    ונניח שהמצלמה מצלמת רק פריים אחד לשנייה, אם יש אלפי לווינים אז לפחות היא תקלוט 1 נצנוץ מ-1 לויין כל 1 שנייה. אבל שום, כלום. אין כלום.

    אתם באמת מאמינים שהטסלה בחלל? זה נראה כל כך פייק. אפילו אלון מאסק אמר בראיון לאחר השיגור על הטסלה בחלל:
    “you can tell it is real, because it looks fake, honestly”
    אמיתי – תבדוק זאת שהוא אמר

    אבל שוב, איך אתם יודעים שיש לווינים בחלל??? כי אומרים לנו?? למה אתה לא רואה אף לווין חג מסביב לארץ?

    וניסים – מה שאמרת על נצנוני לויינים מהארץ – מי אמר לך שזה לווין? מי אמר לך שזה מרוחק מאות ק”מ ממך? הידעת? נאסא משגרת 15-20 בלונים בשנה! לגובה 10-20 ק”מ! כל בלון שוקל 2 טון! יש לו ציוד של מטורולגיה, תקשורת, טלסקופ ועוד! כל בלון מחזיק בין מספר שבועות למספר חודשים! הידעת זאת? למה לכל הרוחות צריך בלונים אם יש אלפי לווינים בחלל? למה אף אחד מהאנושות לא יודע זאת?
    https://www.youtube.com/watch?v=i47s97wBujY&feature=youtu.be

    אין אף סיבה הגיונית למה לא רואים לויין או נצנוץ של לויין בצילום ה LIVE של הטסלה. אלא אם כן……

  200. משתמש אנונימי (לא מזוהה)
    הלווינים מאד רחוקים אחד מהשני, כך שהסיכוי לראות נצנוץ הוא מאד נמוך. הסתכל למעלה קצת אחרי שקיעה – רואים בקושי לווין או 2. דרך אגב – יש לווינים שרואים אותם לפעמים גם לפני השקיעה!

    זמן החשיפה של הצילומים האלה הוא נמוך, ולכן לא רואים כוכבים. נסה לצלם בלילה – גם לא תראה כוכבים.

  201. האם ראיתם את הטסלה שמרחפת מסביב לכדור הארץ?
    תגידו לי 2 דברים – למה לא רואים לוויינים או נצנוצי החזר-אור של לוויינים שרואים את המכונית על הרקע של כדור הארץ הרי יש אלפי לווינים מסביבנו, לא?
    ושאלה שנייה, שרואים את החלל העמוק על רקע המכונית – למה לא רואים כוכבים? רואים רק הכל שחור?
    ובנוסף על כך, למה שרואים את התמונות שתחנת החלפת משדרת גם לא רואים לווינים או נצנוצי החזר-אור של לוויינים?

  202. למה לא מצנחים:
    1. מצנחים שוקלים יותר מכמות הדלק הדרושה בשיטה הזו.
    2. אי אפשר לכוון מצנחים לנחיתה במיקום הדרוש. קצת בעייה בהנחתת משהו באורך של כ-40 מטרים ומשקל של 30 טונות.
    3. באמצעות מצנחים המשגר מגיע לקרקע במהירות רבה מדי, שקצת תעקום את השסי, או אפילו יותר מקצת.
    4. אם מצניחים את זה בים, אז כמובן שאי אפשר להגיע לאסדה. המשגר נרטב ומי המלח גורמים לקורזיה שלא תאפשר שיגור מחודש שלו.

  203. ליוני,
    אי אפשר להשתמש במונחים משום שהם לא עמידים מספיק. הטיל נע במהירות על קולית (יותר מ 1,200 קמ”ש!). מצנח שנפתח במהירות כזו נקרע מייד.

    בנוסף מצנחים לא מאפשרים נחיתה מדוייקת כמו שצריך עבור נחיתה על ספינה או כן נחיתה.

  204. לא מבין למה אין לו מצנחים? זה לא יותר פשוט? בטח זה טוב למקרה שנגמר הדלק.

  205. תודה רבה!

    כשצפיתי בשידור ממצלמות הרכב שמתי לב שעפים ממנו כל הזמן מעין פירורים / חלקיקים / ניצוצות כלפי מעלה, זה נמשך גם שעות אחרי השיגור והתגבר עם הזמן, יש לך מושג מה זה יכול להיות?

  206. יש לציין כי על צג מחשב הרכב כתוב “!Don’t Panic” באותיות גדולות ומסבירות פנים – המוטו של מדריך הטרמפיסט לגלקסיה.

    מעניין מה זאפוד ביבילברוקס היה אומר על זה.

כתיבת תגובה

האימייל לא יוצג באתר. שדות החובה מסומנים *

אתר זה עושה שימוש באקיזמט למניעת הודעות זבל. לחצו כאן כדי ללמוד איך נתוני התגובה שלכם מעובדים.